Vous êtes sur la page 1sur 234

AP PG Mock Part 1

Marking: +3 Marks for correct answer and -1 negative Marking for incorrect answer.

Test Duration (mins): 180

Time Left (mins) : 180

Instructions

Once the test has started, do not press the refresh button (or F5 on your keyboard)

Itis advisable to save the test regularly to avoid losing your information, save test refers to storing
the attempted part of the test.

In case of accidental failure of internet connectivity the system will save the attempted portion of
the test automatically.

Once the test time is over, you will be awarded a grace duration to wrap up the test and "Submit"
it.

Submit test refers to the final completion of test; once you submit the test you will not be able to
edit / preview your answers.

Please ensure that you are connected to the internet, while submitting the test.

Section

Part

(Q.1) Which of the following contributes to the development of the thoraco-abdominal


diaphragm ?

(a) Pleuropericardial membrane

(b) Dorsal mesentery of oesophagus

(c) Splanchnopleuric mesoderm

(d) Intermediate mesoderm

Your Response :

Correct Answer : b

Exp: Dorsal mesentery of esophagus

Development of the Diaphragm

The diaphragm is formed from the following structures: (a) the septum
transversum, which forms the muscle and central tendon; (b) the two
pleuroperitoneal membranes, which are largely responsible for the
peripheral areas of the diaphragmatic pleura and peritoneum that cover its
upper and lower surfaces, respectively; and (c) the dorsal mesentery of
the esophagus, in which the crura develop.

The septum transversum is a mass of mesoderm that is formed in the neck


by the fusion of the myotomes of the third, fourth, and fifth cervical
segments. With the descent of the heart from the neck to the thorax, the
septum is pushed caudally, pulling its nerve supply with it; thus, its motor
nerve supply is derived from the third, fourth, and fifth cervical nerves,
which are contained within the phrenic nerve.

The pleuroperitoneal membranes grow medially from the body wall on


each side until they fuse with the septum transversum anterior to the
esophagus and with the dorsal mesentery posterior to the esophagus.
During the process of fusion, the mesoderm of the septum transversum
extends into the other parts, forming all the muscles of the diaphragm.

The motor nerve supply to the entire muscle of the diaphragm is the
phrenic nerve. The central pleura on the upper surface of the diaphragm
and the peritoneum on the lower surface are also formed from the septum
transversum, which explains their sensory innervation from the phrenic
nerve. The sensory innervation of the peripheral parts of the pleura and
peritoneum covering the peripheral areas of the upper and lower surfaces
of the diaphragm is from the lower six thoracic nerves. This is
understandable, since the peripheral pleura and peritoneum from the
pleuroperitoneal membranes are derived from the body wall.

Diaphragmatic Herniae

Congenital herniae occur as the result of incomplete fusion of the septum


transversum, the dorsal mesentery, and the pleuroperitoneal membranes
from the body wall. The herniae occur at the following sites: (a) the
pleuroperitoneal canal (more common on the left side; caused by failure of
fusion of the septum transversum with the pleuroperitoneal membrane), (b)
the opening between the xiphoid and costal origins of the diaphragm, and
(c) the esophageal hiatus.

Acquired herniae may occur in middle-aged people with weak musculature


around the esophageal opening in the diaphragm. These herniae may be
either sliding or paraesophageal

(Q.2) Recurrent laryngeal nerve supplies following muscles EXCEPT

(a) Posterior cricoarytenoid

(b) Lateral cricoarytenoid


(c) Cricothyroid

(d) Obliquearytenoid

Your Response :

Correct Answer : c

Exp: C. Cricothyroid

Testing the Integrity of the Vagus Nerve :

The vagus nerve innervates many important organs, but the examination of
this nerve depends on testing the function of the branches to the pharynx,
soft palate, and larynx. The pharyngeal reflex may be tested by touching
the lateral wall of the pharynx with a spatula. This should immediately
cause the patient to that is, the pharyngeal muscles will contract.

The innervation of the soft palate can be tested by asking the patient to say

Normally, the soft palate rises and the uvula moves backward in the
midline.

All the muscles of the larynx are supplied by the recurrent laryngeal
branch of the vagus, except the cricothyroid muscle, which is supplied
by the external laryngeal branch of the superior laryngeal branch of
the vagus. Hoarseness or absence of the voice may occur. Laryngoscopic
examination may reveal abductor paralysis

(Q.3) First right posterior intercostal vein drains into

(a) Azygousvein

(b) Accessory azygous vein

(c) Superior intercostal vein

(d) Right brachio-cephalic vein

Your Response :

Correct Answer : d

Exp: D. Right brachio-cephalic vein

The posterior intercostal veins run with the posterior intercostal artery. It
passes on the underside of the rib and gives off a dorsal branch which
drains blood from the back muscles. They drain the intercostal spaces
posteriorly. There are eleven posterior intercostal veins on each side that
are usually arranged as: The 1st posterior intercostal vein drains into the
brachiocephalic vein. The 2nd and 3rd posterior intercostal veins drain into
the superior intercostal vein. The rest of the posterior intercostal veins
drain into the azygos vein on the right, or the hemiazygos vein on the left.

(Q.4) Gluteus maximus muscle is innervated by

(a) Superior gluteal nerve

(b) Inferior gluteal nerve

(c) Pudendal nerve

(d) Subcostal nerve

Your Response :

Correct Answer : b

Exp: B. Inferior gluteal nerve

Table: Muscles of the Gluteal Region

Muscle Origin Insertion Nerve Nerve Action


Supply Rootsa

Gluteus Outer surface of Iliotibial Inferior L5; S1, Extends and


maximus ilium, sacrum, tract and gluteal 2 laterally
coccyx, gluteal nerve rotates hip
sacrotuberous tuberosity joint; through
ligament of femur iliotibial tract,
it extends knee
joint

Gluteus Outer surface of Lateral Superior L5; S1 Abducts thigh


medius ilium surface of gluteal at hip joint;
greater nerve tilts pelvis
trochanter when walking
of femur to permit
opposite leg to
clear ground

Gluteus Outer surface of Anterior Superior L5; S1 Abducts thigh


minimus ilium surface of gluteal at hip joint;
greater nerve tilts pelvis
trochanter when walking
of femur to permit
opposite leg to
clear ground
Tensor Iliac crest Iliotibial Superior L4; 5 Assists gluteus
fasciae tract gluteal maximus in
latae nerve extending the
knee joint

Piriformis Anterior surface Upper First and L5; S1, Lateral rotator
of sacrum border of second 2 of thigh at hip
greater sacral joint
trochanter nerves
of femur

Obturator Inner surface of Upper Sacral L5; S1 Lateral rotator


internus obturator border of plexus of thigh at hip
membrane greater joint
trochanter
of femur

Gemellus Spine of ischium Upper Sacral L5; S1 Lateral rotator


superior border of plexus of thigh at hip
greater joint
trochanter
of femur

Gemellus Ischial Upper Sacral L5; S1 Lateral rotator


inferior tuberosity border of plexus of thigh at hip
greater joint
trochanter
of femur

Quadratus Lateral border of Quadrate Sacral L5; S1 Lateral rotator


femoris ischial tubercle of plexus of thigh at hip
tuberosity femur joint
a
The predominant nerve root supply is indicated by boldface type.

(Q.5) Which of the following structures pass between the External and Internal
carotidarteries ?

(a) Styloglossus

(b) Stylopharyngeus

(c) IXth Cranial nerve

(d) All of above

Your Response :
Correct Answer : d

Exp: D. All of above

ORDER OF ARRANGEMENT AT SPECIFIC SITES

FEMORAL TRIANGLE Within femoral sheath from medial to


lateral side- VAN (vein artery - nerve
)

Femoral canal- lymph node of cloquet/


Rosenmuller

Middle compartment- femoral vein

Lateral compartment- femoral artery


with femoral branch of genitofemoral
nerve

Outside of sheath and lateralmost-


femoral nerve

AT APEX OF FEMORAL From anterior to posterior


TRIANGLE
Femoral artery

Femoral vein

Profunda femoris artery and vein

POPLITEAL FOSSA Upper part: medial to lateral artery-


vein- tibial nerve

Middle part: superior to deep- nerve-


vein-artery

Lower part: medial to lateral- nerve-


vein-artery

ANTERIOR ASPECT OF Medial to lateral (The Himalayas Are


ANKLE THROUGH Never Dry Places )
EXTENSOR RETINACULUM
Tibial anterior muscle

Extensor Hallucis longus

Anterior tibial artery

Deep Peroneal nerve

Extensor Digitorum longus


Peroneus tertius

THROUGH FLEXOR Medial to lateral(Anterior to


RETINACULUM BEHIND posterior) -( The Doctors Are Not
MEDIAL MALLEOUS Here)

Tibialis posterior

Flexor Digitorum longus

Posterior tibial artery

Tibial nerve

Flexor Hallucis longus

CUBITAL FOSSA Medial to lateral (MBBR)

- Median nerve - Brachial artery -


Biceps tendon - Radial nerve

STRUCTURES WITHIN THE Medial to lateral


PAROTID GLAND
Arteries- Maxillary and Superficial
temporal

Retromandibular vein

Facial nerve

Parotid lymph node

THYROID GLAND Dense capillary plexus lies deep to the


capsule

Thyroid is removed along with the true


capsule

PROSTATE Capillary plexus is present between true


and false capsule

Prostate removed and both the capsules


are left behind

At aortic opening of Left to Right- - Aorta - Thoracic


Diaphragm(T12) duct - Azygous vein

ROOT OF LUNG From before backwards (SVAB) (same


on both sides)

Superior Pulmonary Vein


Pulmonary Artery

Bronchus

From Above downwards

RIGHT SIDE

- Eparterial Bronchus - Pulmonary


Artery

- Hyparterial branch - Inferior


pulmonary vein

LEFT SIDE (ABI)

- Pulmonary artery - Bronchus -


Inferior pulmonary vein

IN BRONCHOPULMONARY Pulmonary artery lies dorsolateral to


SEGMENT bronchus

AT HILUM Pulmonary veins are ventromedial to


bronchus

NECK OF 1ST RIB Related anteriorly from medial to


lateral side (SPIT)

- Sympathetic chain - 1st posterior


intercostal vein

- Intercostal artery - 1st thoracic


nerve

CAVERNOUS SINUS Lateral wall above downwards

3rd nerve

4th nerve

5th nerve

Trigeminal ganglion

Within centre of sinus

ICA with venous and symphatetic


plexus

Abducent nerve
STRUCTERS PASSING - Styloid process - Styloglossus -
BETWEEN EXTERNAL AND Stylopharyngeus
INTERNAL CAROTID
ARTERY - IXth nerve - Pharyngeal branch
of vagus nerve

- Part of parotid gland

STRUCTERS DEEP TO From above downwards (NiSHA)


POSTERIOR BORDER OF
HYOGLOSSUS - IX th nerve - Stylohyoid
ligament - Lingual artery

COSTAL GROOVE OF RIBS From above downwards(VAN)

- Posterior intercostal vein - Posterior


intercostal artery - Intercostal nerve

AT COMMON CAROTID Internal carotid artery is posterolateral


ARTERY BIFURCATION to external carotid artery

IN FRONT OF ALA OF (SLIO)


SACRUM
- Symphatetic trunk - Lumbosacral
trunk

- Iliolumbar artery - Obturator nerve

HILUM OF KIDNEY Anterior to posterior(VAP) - Vein -


Artery - Pelvis

STRUCTURES PIERCING (TALC)


CLAVIPECTORAL FASCIA
Thoracoacromial vein

Lateral pectoral nerve

Cephalic vein

AT PORTA HEPATIS Anterior to posterior (DAV) - CBD -


Hepatic artery - Portal vein

IN RADIAL/SPIRAL GROOVE Radial nerve


OF HUMERUS
Profunda brachial artery

INBETWEEN SUPERIOR Sinus of Morgagni


CONSTRICTOR AND BASE OF
SKULL Auditory tube

Levator palatine MUSCLE nerve


INBETWEEN SUPERIOR AND Stylopharyngeus muscle
MIDDLE CONSTRICTOR
Glossopharyngeal nerve

Ascending palatine artery

INBETWEEN MIDDLE AND Internal laryngeal nerve


INFERIOR CONSTRICTOR
Superior laryngeal vessels

INBETWEEN INFERIOR - Recurrent laryngeal nerve - Inferior


CONSTRICTOR AND laryngeal vessels
ESOPHAGUS

Greater sciatic foramen 1. Pyriformis 2. Superior and inferior


(structures passing through) gluteal nerve and vessels

3. Sciatic nerve 4. Pudendal


nerve

5. Internal pudendal vessels 6.


Posterior cutaneous nerve of thigh

7. Nerve to quadratus femoris 8.


Nerve to obturator internus.

Lesser sciatic foramen 1. Tendon of obturator internus 2.


(structures passing through) Nerve to obturator internus

3. Pudendal nerve 4.
Internal pudendal vessels

5. Origin of gemelli

(Q.6) Largestcommissure of the brain is ?

(a) Anterior commissure

(b) Posterior commissure

(c) Corpus callosum

(d) None of the above

Your Response :

Correct Answer : c

Exp: (C). Corpus callosum

The -est list in Anatomy :


Largest and deepest cranial fossa Posterior

Thickest/strongest orbital wall Lateral

Thinnest/weakest Medial

Farthest and deepest quadrant of Antero-inferior


tympanic membrane

Smallest and most numerous lingual Filiform


papillae

Largest diploic vein Occipital diploic vein

Largest terminal branch of Frontal nerve


ophthalmic nerve

Smallest terminal branch of Lacrimal nerve


ophthalmic nerve

Largest of the sympathetic cardiac Middle cervical cardiac branch


branches

Largest scalenous muscle S.medius

Smallest scalenous muscle S.posterior

Thickest intervertebral disc At lumbar region

Thinnest intervertebral disc At superior thoracic region

Largest papillary muscle of rt. Anterior papillary muscle


Ventricle

Rib with the largest obliquity 9th rib

Shortest,broadest and most curved 1st rib


rib

Largest rib 7th rib

Thickest and toughest of the Duramater


meninges

Largest branch of vertebral artery PICA

Largest branch of ICA MCA

Largest horn of lateral ventricle Inferior horn


Largest commissure of brain Corpus callosum

Thickest part of corpus callosum Splenium

Thickest cutaneous nerve Greater occipital nerve

Largest organnele in eukaryotes Endoplasmic reticulum

Male urethra : Shortest part Membranous

Male urethra : Longest part Penile

Male urethra : widest and most Prostatic


dilatable part

Male urethra : Narrowest and least Urethral orifice


dilatable part

Strongest ligament of the body Ilio-femoral/Bigelow ligament

Largest peripheral parasympathetic Sphenopalatine ganglion


ganglion

Largest bursa of the body Subacromial bursa

Largest joint Knee

Longest muscle Sartorius

Largest branch of coeliac trunk Splenic artery

Smallest bone and muscle Stapes and stapedius

Largest sesamoid bone Patella

Longest vein GSV

(Q.7) False regarding Sympathetic nervoussystem ?

(a) Adrenal medulla is essentially a sympathetic ganglion.

(b) Noradrenaline is major transmitter

(c) Arises from Dorso-lumbar region of cord

(d) Sympathetic ganglia are located on the target organ

Your Response :

Correct Answer : d
Exp: (D) Sympathetic ganglia are located on the target organ

ANS (Autonomic Nervous system) Parasympathetic and


sympathetic

Chemical transmission at autonomic junction: -(between pre and post-


ganglionic neuron; and between the postganglion neurons and the
autonomic effectors) The principal transmitter agents involved are
Acetyl choline and Non-epinephrine, although Dopamine is also secreted
by interneurons in the sympathetic ganglia and GnRH is secreted by some
of the preganglionic neurons

On the basis of the chemical mediator, ANS is divided into: I-


Cholinergic division (Ach transmitter) And II- Nor-adrenergic division
(Nor Ad transmitter)

Examples of cholinergic neurons:- (Ach transmitter)

All preganglionic neuron Q

Parasympathetic postganglionic neurons Q

Sympathetic post ganglion neurons which innervate sweat gland Q

Sympathetic neurons which ends in the blood vessels (= produce


vasodilation) Q

Remaining post ganglionic sympathetic neurons are Noradrenergic Q

The highest seat regulating ANS is in hypothalmus; posterior and lateral


nuclei and primarily sympathetic while anterior and medial nuclei are
primarily parasympathetic

ICA cells (intrinsic cardiac Adrenergic cells) Epinephrine (Adenergic)


arid norepinephrine.

The adrenal medulla is essentially a sympathetic ganglion in which the


post ganglion cells have lost their axons and secrete norepinephrine,
epinephrine, and some Dopamine directly into the blood stream.

Difference between sympathetic and paraysmpathetic division of -

Trait Sympathetic Parasympathetic

1. Origin Q Dorso-lumbar (T1 to Craniosacral (III, VII,


L2 or L3) IX, X, S2-S4)

2. Ganglia Q Away from the organ On or close to organ

3. Distribution Q Wide Limited to head, neck and


trunk
4. Post ganglionic fibers Long Short
Q

5. Pre: Postganglionic 1:20 to 1:100 11 to 1:2 (except in


fibers ratio Q enteric plexus)

6. Transmitter Q Nor adrenaline (major) Acetyl choline

Acetyl choline (Minor)

7. Function Q Tacking stress and Assimilation of food


emergency conservation of energy

(Q.8) Thyroid develops from:

(a) Endoderm of thyroglossal duct

(b) First arch

(c) Second arch

(d) Third arch

Your Response :

Correct Answer : a

Exp: (A). Endoderm of thyroglossal duct

Embryology Of Thyroid Gland

The thyroid develops from a median endodermal thyroid diverticulum


which grows down infront of the neck from the floor of the primitive
pharynx, just caudal to the tuberculum impar. The lower end of the
diverticulum enlarges to form the gland.

The rest of the diverticulum remains narrow and is known as the


thyroglossal duct. Most of the duct soon disappears.

The position of the upper end is marked by the foramen caecum of the
tongue, and the lower end often persists as the pyramidal lobe.

The thyroid gland descends early in foetal life from the base of the tongue
towards its position in the lower neck with the isthmus lying over the
second and third tracheal ring.

At the time of its descent the hyoid bone has not been formed and the track
of the descent of the thyroid gland is variable passing in front, through or
behind the eventual position of the hyoid body.

Thyroglossal duct cysts represent a persistence of this track and may


therefore be found anywhere in or adjacent to the midline from the tongue
base to the thyroid isthmus.

Rarely, a thyroglossal cyst may be the only functioning thyroid tissue in


the body.

HEAD AND NECK : VISCERA/ORGAN DEVELOPMENT - A


COMPREHENSIVE ACCOUNT :

Tongue Appears at 4th week.

Musculature derived from mesoderm of occipital


somites. Precursor muscles cells migrate to region of
tongue and are innervated by general sensory efferent
fibers of CN XII.

Mucosa derived from anterior endoderm lining arches


1-4; accordingly, innervation depends on arch
derivation:

Mucosa of anterior 2/3 of tongue comes from


the first arch CN V

Mucosa of posterior 1/3 of tongue comes from


third and forth arch CN IX, X

Special taste of anterior 2/3 of tongue comes from CN


VII.

Special taste of posterior 1/3 of tongue comes from CN


X.

Tongue freed from floor of mouth by extensive


degeneration of underlying tissue. Midline frenulum
continues to anchor tongue to floor of mouth.

Thyroid Develops as ingrowth of mucosal epithelium located in


Gland the midline of the tongue (at foramen cecum). It
descends along front of pharyngeal gut, but remains
connected to tongue by thyrooglossal duct, which is
obliterated later in development. Thyroid gland
descends to a point just caudal to laryngeal cartilages.

Facial a) medial nasal prominence forms midline of nose,


structures philtrum and primary palate
(general)
b) lateral nasal prominence forms alae of nose

c) maxillary prominence forms cheek region and lateral


lip
d) clefts can form at inter-prominence fusion lines

Nose At the time of anterior neural tube closure,


mesenchyme around forebrain, frontonasal prominence
(FNP), has smooth rounded extended contour. Nasal
placodes (thickening of surface ectoderm to become
peripheral neural tissue) develop on frontolateral
aspects of FNP. Mesenchyme swells around nasal
placode producing a medial and lateral nasal
prominence (nasomedial and nasolateral
processes). These nasal prominences form the nose.

Mouth Stomadeum (primitive oral cavity) forms between


frontonasal prominence and first pharyngeal arch. The
first pharyngeal arch forms the dorsal maxillary
prominence and ventral mandibular prominence. The
maxillary prominence will merge with medial nasal
prominences, pushing them closer to cause
fusion. Fused medial nasal prominences will form
midline of nose and midline of upper lip (philtrum) and
primary palate (first 4 teeth).

Nasolacrimal Maxillary and lateral nasal prominences are separated


structures by deep furrow, the nasolacrimal groove. Ectoderm in
floor of groove forms epithelial cord, which detaches
from overlying ectoderm. The epithelial cord canalizes
to form the nasolacrimal duct. The upper end of the
duct widens to form the lacrimal sac. After detachment
of the cord, the maxillary and lateral nasal prominences
merge with each other, resulting in the formation of a
nasolacrimal duct that runs from the medial corner of
the eye to the inferior meatus of the nasal cavity.

The maxillary prominences enlarge to form the cheeks


and maxillae.

The lateral nasal prominences form the alae of the


nose.

Secondary (hard) Main part of definitive palate formed by two palatine


palate shelves derived from intraoral bilateral extensions of
the maxillary prominences. These appear at the 6th
week. They are directed obliquely downward on each
side of the tongue; they move down when mandible
gets bigger.

At the seventh week, they ascend to attain a horizontal


position, then fuse to form the secondary palate. At the
time the palatine shelves fuse, the nasal septum (an
outgrowth of median tissue of the frontonasal
prominence) grows down and joins the cephalic aspect
of the newly formed palate

Anteriorly, shelves fuse with triangular primary


palate. The incisive foramen marks the midline
between the primary and secondary palate.

External Ear The auricle is derived from 6 auricular hillocks


(mesenchymal proliferations) along the dorsal aspect of
arches 1 (top of ear) and 2 (bottom of ear). These fuse
to form the definitive auricle. At the mandible grows,
the ear is pushed upward and backward from its initial
horizontal position on the neck.

The EAM is derived from the 1st pharyngeal arch.

The eardrum (tympanic membrane) is composed of 3


layers of cells: 1) ectodermal epithelial lining of
bottom of EAM; 2) endodermal epithelium lining of
tympanic cavity; 3) intermediate layer of connective
tissue.

The eardrum is composed of multiple cell layers


because it represents the first pharyngeal membrane,
and thus lies at the junction of the first pharyngeal
pouch and cleft.

Middle Ear The middle ear consists of an auditory tube (from the
1st pharyngeal pouch, along with tympanic cavity) and
the ossicles (from pharyngeal arches 1 and 2
cartilage).

The first arch cartilage forms the malleus and


incus. The tensor tympani (muscle of the malleus) is
derived from the fourth somitomere (associated with
the first arch) and is therefore innervated by CN V.

The second arch cartilage forms the stapes. The


stapedius (muscles of the stapes) is derived from the
sixth somitomere (associated with the second arch) and
is therefore innervated by CN VII.

The ossicles are initially embedded in mesenchyme,


but in the 8th month, the mesenchyme degenerates and
an endodermal epithelial lining of the tympanic cavity
envelops the ossicles and connects them to the wall of
the cavity in a mesentery-like fashion.
Inner Ear The inner ear is derived thickening of surface ectoderm
on both sides of the hindbrain (otic placodes). The
placodes invaginate to form otic vesicles
(otocytes). The vesicles then divide into ventral and
dorsal components.

The ventral component forms the saccule and cochlear


duct.

The dorsal component forms the utricle and


semicircular canals and endolymphatic duct.

Cochlear Duct Derived from an outgrowth of the saccule during the 6th
week. The outgrowth penetrates the surrounding
mesenchyme in a spiral fashion. The surrounding
mesenchyme forms the cartilage and undergoes
vacuolization.

The scala vestibule and scale tympani form and


surround the cochlear duct. They are filled with
periplymp to receive mechanical vibrations of ossicles.
The mechanical stimuli activates sensory (ciliary) cells
in the cochlear duct.

Semicircular The utricle is initially three flattened outpocketings,


canals which lose the central core. From this three
semicircular canals are forms, each at 90 degree angles
from one another. Sensory cells arise in the ampulla at
one end of each canal, in the utricle and saccule.

Eye At week 4, two depressions are evident on each of the


forebrain hemispheres. As the anterior neural fold
closes, the optic pits elongate to form the optic
vesicles. The optic vesicles remain connected to the
forebrain by optic stalks.

The invagination of the optic vesicles forms a bilayered


optic cup. The bilayered cup becomes the dual layered
retina (neural and pigmented layer)

Surface ectoderm forms the lens placode, which


invaginates with the optic cup.

The optic stalk is deficient ventrally to contain


choroids fissure to allow blood vessels into the eye
(hyaloid artery). The artery feeds the growing lens, but
will its distal portion will eventually degenerate such
that the adult lens receives no hyaloid vasculature.
At the 7th week, the choroids fissure closes and walls
fuse as the retinal nerve get bigger.

The anterior rim of the optic vesicles forms the retina


and iris. The iris is an outgrowth of the distal edge of
the retina.

Optic vesicles induces/maintains the development of


the lens vesicle, which forms the definitive
lens. Following separation of the lens vesicle from the
surface ectoderm, the cornea develops in the anterior
1/5th of the eye.

The lens and retina are surrounded by mesenchyme


which forms a tough connective tissue, the sclera, that
is continuous with the dura mater around the optic
nerve.

Iridopupillary membrane forms to separate the anterior


and posterior chambers of the eye. The membrane
breaks down to allow for the pupil

Mesenchyme surrounding the forming eye forms


musculature (ciliary muscles and pupillary muscles
from somitomeres 1 and 2; innervated by CN III),
supportive connective tissue elements and vasculature.

Eyelids Formed by an outgrowth of ectoderm that is fused at its


midline in the 2nd trimester, but later reopen.

(Q.9) What is the clearance of a substance when its concentration in the plasma is 10 mg/dL,
its concentration in the urine is 100 mg/dL, and urine flow is 2 mL/min ?

(a) 10 mL/min

(b) 12 mL/min

(c) 20 mL/min

(d) 25 mL/min

Your Response :

Correct Answer : c

Exp: Ans C. 20 mL/min

When referring to the function of thekidney, clearance is considered to be


theamount of liquid filtered out of the blood that gets processed by
thekidneysorthe amount of blood cleaned per timebecause it has the units
of avolumetric flow rate[volume/time]. However, it does not refer to a real
value; "the kidney does not completely remove a substance from the total
renal plasma flow. From amass transferperspective andphysiologically,
volumetric blood flow (to the dialysis machine and/or kidney) is only one
of several factors that determine blood concentration and removal of a
substance from the body. Other factors include themass transfer
coefficient, dialysate flow and dialysate recirculation flow for
hemodialysis, and theglomerular filtration rateand thetubularreabsorption
rate, for the kidney. A physiologic interpretation of clearance (at steady-
state) is that clearance isa ratio of the mass generation and blood
(orplasma) concentration.

Measurement of renal clearance : Renal clearance can be measured with a


timed collection ofurineand an analysis of its composition with the aid of
the following equation (which follows directly from the derivation of ):

Where:

K is the clearance [mL/min]

CU is the urine concentration [mmol/L]

Q is the urine flow (volume/time) [mL/min]

CB is the plasma concentration [mmol/L]

When the substance "C" is creatinine, an endogenous chemical that is


excreted only by filtration, the calculated clearance is equivalent to
theglomerular filtration rate.Inulin clearance is also used to estimate
glomerular filtration rate.

Note- the above equation is validonlyfor the steady-state condition. If the


substance being cleared isnotat a constant plasma concentration (i.e.notat
steady-state)Kmust be obtained from the (full) solution of the differential
equation

So here

K = 100 x 2 / 10 = 200/ 10 = 20 mL/min.

(Q.10) The dicrotic notch on the aortic pressure curve is caused by the closure of

(a) Mitral valve

(b) tricuspid valve


(c) Aortic valve

(d) pulmonary valve

Your Response :

Correct Answer : c

Exp: Ans C. Aortic valve.

The Normal Arterial Line Waveform :

The arterial pressure wave is a shockwave; it travels much faster than the
actual blood which is ejected. It represents the impulse of left ventricular
contraction, conducted though the aortic valve and vessels along a fluid
column (of blood), then up a catheter, then up another fluid column (of
hard tubing) and finally into Wheatstone bridge transducer. It has the
following components :

Systolic Upstroke :

This is the ventricular ejection.

The slope of this segment has some vague relationship with the rate of
flow through the aortic valve (probably more so when measured in the
actual aorta). When its slope is slurred, there may be aortic stenosis.

Peak Sytolic Pressure :

This is the maximum pressure generated during the systolic ejection.

Added to it is the reflected pressure from the rest of the vascular tree.

If the rest of the vascular tree is hardened and atheromatous, its poor
compliance causes a powerful reflected wave, which when added to the
systolic effort of the ventricle makes for a high peak systolic pressure.
The peak systolic pressure is what you bleed with.This is the pressure
that blows the hemostatic thrombus plugs off the vessels you have so
carefully cauterised, and stresses the wall of the fragile aneurysm.

Systolic Decline :

This is the rapid declinein arterial pressure as the ventricular contraction


comes to an end. This decline is even more rapid when there is a left
ventricular outflow tract obstruction (and systole comes to an abrupt halt
before the left ventricle is finished with the ejection).

Mean Arterial Pressure :

MAP is roughly equivalent to the area under the arterial pressure curve,
divided by the duration of the beat and averaged over several beats.
The MAP is what perfuses your organs, and what commands their blood
flow autoregulation.

Dicrotic Notch :

In perfect circumstances, when measured in the aorta, this notch is very


sharp and it actually does represent the closing of the aortic valve. As
you move further outAs mentioned below, the dicrotic notch position
varies with the position of the arterial line. A suspiciously low dicrotic
notch could mean very poor vascular resistance,eg. in a situation like
severe septic shock.

Diastolic Run-off :

This is the rapid declinein arterial pressure as the ventricular contraction


comes to an end.

End Diastolic Pressure :

This is the pressure exerted by the vascular tree back upon the aortic
valve. Hardened non-compliant vessels will cause this pressure to be
raised. Soft vasoplegic vessels of a septic patient will offer little
resistance, and the diastolic pressure will be lower.A regurgitant aortic
valve will cause this pressure to be lower than normal, because instead of
meeting the aortic valve the pressure wave travels all the way though to
the ventricle via the regurgitant jet. The diastolic pressure is what fills
your coronary arteries, and should not be ignored.

Pulse pressure :

A very widened pulse pressuresuggests aortic regurgitation (as in


diastole, the arterial pressure drops to fill the left ventricle though the
regurgitating aortic valve) A very narrow pulse pressuresuggests
cardiac tamponade, or any other sort of low output state (eg. severe
cardiogenic shock, massive pulmonary embolism or tension
pneumothorax)

(Q.11) 1b type of nerve fiberscarry afferent impulses from

(a) Muscle spindles

(b) Pacinian corpuscles

(c) Golgi tendon organs

(d) Nociceptors

Your Response :
Correct Answer : c

Exp: Ans C. Golgi tendon organs

Erlanger and Gasser divided mammalian nerve fibers into A, B, and C


groups, further subdividing the A group into alpha, beta, gamma, and
deltafibers. In the following table, the various fiber types are listed with their
diameters, electrical characteristics, and functions. By comparing the
neurologic deficits produced by careful dorsal root section and other nerve-
cutting experiments with the histologic changes in the nerves, the functions
and histologic characteristics of each of the families of axons responsible for
the various peaks of the compound action potential have been established. In
general, the greater the diameter of a given nerve fiber, the greater its speed
of conduction. The large axons are concerned primarily with proprioceptive
sensation, somatic motor function, conscious touch, and pressure, while the
smaller axons subserve pain and temperature sensations and autonomic
function. The dorsal root C fibers conduct some impulses generated by touch
and other cutaneous receptors in addition to impulses generated by pain and
temperature receptors.

Further research has shown that not all the classically described lettered
components are homogeneous, and a numerical system (Ia, Ib, II, III, IV) has
been used by some physiologists to classify sensory fibers. Unfortunately,
this has led to confusion.

In addition to variations in speed of conduction and fiber diameter, the


various classes of fibers in peripheral nerves differ in their sensitivity to
hypoxia and anesthetics. This fact has clinical as well as physiologic
significance. Local anesthetics depress transmission in the group C fibers
before they affect group A touch fibers. Conversely, pressure on a nerve can
cause loss of conduction in large-diameter motor, touch, and pressure fibers
while pain sensation remains relatively intact. Patterns of this type are
sometimes seen in individuals who sleep with their arms under their heads
for long periods, causing compression of the nerves in the arms. Because of
the association of deep sleep with alcoholic intoxication, the syndrome is
most common on weekends and has acquired the interesting name Saturday
night or Sunday morning paralysis.

Nerve Fiber Types in Mammalian Nerve.

Susceptibility to: Most Susceptible Intermediate Least


Susceptible

Hypoxia B A C

Pressure A B C
Local anesthetics C B A

Nerve Fiber Types in Mammalian Nerve.

Fiber Type Function Fiber Conduction Spike Absolute


Diameter Velocity Duration Refractory
(micro- (m/s) (ms) Period
m) (ms)

Proprioception; 1220 70120


somatic motor

Touch, 512 3070 0.40.5 0.41


pressure

Motor to 36 1530
muscle
spindles

Pain, cold, 25 1230


touch

B Preganglionic <3 315 1.2 1.2


autonomic

Dorsal root Pain, 0.41.2 0.52 2 2


temperature,
some
mechano-
reception

Sympathetic Postganglionic 0.31.3 0.72.3 2 2


sympathetic

Relative Susceptibility of Mammalian A, B, and C Nerve Fibers to


Conduction Block Produced by Various Agents.

Number Origin Fiber Type

Ia Muscle spindle, annulo-spiral ending


A
Ib Golgi tendon organ
A

II Muscle spindle, flower-spray ending; touch,


pressure A

III Pain and cold receptors; some touch receptors


A

IV Pain, temperature, and other receptors Dorsal root C

(Q.12) Resting membrane potential in mammalian spinal motor neurone is close to


equilibrium potential of

(a) Na+

(b) Cl

(c) K+

(d) Mg++

Your Response :

Correct Answer : b

Exp: Ans B. Cl-

The soma of a spinal motor neuron has aresting membrane potentialof


about 65 millivolts. This is somewhat less negative than the 90
millivolts found in large peripheral nerve fibers and in skeletal muscle
fibers; the lower voltage is important because it allows both positive and
negative control of the degree of excitability of the neuron. That is,
decreasing the voltage to a less negative value makes the membrane of the
neuron more excitable, whereas increasing this voltage to a more negative
value makes the neuron less excitable. T

If the concentration gradient for a given ion is known, the equilibrium


potential for that ion can be calculated (using the Nernst Equation).
Shown below are the equilibrium potentials calculated for K+, Na+ and
Cl- using the ionic concentrations for a typical neuron.

Notice that the equilibrium potential for Cl- is near the resting
membrane potential of the cell. This is because no ATP energy is being
expended to keep it out of equilibrium. For this reason, the resting
membrane potential of a cell will cause the concentration of Cl- inside the
cell to remain at a much lower level so that the concentration gradient
(inward) and electrical potential difference (outward) have equal and
opposite effects.

EQUIL. POT.

K+ -90 mV

Na+ +60 mV

Cl- -70 mV

(Q.13) Sperms develop the capability of motility in the

(a) Seminiferous tubules

(b) Epididymis

(c) Vas deferens

(d) Female genital tract

Your Response :

Correct Answer : b

Exp: B. Epididymis

Spermatogenesis

Spermatogonia, the primitive germ cells next to the basal lamina of the
seminiferous tubules, mature into primary spermatocytes. This process
begins during adolescence. The primary spermatocytes undergo meiotic
division, reducing the number of chromosomes. In this two-stage process,
they divide into secondary spermatocytes and then into spermatids,
which contain the haploid number of 23 chromosomes. The spermatids
mature into spermatozoa (sperms). As a single spermatogonium divides
and matures, its descendants remain tied together by cytoplasmic bridges
until the late spermatid stage. This apparently ensures synchrony of the
differentiation of each clone of germ cells. The estimated number of
spermatids formed from a single spermatogonium is 512. In humans, it
takes an average of 74 d to form a mature sperm from a primitive germ
cell by this orderly process of spermatogenesis.

Each sperm is an intricate motile cell, rich in DNA, with a head that is
made up mostly of chromosomal material. Covering the head like a cap is
the acrosome, a lysosome-like organelle rich in enzymes involved in
sperm penetration of the ovum and other events involved in fertilization.
The motile tail of the sperm is wrapped in its proximal portion by a sheath
holding numerous mitochondria. The membranes of late spermatids and
spermatozoa contain a special small form of angiotensin-converting
enzyme called germinal angiotensin-converting enzyme. The function
of this enzyme in the sperms is unknown, although male mice in which
the function of the angiotensin-converting enzyme gene has been
disrupted have reduced fertility.

The spermatids mature into spermatozoa in deep folds of the cytoplasm of


the Sertoli cells. Mature spermatozoa are released from the Sertoli cells
and become free in the lumens of the tubules. The Sertoli cells secrete
androgen-binding protein (ABP), inhibin, and MIS. They do not
synthesize androgens, but they contain aromatase (CYP19), the enzyme
responsible for conversion of androgens to estrogens, and they can
produce estrogens. ABP probably functions to maintain a high, stable
supply of androgen in the tubular fluid. Inhibin inhibits FSH secretion.

FSH and androgens maintain the gametogenic function of the testis. After
hypophysectomy, injection of LH produces a high local concentration of
androgen in the testes, and this maintains spermatogenesis. The stages
from spermatogonia to spermatids appear to be androgen-independent.
However, the maturation from spermatids to spermatozoa depends on
androgen acting on the Sertoli cells in which the developing spermatozoa
are embedded. FSH acts on the Sertoli cells to facilitate the last stages of
spermatid maturation. In addition, it promotes the production of ABP.

An interesting observation is that the estrogen content of the fluid in the


rete testis is high, and the walls of the rete contain numerous ER estrogen
receptors. In this region, fluid is reabsorbed and the spermatozoa are
concentrated. If this does not occur, the sperm entering the epididymis are
diluted in a large volume of fluid, and infertility results.

Further Development of Spermatozoa

Spermatozoa leaving the testes are not fully mobile. They continue
their maturation and acquire motility during their passage through
the epididymis. Motility is obviously important in vivo, but fertilization
occurs in vitro if an immotile spermatozoon from the head of the
epididymis is microinjected directly into an ovum. The ability to move
forward (progressive motility), which is acquired in the epididymis,
involves activation of a unique protein called CatSper, which is localized
to the principal piece of the sperm tail.

(Q.14) After a carbohydrate rich meal insulin secretion is stimulated by

(a) Cholecystokinin

(b) VIP

(c) GRP

(d) GLP1
Your Response :

Correct Answer : d

Exp: D. GLP-1

GIP

GIP contains 42 amino acid residues and is produced by K cells in the


mucosa of the duodenum and jejunum. Its secretion is stimulated by
glucose and fat in the duodenum, and because in large doses it inhibits
gastric secretion and motility, it was named gastric inhibitory peptide.
However, it now appears that it does not have significant gastric inhibiting
activity when administered in smaller amounts comparable to those seen
after a meal. In the meantime, it was found that GIP stimulates insulin
secretion. Gastrin, CCK, secretin, and glucagon also have this effect, but
GIP is the only one of these that stimulates insulin secretion when
administered in doses that produce blood levels comparable to those
produced by oral glucose. For this reason, it is often called glucose-
dependent insulinotropic polypeptide. The glucagon derivative GLP-
1 also stimulates insulin secretion and is said to be more potent in
this regard than GIP. Therefore, it may also be a physiologic B cell-
stimulating hormone of the gastrointestinal tract.

Orally administered glucose exerts a greater insulin-stimulating effect


than intravenously administered glucose, and orally administered amino
acids also produce a greater insulin response than intravenous amino
acids. These observations led to exploration of the possibility that a
substance secreted by the gastrointestinal mucosa stimulated insulin
secretion. Glucagon, glucagon derivatives, secretin, cholecystokinin
(CCK), gastrin, and gastric inhibitory peptide (GIP) all have such an
action, and CCK potentiates the insulin-stimulating effects of amino
acids. However, GIP is the only one of these peptides that produces
stimulation when administered in doses that reflect blood GIP levels
produced by an oral glucose load.

Recently, attention has focused on glucagon-like polypeptide 1 (GLP-1 )


as an additional gut factor that stimulates insulin secretion. This
polypeptide is a product of preproglucagon.

B cells have GLP-1 receptors as well as GIP receptors, and GLP-1 is a


more potent insulinotropic hormone than GIP. GIP and GLP-1 both
appear to act by increasing Ca2+ influx through voltage-gated Ca2+
channels.

Factors Affecting Insulin Secretion.

Stimulators Inhibitors
Glucose Somatostatin

Mannose 2-Deoxyglucose

Amino acids (leucine, arginine, Mannoheptulose


others)

Intestinal hormones (GIP, GLP-1 [7 alpha-Adrenergic stimulators


36], gastrin, secretin, CCK; others?) (norepinephrine, epinephrine)

beta-Keto acids alpha-Adrenergic blockers


(propranolol)
Acetylcholine

Glucagon Galanin

Cyclic AMP and various cAMP- Diazoxide


generating substances
Thiazide diuretics

beta-Adrenergic stimulators K+ depletion

Theophylline Phenytoin

Sulfonylureas Alloxan

Microtubule inhibitors

Insulin

(Q.15) Father of Physiologyis?

(a) Herophilis of Chalcedon

(b) Claude Bernard

(c) Louis Pasteur

(d) Robin Virchow.

Your Response :

Correct Answer : b

Exp: Ans B. Claude Bernard

Father of Anatomy Herophilus of Chalcedon

Father of Physiology Claude Bernard


Father of Microbiology Louis Pasteur

Father of Bacteriology Sir Robert Koch

Father of Chemotherapy Paul Ehrlich

Father of Pharmacology Oswald Schmiedberg

Father of Modern Pathology Rudolf Virchow

Father of Forensic Medicine Bernard Spillsbury

Father of Modern Epidemiology John Snow

Father of Occupational Health Ramazzini

Father of Modern Medicine Hippocrates

Father of Indian Medicine Charaka

Father of American Medicine William Osler

Father of Modern Surgery Ambroise Pare

Father of Indian Surgery Sushruta

Father of Antiseptic Surgery Joseph Lister

Father of Modern Inguinal Hernia Edoardo Bassini


Surgery

Father of Thyroid Surgery Emil Kocher

Father of Genetics Gregor Mendel

Father of Modern Human Genetics Dr Victor A. McKusick

Father of Psychoanalysis Sigmund Fruend

Father of Porphyrin chemistry Hans Fischer

Father of PTCA Andreas Gruntzig

Father of Modern Psychiatry Johann Weyer

Father of American Psychiatry Benjamin Rush

Father of Biology Aristotle

Father of Diagnostic Imaging Roentgen


(Q.16) Which of the following plays a very important role in growth during
fetaldevelopment ?

(a) Insulin

(b) IGF I

(c) IGF II

(d) All of the above

Your Response :

Correct Answer : c

Exp: Comparison of Insulin and the Insulin-Like Growth


Factors.

Insulin IGF-I IGF-II

Other names ... Somatomedin C Multiplication-


stimulating
activity (MSA)

Number of 51 70 67
amino acids

Source Pancreatic B Liver and other tissues Diverse tissues


cells

Level regulated Glucose Growth hormone after Unknown


by birth, nutritional status

Plasma levels 0.32 ng/mL 10700 ng/mL; peaks at 300800 ng/mL


puberty

Plasma- No Yes Yes


binding
proteins

Major Control of Skeletal and cartilage Growth during


physiologic metabolism growth fetal
role development

(Q.17) Stimulation of the sympathetic nerves to the normal heart

(a) Increases duration of the TP interval


(b) Increases the duration of the PR interval

(c) Decreases the duration of the QT interval

(d) Leads to fewer P waves than QRS complexes

Your Response :

Correct Answer : c

Exp: Ans C. Decreases the duration of the QT interval

Stimulation of the sympathetic nerves to the normal heart decreases the


duration of the ventricular action potential and, therefore, decreases the
QT interval. As heart rate increases, the duration of diastole and,
therefore, the TP interval decreases. Increased conduction velocity in the
AV node decreases the duration of the PR interval. Fewer P waves than
QRS complexes are indicative of AV block. On the contrary, sympathetic
stimulation may reverse AV block. The frequency of QRS complexes
increases with the heart rate.

ECG Intervals.

Normal
Durations

Intervals Average Range Events in the Heart during


Interval

PR intervala 0.18b 0.12 Atrial depolarization and


0.20 conduction through AV node

QRS duration 0.08 to 0.10 Ventricular depolarization and


atrial repolarization

QT interval 0.40 to 0.43 Ventricular depolarization plus


ventricular repolarization

ST interval (QT 0.32 ... Ventricular repolarization (during


minus QRS) T wave)

(Q.18) Formation of Okasaki fragments occurs in

(a) Transcription

(b) Replication

(c) Translation
(d) Reverse transcription

Your Response :

Correct Answer : b

Exp: B. Replication

Formation of the Replication Fork

A replication fork consists of four components that form in the following


sequence: (1) the DNA helicase unwinds a short segment of the parental
duplex DNA; (2) a primase initiates synthesis of an RNA molecule that is
essential for priming DNA synthesis; (3) the DNA polymerase initiates
nascent, daughter strand synthesis; and (4) SSBs bind to ssDNA and
prevent premature reannealing of ssDNA to dsDNA.

The polymerase III enzyme (the dnaE gene product in E coli ) binds to
template DNA as part of a multiprotein complex that consists of several
polymerase accessory factors. DNA polymerases only synthesize DNA in
the 5' to 3' direction, and only one of the several different types of
polymerases is involved at the replication fork. Because the DNA strands
are antiparallel (Chapter 34), the polymerase functions asymmetrically.
On the leading (forward) strand, the DNA is synthesized continuously.
On the lagging (retrograde) strand, the DNA is synthesized in short (1
5 kb;) fragments, the so-called Okazaki fragments. Several Okazaki
fragments (up to a thousand) must be sequentially synthesized for each
replication fork. To ensure that this happens, the helicase acts on the
lagging strand to unwind dsDNA in a 5' to 3' direction. The helicase
associates with the primase to afford the latter proper access to the
template. This allows the RNA primer to be made and, in turn, the
polymerase to begin replicating the DNA. This is an important reaction
sequence since DNA polymerases cannot initiate DNA synthesis de novo.
The mobile complex between helicase and primase has been called a
primosome. As the synthesis of an Okazaki fragment is completed and
the polymerase is released, a new primer has been synthesized. The same
polymerase molecule remains associated with the replication fork and
proceeds to synthesize the next Okazaki fragment.

(Q.19) Respiratory acidosis is characterized by primary

(a) deficit of carbonic acid

(b) excess of carbonic acid

(c) deficit of bicarbonate

(d) excess of bicarbonate


Your Response :

Correct Answer : b

Exp: B. Excess of carbonic acid

Ordinarily, chemical and physiological buffer systems maintain the


hydrogen ion concentration of body fluids within very narrow pH ranges.
Abnormal conditions may disturb the acid-base balance. For example, the
pH of arterial blood is normally 7.35-7.45. A value below 7.35 produces
acidosis. A pH above 7.45 produces alkalosis. Such shifts in the pH of
body fluids may be life threatening. In fact, a person usually cannot
survive if the pH drops to 6.8 or rises to 8.0 for more than a few hours.

Acidosis results from an accumulation of acids or a loss of bases, both of


which cause abnormal increases in the hydrogen ion concentrations of
body fluids. Conversely, alkalosis results from a loss of acids or an
accumulation of bases accompanied by a decrease in hydrogen ion
concentrations.

The two major types of acidosis are respiratory acidosis and metabolic
acidosis. Factors that increase carbon dioxide, also increasing the
concentration of carbonic acid (the respiratory acid), cause
respiratory acidosis. Metabolic acidosis is due to an abnormal
accumulation of any other acids in the body fluids or to a loss of bases,
including bicarbonate ions.

Similarly, the two major types of alkalosis are respiratory alkalosis and
metabolic alkalosis. Excessive loss of carbon dioxide and consequent loss
of carbonic acid cause respiratory alkalosis. Metabolic alkalosis is due to
excess loss of hydrogen ions or gain of bases.

Since in respiratory acidosis carbon dioxide accumulates, it can result


from factors that hinder pulmonary ventilation. These include the
following: Injury to the respiratory center of the brain stem, decreasing
rate and depth of breathing. Obstructions in air passages that interfere
with air movement into the alveoli. Diseases that decrease gas exchanges,
such as pneumonia, or those that reduce surface area of the respiratory
membrane, such as emphysema. Any of these conditions can increase the
level of carbonic acid and hydrogen ions in body fluids, lowering pH.
Chemical buffers, such as hemoglobin, may resist this shift in pH. At the
same time, increasing concentrations of carbon dioxide and hydrogen ions
stimulate the respiratory center, increasing breathing rate and depth and
thereby lowering carbon dioxide concentration. Also, the kidneys may
begin to excrete more hydrogen ions. Eventually, thanks to these chemical
and physiological buffers, the pH of the body fluids may return to normal.
When this happens, the acidosis is said to be compensated. The symptoms
of respiratory acidosis result from depression of central nervous system
function, and include drowsiness, disorientation, and stupor. Evidence of
respiratory insufficiency, such as labored breathing and cyanosis, is
usually also evident. In uncompensated acidosis, the person may become
comatose and die.

(Q.20) Which of the following enzymes produce a product used for synthesis of ATP by
substrate level phosphorylation ?

(a) Phosphofructokinase

(b) Aldolase

(c) Phosphoglycerate mutase

(d) Enolase

Your Response :

Correct Answer : d

Exp: D. Enolase

Enolase, also known as phosphopyruvate hydratase, is


ametalloenzymeresponsible for the catalysis of the conversion of2-
phosphoglycerate(2-PG) tophosphoenolpyruvate(PEP), the ninth and
penultimate step ofglycolysis. Enolase belongs to the class Lyase.
Enolase can also catalyze the reverse reaction, depending on
environmental concentrations of substrates. The optimum pH for this
enzyme is 6.5. Enolase is present in all tissues and organisms capable of
glycolysis or fermentation. The enzyme was discovered by Lohmann
andMeyerhofin 1934, and has since been isolated from a variety of
sources including human muscle anderythrocytes. In humans, deficiency
of ENO1 is linked to hereditaryhaemolytic anemiawhile ENO3 deficiency
is linked toglycogen storage disease XIII.

Thus the product of step catalysed by Enolase is PEP, is used in


substrate level phosphorylation.

Substrate-level phosphorylationis a type of metabolic reaction that


results in the formation ofadenosine triphosphate(ATP) orguanosine
triphosphate(GTP) by the direct transfer and donation of
aphosphoryl(PO3) group toadenosine diphosphate(ADP) orguanosine
diphosphate(GDP) from a phosphorylatedreactive intermediate. Note that
the phosphate group does not have to come directly from the substrate. By
convention, the phosphoryl group that is transferred is referred to as a
phosphate group.

An alternative way to create ATP is throughoxidative phosphorylation,


which takes place during the process ofcellular respiration, in addition to
the substrate-level phosphorylation that occurs during glycolysis and the
Krebs cycle. During oxidative phosphorylation, NADH is oxidized to
NAD+, yielding 2.5 ATPs, and FADH2(flavin adenine dinucleotide) yields
1.5 ATPs when it is oxidized. Oxidative phosphorylation uses an
electrochemical or chemiosmoticchemiosmosisgradientof protons (H+)
across the inner mitochondrial membrane to generate ATP from ADP and
a molecule of inorganic phosphate, which is a key difference from
substrate-level phosphorylation.

Unlikeoxidative phosphorylation,oxidationandphosphorylationare not


coupled in the process of substrate-level phosphorylation, although both
types of phosphorylation result in the formation of ATP, and reactive
intermediates are most often gained in course ofoxidationprocesses
incatabolism. However, usually most of the ATP is generated by oxidative
phosphorylation in aerobic or anaerobic respiration. Substrate-level
phosphorylation serves as fast source of ATP independent of external
electron acceptors and respiration. This is the case for example in
humanerythrocytes, which have no mitochondria, and in the muscle
during oxygen depression.

The main part of substrate-level phosphorylation occurs in the cytoplasm


of cells as part ofglycolysisand in mitochondria as part of theKrebs
Cycleunder bothaerobicandanaerobicconditions. In thepay-off phase of
glycolysis, two ATP are produced by substrate-level phosphorylation: two
and only two1,3-bisphosphoglycerateare converted to3-
phosphoglyceratebytransferringa phosphate group to ADP by a kinase;
twophosphoenolpyruvateare converted topyruvateby the transfer of
their phosphate groups to ADP by another kinase. The first reaction
occurs after the generation of 1,3-bisphosphoglycerate from 3-
phosphoglyceraldehyde and an organic phosphate via adehydrogenase.
ATP is generated in a following separate step (key difference from
oxidative phosphorylation) by transfer of the high-energy phosphate on
1,3-bisphosphoglycerate to ADP via the enzymephosphoglycerate kinase,
generating 3-phosphoglycerate. As ATP is formed of a former inorganic
phosphate group, this step leads to the energy yield of glycolysis. The
second substrate-level phosphorylation occurs later by means of the
reaction of phosphenolpyruvate (PEP) to pyruvate via thepyruvate kinase.
This reaction regenerates the ATP that has been used in the preparatory
phase of glycolysis to activate glucose to glucose-6-phosphate and
fructose-6-phosphate to fructose-1,6-bisphosphate, respectively.

(Q.21) The lipoprotein with the fastest electrophoretic mobility and the lowest TG content is

(a) VLDL

(b) HDL

(c) LDL
(d) Chylomicrons

Your Response :

Correct Answer : b

Exp: B. HDL

Plasma lipoproteins can be separated by analytical capillary isotachophoresis


(ITP) according to their electrophoretic mobility in a defined buffer system.
As in lipoprotein electrophoresis, HDL show the highest mobility followed
by VLDL, IDL, and LDL. Chylomicrons migrate according to their net-
charge between HDL and VLDL, because ITP has negligible molecular sieve
effects. Three HDL subfractions were obtained which were designated fast-,
intermediate-, and slow-migrating HDL.

Physical properties and lipid compositions of lipoprotein classes.

CM VLDL LDL HDL

Density (g/ml) < 0.94 0.94-1.006 1.006- 1.063-


1.063 1.210

Diameter () 6000-2000 600 250 70-120

Total lipid (wt%) * 99 91 80 44

Triacylglycerols 85 55 10 6

Cholesterol esters 3 18 50 40

Cholesterol 2 7 11 7

Phospholipids 8 20 29 46

* Most of the remaining material comprises the various apoproteins.

Major Lipoprotein Classes

Apolipoproteins

Lipoprotein Density, Size, Electrophoretic Major Other Other


g/mLa nmb Mobilityc Constituents

Chylomicrons 0.930 75 Origin ApoB- A-I, Retinyl


1200 48 A-IV, esters
C-I, C-
II, C-
III, E
Chylomicron 0.930 30 Slow pre-beta ApoB- A-I, Retinyl
remnants 1.006 80 48 A-IV, esters
C-I, C-
II, C-
III, E

VLDL 0.930 30 Pre-beta ApoB- A-I, Vitamin E


1.006 80 100 A-II,
A-V,
C-I, C-
II, C-
III, E

IDL 1.006 25 Slow pre-beta ApoB- C-I, C- Vitamin E


1.019 35 100 II, C-
III, E

LDL 1.019 18 beta ApoB- Vitamin E


1.063 25 100

HDL 1.063 512 alpha ApoA- A-II, LCAT,


1.210 I A-IV, CETP
A-V, paroxonase
C-III,
E

Lp(a) 1.050 25 Pre-beta ApoB- Apo(a)


1.120 100

(Q.22) Which of the following amino acids has a positively charged side chain?

(a) Arginine

(b) Aspartate

(c) Asparagine

(d) Glutamate

Your Response :

Correct Answer : a

Exp: A. Arginine

Both arginine and lysine are positively charged. Arginine has a bidentate,
resonant guanido group, whereas lysine has a second primary amino
group. Aspartate and glutamate are both negatively charged through their
carboxyl groups. Asparagine and glutamine are the uncharged derivatives
of aspartate and glutamate, respectively.

To Summarize :

Of 20 Amino acids :

AA Postively AA Negatively AA Uncharged Side


charged Side chains : charged Side chains : 2 chains : 15
3

Lysine Aspartic acid Rest all.

Arginine Glutamic acid

Histidine

(Q.23) Which of the following may be presenting feature of phenylketonuria?

(a) Salaam spasms

(b) Mental retardation

(c) Eczema

(d) All of above

Your Response :

Correct Answer : d

Exp: D. All of above (Ref: Harrison-medicine, 16th ed., 2305)

Phenyl ketonurea presents as Seizures , albinism and musty odor to the


babies sweat and urine

They normally never present with diarrhea or hematemesis

Huntington chorea is seen in Huntington disease and this present in


adult life.

Salaam spasm is type of epileptic seizure in which flexor spasm with


rapid bending of head and torso forward and simultaneous raising and
bending of arms which mimics as if child is doing salaam .it is seen in

The hyperphenylalaninemias result from impaired conversion of


phenylalanine to tyrosine. The most common and clinically important is
phenylketonuria (frequency 1:10,000), which is an autosomal recessive
disorder characterized by an increased concentration of phenylalanine and
its by-products in body fluids and by severe mental retardation if
untreated in infancy. It results from reduced activity of phenylalanine
hydroxylase (PAH deficiency phenylketonuria). The accumulation of
phenylalanine inhibits the transport of other amino acids required for
protein or neurotransmitter synthesis, reduces synthesis and increases
degradation of myelin, and leads to inadequate formation of
norepinephrine and serotonin. Phenylalanine is a competitive inhibitor of
tyrosinase, a key enzyme in the pathway of melanin synthesis, and
accounts for the hypopigmentation of hair and skin. Untreated children
with classic phenylketonuria are normal at birth but fail to attain early
developmental milestones, develop microcephaly, and demonstrate
progressive impairment of cerebral function. Hyperactivity, seizures,
and severe mental retardation are major clinical problems later in
life. Electroencephalographic abnormalities; "mousy" odor of skin,
hair, and urine (due to phenylacetate accumulation); and a tendency
to hypopigmentation and eczema complete the devastating clinical
picture. In contrast, affected children who are detected and treated at birth
show none of these abnormalities.

Treatment: Phenylketonuria

To prevent mental retardation, diagnosis and initiation of dietary


treatment of classic phenylketonuria must occur before the child is 3
weeks of age. For this reason, most newborns in North America,
Australia, and Europe are screened by determinations of blood
phenylalanine levels. Abnormal values are confirmed using quantitative
analysis of plasma amino acids. Dietary phenylalanine restriction is
usually instituted if blood phenylalanine levels are >300 micromol/L (5
mg/dL). Treatment consists of a special diet low in phenylalanine and
supplemented with tyrosine, since tyrosine becomes an essential amino
acid in phenylalanine hydroxylase deficiency. With therapy, plasma
phenylalanine concentrations should be maintained between 60 and 360
mol/L (1 and 6 mg/dL). Dietary restriction should be continued and
monitored indefinitely. About one-third of all patients with
phenylketonuria and the majority of those with milder forms
(phenylalanine <1200 micromol/L at presentation) show increased
tolerance to dietary proteins and improved metabolic control when treated
with tetrahydrobiopterin (520 mg/kg per day), an essential cofactor of
phenylalanine hydroxylase. This drug should be used in addition to
dietary therapy.

Women with phenylketonuria, who have been treated since infancy, reach
adulthood and can become pregnant. If maternal phenylalanine levels are
not strictly controlled before and during pregnancy, their offspring are at
increased risk for congenital defects and microcephaly (maternal
phenylketonuria). After birth, these children have severe mental and
growth retardation. Pregnancy risks can be minimized by continuing
lifelong phenylalanine-restricted diets and assuring strict phenylalanine
restriction 2 months prior to conception and throughout gestation.
(Q.24) Enzyme involved in binding of & for conjugated bilirubin formation is:

(a) Kinases

(b) Ligases

(c) Syntheses

(d) Transferases

Your Response :

Correct Answer : d

Exp: : D. Transferases (Harpers Biochemistry - 27th edition - 368)

Bilirubin is nonpolar and would persist in cells (eg, bound to lipids) if not
rendered water-soluble. Hepatocytes convert bilirubin to a polar form,
which is readily excreted in the bile, by adding glucuronic acid molecules
to it. This process is called conjugation and can employ polar molecules
other than glucuronic acid (eg, sulfate). Many steroid hormones and drugs
are also converted to water-soluble derivatives by conjugation in
preparation for excretion . The conjugation of bilirubin is catalyzed by a
specific glucuronosyltransferase. The enzyme is mainly located in the
endoplasmic reticulum, uses UDP-glucuronic acid as the glucuronosyl
donor, and is referred to as bilirubin-UGT. Bilirubin monoglucuronide is
an intermediate and is subsequently converted to the diglucuronide . Most
of the bilirubin excreted in the bile of mammals is in the form of bilirubin
diglucuronide. However, when bilirubin conjugates exist abnormally in
human plasma (eg, in obstructive jaundice), they are predominantly
monoglucuronides. Bilirubin-UGT activity can be induced by a number
of clinically useful drugs, including phenobarbital. More information
about glucuronosylation is presented below in the discussion of inherited
disorders of bilirubin conjugation. Secretion of conjugated bilirubin into
the bile occurs by an active transport mechanism, which is probably rate-
limiting for the entire process of hepatic bilirubin metabolism. The protein
involved is MRP-2 (multidrug-resistance-like protein 2), also called
multispecific organic anion transporter (MOAT). It is located in the
plasma membrane of the bile canalicular membrane and handles a number
of organic anions. It is a member of the family of ATP-binding cassette
(ABC) transporters. The hepatic transport of conjugated bilirubin into the
bile is inducible by those same drugs that are capable of inducing the
conjugation of bilirubin. Thus, the conjugation and excretion systems for
bilirubin behave as a coordinated functional unit.

(Q.25) Phospholipid associated with mechanism of hormone action is

(a) Phosphatidyl choline


(b) Phosphatidyl lecithin

(c) Plasmalogen

(d) Phosphatidyl inositol

Your Response :

Correct Answer : d

Exp: : D) Phosphotidyl inositol (Harpers Biochemistry - 27th edition -


165)

Phosphatidylcholines (Lecithins) Occur in Cell Membranes

Phosphoacylglycerols containing choline are the most abundant


phospholipids of the cell membrane and represent a large proportion of
the body's store of choline. Choline is important in nervous transmission,
as acetylcholine, and as a store of labile methyl groups. Dipalmitoyl
lecithin is a very effective surface-active agent and a major constituent of
the surfactant preventing adherence, due to surface tension, of the inner
surfaces of the lungs. Its absence from the lungs of premature infants
causes respiratory distress syndrome. Most phospholipids have a
saturated acyl radical in the sn-1 position but an unsaturated radical in the
sn-2 position of glycerol. Phosphatidylethanolamine (cephalin) and
phosphatidylserine (found in most tissues) are also found in cell
membranes and differ from phosphatidylcholine only in that ethanolamine
or serine, respectively, replaces choline. Phosphatidylserine also plays a
role in apoptosis (programmed cell death).

Phosphatidylinositol Is a Precursor of Second Messengers

The inositol is present in phosphatidylinositol as the stereoisomer,


myoinositol. Phosphatidylinositol 4,5-bisphosphate is an important
constituent of cell membrane phospholipids; upon stimulation by a
suitable hormone agonist, it is cleaved into diacylglycerol and inositol
trisphosphate, both of which act as internal signals or second
messengers.

Cardiolipin Is a Major Lipid of Mitochondrial Membranes

Phosphatidic acid is a precursor of phosphatidylglycerol which, in turn,


gives rise to cardiolipin. This phospholipid is found only in mitochondria
and is essential for mitochondrial function. Decreased cardiolipin levels or
alterations in its structure or metabolism cause mitochondrial dysfunction
in aging and in pathological conditions including heart failure,
hypothyroidism and Barth syndrome (cardioskeletal myopathy).

Lysophospholipids Are Intermediates in the Metabolism of


Phosphoglycerols

These are phosphoacylglycerols containing only one acyl radical, eg,


lysophosphatidylcholine (lysolecithin) (Figure 159), important in the
metabolism and interconversion of phospholipids. It is also found in
oxidized lipoproteins and has been implicated in some of their effects in
promoting atherosclerosis.

Plasmalogens Occur in Brain & Muscle

These compounds constitute as much as 10% of the phospholipids of


brain and muscle. Structurally, the plasmalogens resemble
phosphatidylethanolamine but possess an ether link on the sn-1 carbon
instead of the ester link found in acylglycerols. Typically, the alkyl radical
is an unsaturated alcohol. In some instances, choline, serine, or inositol
may be substituted for ethanolamine.

(Q.26) Priming Mixture in percussion cap of a shotgun cartridge contains

(a) Potassium Chlorate

(b) Potassium Nitrate

(c) Potassium Carbonate

(d) Potassium Oxalate

Your Response :

Correct Answer : a

Exp: Ans A) Potassium Chlorate

The primer mixtures consist of four basic chemical components: the


initiating explosive, oxidizing agent, fuel, and sensitizer. Each component
can contribute more or less ingredients to the GSR after a gun has been
fired. Primer mixtures are classified as mercuric, chlorate, non-corrosive,
and lead-free primers, depending on their chemical composition.
Potassium chlorate was added to mercury fulminate priming mixtures so
incandescent potassium chloride would have a similar effect in small arms
cartridges.The most popular primer composition at present is the non-
corrosive primer.

The initiator in non-corrosive primers is lead styphnate although formerly


lead azide, potassium chlorate and mercury fulminate were used to serve
as initiator in the primers. Barium nitrate is the most commonly used
oxidant in small-arms ammunition, but barium peroxide, lead nitrate, or
lead peroxide may also be used. Antimony sulfide is commonly used as
fuel in primers, but calcium silicide, lead thiocyanate, powdered
aluminum, and powdered zirconium, magnesium, and titanium have also
been employed

(Q.27) Mc Naughten Rule has also been accepted in India as the Law of Criminal
Responsibility and is included in Sec.________

(a) 84 IPC

(b) 85 IPC

(c) 89 IPC

(d) 91 IPC

Your Response :

Correct Answer : a

Exp: Ans A) 84 IPC

Mc Naughten rule is provided in sec 84 of IPC . it states that

the jurors ought to be told in all cases that every man is to be presumed
to be sane, and to possess a sufficient degree of reason to be responsible
for his crimes, until the contrary be proved to their satisfaction; and that to
establish a defence on the ground of insanity, it must be clearly proved
that, at the time of the committing of the act, the party accused was
labouring under such a defect of reason, from disease of the mind, as not
to know the nature and quality of the act he was doing; or, if he did know
it, that he did not know he was doing what was wrong

(Q.28) Following are the postmortem diagnostic signs of strangulation EXCEPT

(a) Ligature mark completely encircling neck

(b) Ecchymosis about the edges of ligature mark

(c) Dribbling saliva from angle of mouth

(d) Fracture of thyroid cartilage

Your Response :

Correct Answer : c

Exp: Ans C) Dribbling saliva from angle of mouth

STRANGULATION

Death by strangulation is usually homicidal but less commonly may be


accidental or suicidal. Strangulation is due to constriction of the neck by a
force other than the weight of the body. One may use a ligature, but the
hands may also be used (manual strangulation or throttling).

The ligature mark is usually at a lower level and more horizontal than in
hanging and is usually located in the mid-larynx region i.e. at about the
level of the thyroid cartilage. The mark is usually less prominent than in
hanging, especially if a soft ligature is used. However, if a narrow wire is
used, the mark may be deep and the skin may even be cut.

Postmortem Findings

The face is markedly swollen and cyanosed with bulging eyes and a
protruding tongue. Petechial haemorrhages are often seen. The skin shows
a constriction mark from the ligature or bruises from the fingers (in cases
of throttling). There is haemorrhage and bruising of the neck muscles.
Fractures of the larynx may occurespecially in cases of manual
strangulationparticularly of the superior cornu of the thyroid cartilage
and/or the greater cornu of the hyoid bone. The lungs are oedematous and
congested.

It is obvious that throttling can only be homicidal in nature. Attempted


self-throttling would be self-defeating because the hands would loosen
their grip when unconsciousness supervenes. Suicidal strangulation is
possible but uncommon. It may be achieved by using a tourniquet
mechanism or adhesive ligatures, e.g. Velcro-type bands Accidental
strangulation does occur e.g. from clothing such as a necktie or scarf
becoming caught in moving machinery.

HANGING

The neck is constricted by a ligature and the constriction force is provided


by the victim's body weight.

Hanging is usually suicidal, but less commonly, may be accidental. Cases


of homicidal hanging are unusual.

Note that suspension of the body in fatal hanging does not have to be
complete one may die from hanging with only a part of the body being
suspended. Hanging can occur in the sitting or slumped position, where
the suspension point is a door knob or something at a similarly low level.

Causes of death in hanging

Suicidal (Non-judicial)

1) Asphyxia - due to compression of the larynx and/or blocking of the air


passages by the root of the tongue pressing against the pharynx.

2) Cerebral hypoxia/anaemia - due to carotid artery compression


3) Vagal inhibition

4) A combination of any of the above

Postmortem Findings

These are similar to those seen in strangulation except that fractures of the
laryngeal cartilages are not as common.

The ligature mark on the neck tends to be at a higher level than in


strangulation and rises to a suspension point (inverted V shape) behind or
under one ear, or at the back of the head.

As in strangulation, the ligature mark may be absent if the ligature


material is soft (eg a bed sheet), or where the deceased was cut down
shortly after hanging him/herself, or where the body is decomposed.

(Q.29) Most specific test to detect blood stains is:

(a) Benzidine test

(b) Teichmanns test

(c) Orthotoluidine test

(d) Spectroscopic test

Your Response :

Correct Answer : d

Exp: D. Spectroscopic test

Spectroscopic test is the most specific for blood stains. Tests for blood
stains-

(1) Benzidine test-

Saturated benzidine in glacial acetic acid + 10v H2O2), intense dark blue
colour, blood of any age, blood exposed to heat/cold, treated with
detergents, most sensitive- sensitivity 1 in 3 lakhs, more Negative value,
Weakly +ve by pus, saliva, milk, rust, formalin, bacteria, oxidizing
agents, vegetables.

(2) Hemin crystal test=Teichmanns test- NaCl+glacial acetic


acidheat, yellow-red to brownish-black rhombic crystals of
hemin/hematin chloride, arranged singly or in clusters.

(3) Hemochromogen crystal test=Takayama test-


NaOH+pyridine+glu+H2O, pink feathery crystals of hemochromogen or
reduced alkaline hematin in clusters.
(4) Spectroscopy test- most specific, most reliable, most delicate.

SOME IMPORTANT TESTS IN FMT:

Gurd Test Microscopy of blood, sputum


and urine for fat globules In Fat
Embolism

Hoppe-Seylers Test CO Poisoning

Kunkels Test(Tannic acid Test)

Kozelka and Hine test Determine alcohol levels in blood


and urine
Cavett Test

Alcohol dehydrogenase Test

Gas chromatography

Breath analysis based on Henrys law

Foderes Test The Static test for live birth

Galli-mainani Test Biological tests of pregnancy

Ascheim-Zondek Test

Freidman Test

Hogben Test

Leuko malachite green Test

Orthotoluidine Test

Guaicum Test

Kastle-Meyer Test/Phenolphthalein Detection of blood in stains


Test

Benzidine Test

Takayama hemochromogen crystal


test

Teichmanns Test/hemin crystal test

Precipitin Test

Hemagglutination inhibition Test

Latex Test
Stomach Bowel Test Establishment of respiration in a
fetus
Brslaus second life Test

Wredins Test Detection of live birth

Marsh test Arsenic poisoning

Reinsch Test

Barberio Test Seminal stains

Florence Test

Acid phosphatase Test

Creatinine phosphatase V

ELISA

Hypersensitive Instrumental analysis Detection of Gun Shot Primer


Residues(GSR)

Magnus Test Tests of circulation

Icard Test

Diaphanous Test

(Q.30) All of thefollowing are types of chemical antidotes except

(a) Activated charcoal

(b) Common salt

(c) Tannic acid

(d) Potassium permanganate

Your Response :

Correct Answer : a

Exp: A. Activated charcoal

TYPES OF EXAMPLES
ANTIDOTES

Mechanical Activated charcoal, Demulcents, Bulky food

Chemical Common salt (Silver nitrate), Albumen


(Silver chloride). Dialysed iron (Arsenic)

Copper sulphate (Phosphurus), Pottasium


permanganate, Tannic acid, Alkalis,

Acids

Physiological / Physostigmine and Atropine


Pharmacological
Cyanides and amyl nitrate

Barbiturates and picrotoxin or amphetamine

Strychnine and barbiturates

(Q.31) Characteristic of antemortem woundsis ?

(a) Gaping edges

(b) Deep staining of edges & tissues, not removed by washing

(c) Vital reaction seen

(d) All of above

Your Response :

Correct Answer : d

Exp: D. All of above

Ante-mortem wounds Post-mortem wounds

Haemorrhage- copious, arterial. Haemorrhage- slight, venous.

Spurting of blood. No spurting of blood.

Firmly clotted blood. Not clotted blood; Currant-jelly or


chicken-fat clot.

Deep staining of edges & tissues, not Edges & tissues not stained;
removed by washing. removed by washing.

Edges gape due to normal elasticity, Edges do not gape but closely
are swollen, everted. approximated.

Vital reaction/ Inflammation & No vital reaction/ No inflam


repair seen.
mation or repair.
Esterases, ATP, aminopeptidase, No enzyme reaction.
acid PO4ase, alkaline PO4ase
enzyme reaction seen.

(Q.32) Which type of viral hepatitis is associated with high mortality in pregnantwomen ?

(a) Hepatitis B virus

(b) Hepatitis E virus

(c) Hepatitis D virus

(d) Hepatitis C virus

Your Response :

Correct Answer : b

Exp: B) Hepatitis E virus

Hepatitis E virus (HEV) is a single-stranded RNA virus that causes large-


scale epidemics of acute viral hepatitis, particularly in developing
countries. In men and non pregnant women, the disease is usually self-
limited and has a case-fatality rate of less than <0.1%. However in
pregnant women particularly from certain geographic areas in India,
HEV infection is more severe, often leading to fulminant hepatic
failure and death in a significant proportion of patients. In contrast,
reports from Egypt, Europe and the USA have shown that the course and
severity of viral hepatitis during pregnancy is not different from that in
non-pregnant women. The reasons for this geographical difference are not
clear. The high mortality rate in pregnancy has been thought to be
secondary to the associated hormonal (estrogen and progesterone)
changes during pregnancy and consequent immunological changes. These
immunological changes include down regulation of p65 component of NF
B with a predominant Th2 bias in the T-cell response along with host
susceptibility factors, mediated by HLA expression.

(Q.33) The major compatibility test before blood transfusion consists of cross matching of

(a) Donors red cells andRecipients serum

(b) Donors serum andRecipients red cells

(c) Donors serum andRecipients serum

(d) Donors red cells andRecipients red cells

Your Response :
Correct Answer : a

Exp: A) Donors red cells and Recepients serum

Major and Minor cross-match tests

Major cross-match test, consisting of mixing the patients serum with


donor RBCs.

Minor cross-match test, consisting of mixing the donors plasma with


patients RBCs

The minor cross-match test has been completely eliminated in most blood
banks, because donor samples are screened beforehand for the more
common Abs.

(Q.34) The type of vegetative endocarditis associated with mucinous adenocarcinoma is

(a) Infective endocarditis

(b) Non-bacterial thrombotic endocarditis

(c) Both a and b

(d) Rheumatic endocarditis

Your Response :

Correct Answer : b

Exp: B) Non-bacterial thrombotic endocarditis

Non-bacterial thrombotic endocarditis (NBTE) is the deposition of small


sterile vegetations on valve leaflets. Formerly known as marantic
endocarditis.

Marantic vegetations are often associated with previous rheumatic fever.

Other risk factors include:

hypercoagulable states

malignant cancers, especially mucin-producing adenocarcinomas (most


commonly associated with pancreatic adenocarcinomas)

systemic lupus erythematosus

trauma (e.g., catheters)

(Q.35) The absence of ganglion cells within the affected segment of bowel is a feature of
(a) Tropical sprue

(b) Hirschsprungs disease

(c) Celiac disease

(d) Crohns disease

Your Response :

Correct Answer : b

Exp: B) Hirschsprungs disease

Hirschsprung's disease (HD) is a disorder of the abdomen that occurs


when part or all of the large intestine or antecedent parts of the
gastrointestinal tract have no ganglion cells and therefore cannot function.
During normal fetal development, cells from the neural crest migrate into
the large intestine (colon) to form the networks of nerves called
Auerbach's plexus and Meissner's plexus. In Hirschsprung's disease, the
migration is not complete and part of the colon lacks these nerve bodies
that regulate the activity of the colon.

(Q.36) Cystic Hygroma is found in

(a) Klinefelters syndrome

(b) Downs syndrome

(c) Edwards syndrome

(d) Turners syndrome

Your Response :

Correct Answer : d

Exp: D) Turners syndrome

The following is a list of common symptoms of Turner syndrome. It is


important to note that an individual may have any combination of
symptoms and is unlikely to have all symptoms.

Short stature

Lymphedema (swelling) of the hands and feet

Broad chest (shield chest) and widely spaced nipples

Low hairline
Low-set ears

Reproductive sterility

Rudimentary ovaries gonadal streak (underdeveloped gonadal structures


that later become fibrosed)

Amenorrhoea, or the absence of a menstrual period

Increased weight, obesity

Shield shaped thorax of heart

Shortened metacarpal IV

Small fingernails

Characteristic facial features

Webbed neck from cystic hygroma in infancy

Aortic valve stenosis

Coarctation of the aorta

Bicuspid aortic valve

Horseshoe kidney

Visual impairments sclera, cornea, glaucoma, etc.

Ear infections and hearing loss

High waist-to-hip ratio (the hips are not much bigger than the waist)

Attention Deficit/Hyperactivity Disorder or ADHD (problems with


concentration, memory, attention with hyperactivity seen mostly in
childhood and adolescence)

Nonverbal Learning Disability (problems with math, social skills and


spatial relations)

(Q.37) Munro microabscesses occur in

(a) Psoriasis

(b) Lichen planus

(c) Lupus vulgaris

(d) Impetigo
Your Response :

Correct Answer : a

Exp: A) Psoriasis

Munro's microabscess is an abscess (collection of neutrophils) in the


stratum corneum of the epidermis due to the infiltration of neutrophils
from papillary dermis into the epidermal stratum corneum. They are
associated with psoriasis where they are seen in the hyperkeratotic and
parakeratotic areas of the stratum corneum and with seborrheic dermatitis.

There are various subtypes of psoriasis, a common dermatosis with well


demarcated erythematous scaly plaques.

Due to the dynamic nature of psoriasis, this disease frequently does not
show all classical histological features, but the typical changes of chronic
plaque psoriasis are:

Hyperkeratosis: mainly composed of parakeratosis, some orthokeratosis

Neutrophils in stratum corneum (Munro's microabscesses) and squamous


cell layer (spongiform pustules of Kogoj)

Hypogranulosis

Epidermis is thin over dermal papillae (thinned suprapapillary plates)

Regular acanthosis, often with clubbed rete ridges

Relatively little spongiosis

Dilated capillaries in dermal papillae

Perivascular lymphohistiocytic infiltrate

(Q.38) All of the following are related to arachidonic acid metabolism except

(a) Prostacyclin

(b) Linoleic acid

(c) Thromboxane

(d) Cationic proteins

Your Response :

Correct Answer : d

Exp: (D) Cationic proteins


Inflammatory mediators from arachidonic are:

TxA2, PGD2, PGE2, PGF2 alpha, PGI2 (Cycloxygenase pathway)


leukotrines (Lipoxygenase pathway)

LTB4 is a neutrophil chemotactic agent.

LTC4, D4 and E4 (SRS-A) function in bronchoconstriction,


vasoconstriction, contractions of smooth muscle and increased vascular
permeability. Monteleucast and Zafirleucast are Leukotriene inhibitors
used in Asthma prophylaxis.

PGE2Bronchoconstriction, platelet aggregation.

PGE 2Potent inducer of bone resorption and releases Ca++ from bones
and cervical effacement.

PGF2bronchoconstriction, role in uterine and ovarian function (fundal


contraction).

PG12bronchodilation, vasoconstriction, and platelet aggregation.

TXA2bronchodilation, vasoconstriction, and platelet aggregation


stimulation (Bradykinin and histamine are potent vasodilators)

Action Eicosanoid

Vasodilation PGI2 (prostacyclin), PGE1, PGE2,


PGD2

Vasoconstriction Thromboxane A2, leukotrienes C4,


D4, E4

Increased vascular Leukotrienes C4, D4, E4


permeability

Chemotaxis, leukocyte Leukotriene B4


adhesion

(Q.39) Ocular basement membrane is stained by:

(a) Alcaine blue

(b) Periodic acid-Schiff

(c) Methylene blue

(d) Giemsa stain

Your Response :
Correct Answer : b

Exp: (B) Periodic acid-Schiff

Not only ocular almost all basement membrans are stained with PAS
(Periodic acid-Schiff) Because it has mucoploysaccharides

Periodic acid-Schiff reactive (PAS) method is principally used to


demonstrate structures rich in polysaccharides (glycogen),
mucopolysaccharides (e.g., ground substance of connective tissues,
basement membrane, and mucus), glycoproteins (thyroglobulin), and
glycolipids.

Alcian blue is a copper-containing dye for staining acid


mucopolysaccharides; it may be combined with periodic acid-Schiff
reaction.

Methylene blue stains basophilic constituents of the tissue, whereas


erythrocin stains the acidophilic elements.

Giemsa stain is used for differential staining of blood smears, spleen, and
bone marrow cells, for staining protozoan parasites such as Plasmodium
spp. and Trypanosoma spp., for Chlamydia spp. as well as for the
identification of viral inclusion bodies.

Reverse Giemsa staining is a method in which the reciprocal (R-bands) of


the banding pattern seen in the Giemsa method for chromosomes is
obtained.

This question deserves some addition as this STAINING based


questions are now being asked very frequently by the examiners.the
following list has been compiled by me from several references
throughout my preparation period.

SPECIFIC USE
STAIN/STAINING
METHOD

Movat stain Aortic wall in aortic dissection

Warthin Starry silver stain Bacillary angiomatosis {bartonella }

Von kossa stain Calcium

Rhodanine stain Wilsons disease for demonstrating Cu

Orcein stain Wilsons disease for demonstrating Cu-


associated protein

Reticulin stain Differentiation of Pituitary adenoma


{+ve} from hyperplasia{-ve}

Brilliant cresyl blue Reticulocytes


{Supravital stain}

Coomassie blue stain SDS-PAGE analysis of RBC membrane


{for protein demonstration}

PAS SDS-PAGE analysis of RBC membrane


{for Glycophorin demonstration}

SYBR Green dye PCR

Filipin stain Cholesterol

Schultz stain Cholesterol esters

Calcium lipase stain Mono-,di-,and tri-glycerides

Nile blue sulphatase Psosphatidyl choline and phosphatidyl


serine

Plasmal reaction Phosphatidyl ethanolamine

Ferric hematoxylin Sphingomyelin

Fuelgen stain DNA

Methyl green pyronin

Gallocyanin chrome alum


method

Acridine orange

Annexin V Apoptotic cells

Triphenyl tetrazolium Myocardium


chloride
Viable magenta

Nonviable-colourless

Masson-Trichrome stain Mature collagen in tissues

PAS +ve ,Diastase resistant Alpha1 antitrypsin

Pas +ve Diastase sensitive Glycogen

Orcien/Aldehyde fuschin Ground glass hepatocytes in


stain Chr.hepatitis
Hopps Brown stain, H.pylori on gastric mucosa

Modified Giemse stain

Ethin srarry silver stain

Ethidium bromide stain

Toluidine blue Supravital staining of Laryngeal lesions


for diagnosis

(Q.40) The most common cytogenetic abnormality associated with multiple myeloma is

(a) t (14;4)

(b) t (14,11)

(c) del 13q

(d) t (14;16)

Your Response :

Correct Answer : c

Exp: (C).del 13q

Again this aspect of leukemias is a very important component of


questions and is very frequently asked in your exams. So once and for all,
like the previous tables just byheart this one too

CYTOGENETIC ABNORMALITIES :

Leukemia / Lymphoma Cytogenetic abnormalities

ALL Hyperdiploidy (> than 50 chromosomes)-


MOST COMMON OVERALL.

Polyploidy

t (12;21) MC in childhood and B cell


ALL

t (9;22) MC in adults

t (4;11)

Rearrangement of TALL gene

Trisomy 4,10,17
t (1;19)

Hypodiploidy.

CLL / SLL Del 13q MC

Del 11q

Del 17p

Trisomy 12q

Somatic hypermutations of Ig genes

AML t (8;21) M2

t (15;17) M3

Inv 16 M4

Inv 7, Inv 3

3q rearrangement

Presence of PTD of MLL or ITD of FLT3

Del / Monosomy of chromosome 5 or 7

MDR1 gene abnormality

Trisomy 8 and 11

Very rarely t (9;22 )- Philadelphia


chromosome

Myelodysplastic syndrome Monosomy 5

Monosomy 7

Deletion 5q or 7q

Trisomy 8

Deletion 20q

CML Reciprocal t (9;22) BCR ABL 210


kDa fusion protein

Juvenile myelomonocytic Monosomy or deletion of chromosome 7q


leukemia

Multiple myeloma Del 13q

Translocations involving Ig heavy chain


locus on Chr 14q :

t (14;4), t (14;11), t (14,6), t (14;16)

Reed Sternberg cell in Aneuploidy


Hodgkins lymphoma
Gain of chromosome 2p Site of cREL
Proto-oncogene

In lymphocyte predominant HD :

Identical IgH generearrangement

Vh segments showing ongoing somatic


hypermutations

Expression of latent membrane protein 1


(EBV coded protein)

Lymphoplasmacytic Del 6q
lymphoma /
Immunocytoma

Follicular lymphoma t (14;18)

Rearrangement of BCL6 gene on Chr 3

Diffuse large B cell Dysregulation of BCL6


lymphoma
BCL2 rearrangements

t (14;18)

Burkitt lymphoma Translocations involving c- MYC gene on


Chr 8 t (8;14), t(2,8), t (8,22)

Mantle cell lymphoma t (11;14)

(Q.41) Early granulation tissue in acute myocardial infarction is seen with

(a) 1 hr

(b) 24 hrs

(c) 1 week

(d) 1 month

Your Response :

Correct Answer : c
Exp: C. 1 week.

Myocardial Infarction (MI)

TIME GROSS LIGHT ELECTRON


FEATURES MICROSCOPY MICROSCOPY

REVERSIBLE
INJURY :

0 0.5 hrs None None Relaxation of


myofibrils,glycogen
loss,mitochondrial
swelling

IRREVERSIBLE
INJURY

1 4 hrs None Usually none, Sarcollemal


variable waviness disruption ;
of fibres at border mitochondrial
amorphous densities

4 12 hrs Occaisionally Beginning


dark mottling coagulation
necrosis;edema ;
hemorrhage

12 24 hrs Dark mottling Ongoing


coagulation
necrosis ;
pyknosis of nuclei
; myocyte
hypereosinophilia
; marginal
contraction band
necrosis ;
beginning
neutrophilic
infiltrate

1 3 days Mottling with Coagulation


yellow tan necrosis with loss
infarct centre of nuclei and
striations ;
interstitial
infiltration of
neutrophils
3 7 days Hyperemic Beginning
border ; disintegration of
central yellow dead myofibres ;
tan softening with dead
neutrophils ; early
phagocytosis of
dead cells by
macrophages at
infarct border

7 10 days Maximally Well developed


yellow tan and phagocytosis of
soft ; with dead cells; early
depressed red formation of
tan margins fibrovascular
granulation tissue
at margins

10 14 days Red grey Well established


depressed granulation tissue
infarct borders with new blood
vessels and
collagen
deposition

2 3 weeks Gray-white Increased collagen


scar ; deposition with
progressive decreased
from border cellularity
towards core
of infarct

2 months and later Scarring Dense


complete collagenous scar

(Q.42) Pathological change in CNS of AIDS patient:

(a) Perivascular giant cells

(b) Vasculitis

(c) Spongiform changes of white matter

(d) Microglial nodule

Your Response :

Correct Answer : d
Exp: D. Microglial nodule

Ref : Kumar: Robbins and Cotran Pathologic Basis of Disease,


Professional Edition, 8th ed., Chapter 23.

Human Immunodeficiency Virus

HIV can have direct effects on the nervous system as well as setting the
stage for opportunistic infections or tumors that can involve the nervous
system. As many as 60% of individuals with AIDS develop neurologic
dysfunction during the course of their illness; in some, it dominates the
clinical picture. Patterns of direct injury to the brain include:

Aseptic HIV-1 meningitisoccurring within 1 to 2 weeks of


seroconversion in about 10% of patients. This is associated with a
mild lymphocytic meningitis, perivascular inflammation, and some
myelin loss in the hemispheres.

HIV-1 meningoencephalitis (subacute encephalitis)causing AIDS-


dementia complex. This dementia begins insidiously with mental
slowing, memory loss, and mood disturbances, such as apathy and
depression. The brains of individuals with HIV-1 encephalitis show
chronic inflammatory reaction with widely distributed infiltrates of
microglial nodules containing macrophage-derived multinucleated
giant cells. It is thought that neuronal injury follows the secretion of
cytokines and chemokines from HIV-infected cells of the
macrophage lineage.

Vacuolar myelopathyinvolving the tracts of the spinal cord can


resemble subacute combined degeneration, although serum levels
of vitamin B12are normal. The pathogenesis of the lesion is
unknown; it does not appear to be caused directly by HIV-1, and
virus is not present within the lesions.

Primary HIV-Associated Neurologic Disorders

Central Nervous System

Primary HIV encephalopathies

Multinucleate giant-cell encephalitis (HIV


encephalitis)

HIV-associated white matter disease (HIV


leukoencephalopathy)

Neocortical/gray matter disease (HIV poliodystrophy)

Mixed patterns
Vacuolar myelopathy

Lymphocytic meningitis

Acute, monophasic meningitis

Chronic aseptic meningitis

Cerebral vasculitis

Peripheral Nervous System

Distal symmetric polyneuropathy

Inflammatory demyelinating neuropathies

Spinal and cranial radiculitis

Vasculitic neuropathy

Skeletal Muscle

Inflammatory myopathy (polymyositis)

Mitochondrial myopathy

Nemaline myopathy

(Q.43) Graft survival in the initial 48 hours depends on:

(a) Plasma imbibition

(b) Ingrowth of capillaries

(c) Development of lymphatics

(d) Saline dressing

Your Response :

Correct Answer : a

Exp: Ans A. Plasma imbibition

Ref : Schwartz Principles of Surgery 8th Ed, Chapter 44.

Skin Grafts

Skin grafts are mainstays for reconstruction of superficial defects or


adjuncts to more complex reconstructions. The type of skin graft is based
on the thickness of the graft taken. The term "graft" is used to denote the
fact that during harvest of the skin graft, all the vessels nourishing the
graft are cut. This is in contradistinction to the term "flap" which implies
that some aspect of the blood supply to the segment of tissue has
remained intact during transfer. As such, the graft itself is the donor
tissue; the wound bed to which it is applied is known as the recipient site.

Split-Thickness vs. Full-Thickness Skin Grafts

Split-Thickness Full-Thickness
Graft Graft

Reliable take +

Available donor sites +

Primary contracture +

Secondary contracture +

Mechanical durability +

Ability to grow hair, secrete +


sweat and sebum

Pigmentary changes +

A split-thickness graft requires that a portion of the dermis be taken along


with the epidermis. A full-thickness graft is one in which the full portion
of the dermis is taken with the superficial epidermis (Table 44-1). The
grafts will survive transfer based on a defined sequence of events that
culminates in vascular independence. These events are (1) serum
imbibitiondirect absorption of nutrients from recipient capillary beds
that generally takes place in the first 24 hours; (2) inosculationthe
connecting of donor and recipient vessels that typically begins in the 24-
to 72-hour period; and (3) angiogenesisvascular ingrowth of vessels
from the recipient bed into the graft that starts after 72 hours. Factors that
interrupt this processsuch as fluid collection under the graft or
mechanical shear forceswill compromise the graft take.

Thicker grafts have more difficulty with definitive adherence and survival
because of the greater demand on vascular ingrowth. However, the greater
the proportion of dermis the graft has, the greater the inhibition of the
myofibroblasts that cause secondary contraction. The full-thickness skin
graft can also retain functional hair follicles and sweat glands.Split-
thickness skin grafts are taken from sites depending on the thickness,
color, and quality of skin needed. The split-thickness grafts can be
meshed in varying ratios to expand the potential coverage area. The
drawbacks to meshed grafts include their suboptimal appearance and their
tendency to contract. They can be used in conjunction with skin
substitutes, in cases such as large-area burn reconstruction, when there
may be a relative paucity of donor sites. Full-thickness grafts are taken
from areas in which primary closure can be accomplished, such as the
groin or within redundant skin folds. 9 Skin grafts can be fixed in place
with compressive dressings to prevent problems with shear and fluid
collection. Generally, 5 days are required for definitive vascular ingrowth
to occur. Dressings can be left on the recipient site for this period of time.
Donor sites for split-thickness grafts are allowed to re-epithelialize under
occlusive (OpSite, Tegaderm) or moist, antibiotic impregnated gauze
(Xeroform). If allowed to heal uninterrupted, the donor site 12/1000-inch
skin graft can re-epithelialize in 7 to 14 days.

(Q.44) Chemotaxis in response to activation of cells results in:

(a) Random multidirectional movement

(b) Unidirectional motion

(c) Phagocytosis

(d) Augmented oxygen dependent bactericidal effect

Your Response :

Correct Answer : b

Exp: Ans : B Unidirectional motion

Role of Mediators in Different Reactions of Inflammation

Role in Inflammation Mediators

Vasodilation Prostaglandins

Nitric oxide

Histamine

Increased vascular Histamine and serotonin


permeability
C3a and C5a (by liberating vasoactive
amines from mast cells, other cells)

Bradykinin

Leukotrienes C4, D4, E4

PAF
Substance P

Chemotaxis , leukocyte TNF, IL-1


recruitment and activation
Chemokines

C3a, C5a

Leukotriene B4

(Bacterial products, e.g., N-formyl


methyl peptides)

Fever IL-1, TNF

Prostaglandins

Pain Prostaglandins

Bradykinin

Tissue damage Lysosomal enzymes of leukocytes

Reactive oxygen species

Nitric oxide

IL-1, interleukin-1; PAF, platelet-activating factor; TNF, tumor


necrosis factor

After exiting the circulation, leukocytes emigrate in tissues toward the site
of injury by a process called chemotaxis, which is defined as locomotion
oriented along a chemical gradient. Both exogenous and endogenous
substances can act as chemoattractants. The most common exogenous
agents are bacterial products, including peptides that possess an N-
formylmethionine terminal amino acid, and some lipids. Endogenous
chemoattractants include several chemical mediators (described later): (1)
cytokines, particularly those of the chemokine family (e.g., IL-8); (2)
components of the complement system, particularly C5a; and (3)
arachidonic acid (AA) metabolites, mainly leukotriene B4 (LTB4).
leukocytes migrate toward the inflammatory stimulus in the direction of
the gradient of locally produced chemoattractants

Thus from the above explanation it is clear that the most direct and
immediate effect of chemotaxis is migration of leucocytes along the
chemical gradient that is unidirectional motion. Phagocytosis and
Augmented oxygen dependant bactericidal effect will occur but as a
delayed result and not neccesarily a direct consequence of chemotaxis.

(Q.45) PANETH CELLS have


(a) High zinc content

(b) Numerous lysozyme granules

(c) Rich RER

(d) Foamy cell

Your Response :

Correct Answer : b

Exp: B. Numerous lysozyme granules

Ref: Wheaters functional histology , 5th edition

The intestinal villi V are lined by a simple columnar epithelium which is


continuous with that of the crypts C As in other parts of the
gastrointestinal tract, the epithelium includes a variety of cell types, each
with its own specific function. Cell types in the small intestine epithelium
include:

Enterocytes, the most numerous cell type, are tall columnar cells with
surface microvilli. These cells are the main absorptive cells.

Goblet cells are scattered among the enterocytes and produce mucin for
lubrication of the intestinal contents and protection of the epithelium.

Paneth cells are found at the base of the crypts and are distinguished by
their prominent, eosinophilic apical granules. These cells have a defensive
function.

Neuroendocrine cells produce locally acting hormones that regulate


gastrointestinal motility and secretion.

Stem cells, found at the base of the crypts, divide continuously to


replenish all of the above four cell types.

Intraepithelial lymphocytes, which are mostly T cells, provide defence


against invasive organisms

Paneth cells P, which form part of the innate immune system, exhibit
intensely eosinophilic apical cytoplasmic granules; these are stained
bright scarlet by the phloxine-tartrazine method .The granules of
Paneth cells contain antimicrobial peptides (defensins), and
protective enzymes such as lysozyme and phospholipase A. These
products, secreted into the small bowel, provide the first line of
defence against any pathogens that survive passage through the
stomach. The lumen of the small bowel is virtually sterile. Paneth
cells are long-lived (weeks) in comparison to the short lifespan (3-5
days) of enterocytes and goblet cells.

(Q.46) Quantitative cultures are necessary for laboratory diagnosis in infection of

(a) Urinary tract

(b) Blood stream

(c) Lungs

(d) Small intestine

Your Response :

Correct Answer : a

Exp: A. Urinary tract

Useful diagnostic tools include the urine dipstick test and urinalysis, both
of which provide point-of-care information, and the urine culture, which
can retrospectively confirm a prior diagnosis. Understanding the
parameters of the dipstick test is important in interpreting its results. Only
members of the family Enterobacteriaceae convert nitrate to nitrite, and
enough nitrite must accumulate in the urine to reach the threshold of
detection. If a woman with acute cystitis is forcing fluids and voiding
frequently, the dipstick test for nitrite is less likely to be positive, even
when E. coli is present. The leukocyte esterase test detects this enzyme in
the host's polymorphonuclear leukocytes in the urine, whether the cells
are intact or lysed. Many reviews have attempted to describe the
diagnostic accuracy of dipstick testing. The bottom line for clinicians is
that a urine dipstick test can confirm the diagnosis of uncomplicated
cystitis in a patient with a reasonably high pretest probability of this
disease. Either nitrite or leukocyte esterase positivity can be interpreted as
a positive result. Blood in the urine may also suggest a diagnosis of UTI.
A dipstick test negative for both nitrite and leukocyte esterase in the same
type of patient should prompt consideration of other explanations for the
patient's symptoms and collection of urine for culture. A negative dipstick
test is not sufficiently sensitive to rule out bacteriuria in pregnant women,
in whom it is important to detect all episodes of bacteriuria. Performance
characteristics of the dipstick test differ in men (highly specific) and in
noncatheterized nursing home residents (highly sensitive).

Urine microscopy reveals pyuria in nearly all cases of cystitis and


hematuria in ~30% of cases. In current practice, most hospital laboratories
use an automated system rather than manual examination for urine
microscopy. A machine aspirates a sample of the urine and then classifies
the particles in the urine by size, shape, contrast, light scatter, volume, and
other properties. These automated systems can be overwhelmed by high
numbers of dysmorphic red blood cells, white blood cells, or crystals; in
general, counts of bacteria are less accurate than are counts of red and
white blood cells. Our clinical recommendation is that the patient's
symptoms and presentation should outweigh an incongruent result on
automated urinalysis.

The detection of bacteria in a urine culture is the diagnostic "gold


standard" for UTI; unfortunately, however, culture results do not become
available until 24 h after the patient's presentation. Identifying specific
organism(s) can require an additional 24 h. Studies of women with
symptoms of cystitis have found that a colony count threshold of >102
bacteria/mL is more sensitive (95%) and specific (85%) than a
threshold of 105/mL for the diagnosis of acute cystitis in women. In
men, the minimal level indicating infection appears to be 103/mL.
Urine specimens frequently become contaminated with the normal
microbial flora of the distal urethra, vagina, or skin. These contaminants
can grow to high numbers if the collected urine is allowed to stand at
room temperature. In most instances, a culture that yields mixed bacterial
species is contaminated except in settings of long-term catheterization,
chronic urinary retention, or the presence of a fistula between the urinary
tract and the gastrointestinal or genital tract.

Asymptomatic Bacteriuria

The diagnosis of ABU involves both microbiologic and clinical criteria.


The microbiologic criterion is usually >105 bacterial cfu/mL except in
catheter-associated disease, in which case >102 cfu/mL is the cutoff.
The clinical criterion is that the person has no signs or symptoms referable
to UTI.

(Q.47) In first week of illness, laboratory diagnosis of dengue is commonly done by

(a) Suckling mouse inoculation

(b) Chick embryo culture

(c) NS 1 antigen detection

(d) IgG antibody detection

Your Response :

Correct Answer : c

Exp: C. NS1 antigen detection

Testing Algorithms for Dengue:

a.PCR
DENV can be detected in the blood (serum) from patients for
approximately the first 5 days of symptoms. Currently, several PCR tests
are employed to detect the viral genome in serum. In addition, virus can
be isolated and sequenced for additional characterization. Real time RT
PCR assays have been developed and automated; but none of these tests
are yet commercially available. Because antibodies are detected later,
RTPCR has become a primary tool to detect virus early in the course of
illness. Current tests are between 80-90% sensitive, and more that 95%
specific. A positive PCR result is a definite proof of current infection and
it usually confirms the infecting serotype as well. However, a negative
result is interpreted as "indeterminate". Patients receiving negative results
before 5 days of illness are usually asked to submit a second serum
sample for serological confirmation after the 5th day of illness (bellow).

b.MAC ELISA

IgM antibody capture ELISA (MAC-ELISA) format is most commonly


employed in diagnostic laboratories and commercial available diagnostic
kits. The assay is based on capturing human IgM antibodies on a
microtiter plate using anti-human-IgM antibody followed by the addition
of dengue virus specific antigen (DENV1-4). The antigens used for this
assay are derived from the envelope protein of the virus. One of the
limitation of this testing is the cross reactivity between other circulating
flaviviruses. This limitation must be considered when working in regions
where multiple flaviviruses co-circulate. IgM detection is not useful for
dengue serotype determination due to cross-reactivity of the antibody.

c.IgG ELISA

The IgG ELISA used for the detection of a past dengue infection utilizes
the same viral antigens as the MAC ELISA. This assay correlates with the
hemagglutination assay (HI) previously used. In general IgG ELISA lacks
specificity within the flavivirus serocomplex groups. Primary versus
secondary dengue infection can be determined using a simple algorithm.
Samples with a negative IgG in the acute phase and a positive IgG in the
convalescent phase of the infection are primary dengue infections.
Samples with a positive IgG in the acute phase and a 4 fold rise in IgG
titer in the convalescent phase (with at least a 7 day interval between the
two samples) is a secondary dengue infection.

d.NS1 ELISA

The non-structural protein 1 (NS1) of the dengue viral genome has


been shown to be useful as a tool for the diagnosis of acute dengue
infections. Dengue NS1 antigen has been detected in the serum of
DENV infected patients as early as 1 day post onset of symptoms
(DPO), and up to 18 DPO. The NS1 ELISA based antigen assay is
commercially available for DENV and many investigators have evaluated
this assay for sensitivity and specificity. The NS1 assay may also be
useful for differential diagnostics between flaviviruses because of the
specificity of the assay.

e.PRNT

Plaque Reduction and Neutralization Test (PRNT) and the


microneutralization PRNT can be used when a serological specific
diagnostic is required, as this assay is the most specific serological tool
for the determination of dengue antibodies The PRNT test is used to
determine the infecting serotype in convalescent sera. This assay
measures the titer of the neutralizing antibodies in the serum of the
infected individual and determines the level of protective antibodies this
individual has towards the infecting virus. The assay is a biological assay
based on the principle of interaction of virus and antibody resulting in
inactivation of virus such that it is no longer able to infect and replicate in
cell culture. Some of the variability of this assay is differences in
interpretation of the results because of the cell lines and virus seeds used
as well as the dilution of the sera.

To choose between IgG and NS1 is not difficult, as the NS1 is readily
available and widely used today and is proven better by various studies
conducted.

(Q.48) A person is positive for HBsAg, HBeAg and Anti-HBc - IgM, in relation to hepatitis

B, this is a case of

(a) Acute infection

(b) Chronic infection

(c) Remote infection

(d) Highly infections acute infection

Your Response :

Correct Answer : d

Exp: D. Highly infectious acute infection.

Commonly Encountered Serologic Patterns of Hepatitis B Infection

Commonly
Encountered
Serologic
Patterns of
Hepatitis B
Infection

HBsAg Anti- Anti- HBeAg Anti- Interpretation


HBs HBc HBe

+ IgM + Acute hepatitis B, high infectivity

+ IgG + Chronic hepatitis B, high


infectivity

+ IgG + Late acute or chronic hepatitis B,


low infectivity HBeAgnegative
("precoremutant") hepatitis B
(chronic or, rarely, acute)

+ + + +/ +/ HBsAg of one subtype and


heterotypic antiHBs (common)

Process of seroconversion from


HBsAg to anti HBs (rare)

IgM +/ +/ Acute hepatitis B

AntiHBc "window"

IgG +/ Lowlevel hepatitis B carrier

Hepatitis B in remote past

+ IgG +/ Recovery from hepatitis B

+ Immunization with HBsAg (after


vaccination)

Hepatitis B in the remote past (?)

Falsepositive

After a person is infected with HBV, the first virologic marker detectable
in serum within 112 weeks, usually between 812 weeks, is HBsAg.
Circulating HBsAg precedes elevations of serum aminotransferase
activity and clinical symptoms by 26 weeks and remains detectable
during the entire icteric or symptomatic phase of acute hepatitis B and
beyond. In typical cases, HBsAg becomes undetectable 12 months after
the onset of jaundice and rarely persists beyond 6 months. After HBsAg
disappears, antibody to HBsAg (anti-HBs) becomes detectable in serum
and remains detectable indefinitely thereafter. Because HBcAg is
intracellular and, when in the serum, sequestered within an HBsAg coat,
naked core particles do not circulate in serum and, therefore, HBcAg is
not detectable routinely in the serum of patients with HBV infection. By
contrast, anti-HBc is readily demonstrable in serum, beginning within the
first 12 weeks after the appearance of HBsAg and preceding detectable
levels of anti-HBs by weeks to months. Because variability exists in the
time of appearance of anti-HBs after HBV infection, occasionally a gap of
several weeks or longer may separate the disappearance of HBsAg and
the appearance of anti-HBs. During this "gap" or "window" period, anti-
HBc may represent the only serologic evidence of current or recent HBV
infection, and blood containing anti-HBc in the absence of HBsAg and
anti-HBs has been implicated in the development of transfusion-
associated hepatitis B. In part because the sensitivity of immunoassays for
HBsAg and anti-HBs has increased, however, this window period is rarely
encountered. In some persons, years after HBV infection, anti-HBc may
persist in the circulation longer than anti-HBs. Therefore, isolated anti-
HBc does not necessarily indicate active virus replication; most instances
of isolated anti-HBc represent hepatitis B infection in the remote past.
Rarely, however, isolated anti-HBc represents low-level hepatitis B
viremia, with HBsAg below the detection threshold; occasionally, isolated
anti-HBc represents a cross-reacting or false-positive immunologic
specificity. Recent and remote HBV infections can be distinguished by
determination of the immunoglobulin class of anti-HBc. Anti-HBc of the
IgM class (IgM anti-HBc) predominates during the first six months after
acute infection, whereas IgG anti-HBc is the predominant class of anti-
HBc beyond six months. Therefore, patients with current or recent acute
hepatitis B, including those in the anti-HBc window, have IgM anti-HBc
in their serum. In patients who have recovered from hepatitis B in the
remote past as well as those with chronic HBV infection, anti-HBc is
predominantly of the IgG class. Infrequently, in < 15% of patients with
acute HBV infection, levels of HBsAg are too low to be detected; in such
cases, the presence of IgM anti-HBc establishes the diagnosis of acute
hepatitis B. When isolated anti-HBc occurs in the rare patient with
chronic hepatitis B whose HBsAg level is below the sensitivity threshold
of contemporary immunoassays (a low-level carrier), the anti-HBc is of
the IgG class. Generally, in persons who have recovered from hepatitis B,
anti-HBs and anti-HBc persist indefinitely.

The other readily detectable serologic marker of HBV infection, HBeAg,


appears concurrently with or shortly after HBsAg. Its appearance
coincides temporally with high levels of virus replication and reflects the
presence of circulating intact virions and detectable HBV DNA (with the
notable exception of patients with precore mutations who cannot
synthesize HBeAgsee "Molecular Variants"). Pre-S1 and pre-S2
proteins are also expressed during periods of peak replication, but assays
for these gene products are not routinely available. In self-limited HBV
infections, HBeAg becomes undetectable shortly after peak elevations in
aminotransferase activity, before the disappearance of HBsAg, and anti-
HBe then becomes detectable, coinciding with a period of relatively lower
infectivity. Because markers of HBV replication appear transiently during
acute infection, testing for such markers is of little clinical utility in
typical cases of acute HBV infection. In contrast, markers of HBV
replication provide valuable information in patients with protracted
infections.

(Q.49) The most suitable disinfectant for decontamination of HIV contaminated endoscope
is

(a) 1% sodium hypochlorite

(b) 2% glutaraldehyde

(c) 5% phenol

(d) 70% ethanol

Your Response :

Correct Answer : b

Exp: B. 2 % glutaraldehyde

Concern about contamination of fibreoptic endoscopes with human


immunodeficiency virus (HIV) has generated a variety of disruptive and
possibly unnecessary infection control practices in endoscopy units.
Current recommendations on the cleaning and disinfection of endoscopes
have been formulated without applied experimental evidence of the
effective removal of HIV from endoscopes. To study the kinetics of
elimination of HIV from endoscope surfaces, we artificially contaminated
the suction-biopsy channels of five Olympus GIF XQ20 endoscopes with
high titre HIV in serum. The air and water channels of two instruments
were similarly contaminated. Contamination was measured by irrigating
channels with viral culture medium and collecting 3 ml at the distal end
for antigen immunoassay. Endoscopes were then cleaned manually in
neutral detergent according to the manufacturer's recommendations and
disinfected in 2% alkaline glutaraldehyde (Cidex, Surgikos) for two, four,
and ten minutes. Contamination with HIV antigens was measured before
and after cleaning and after each period of disinfection. Initial
contamination comprised 4.8 x 10(4) to 3.5 x 10(6) pg HIV antigen/ml.
Cleaning in detergent achieved a reduction to 165 pg/ml (99.93%) on one
endoscope and to undetectable levels (100%) on four. After two minutes
in alkaline glutaraldehyde all samples were negative and remained
negative after the longer disinfection times. Air and water channels, where
contaminated, were tested after 10 minutes' disinfection and were
negative. These findings underline the importance of cleaning in
removing HIV from endoscope and indicate that the use of dedicated
equipment and long disinfection times are unnecessary.

(Q.50) For determining the efficacy of sterilization in an autoclave, the spores of the
following organism are used as test organisms

(a) Bacillus cereus

(b) Clostridium perfringens

(c) Bacillus stearothermophilus

(d) Clostridium histolyticum

Your Response :

Correct Answer : c

Exp: C. Bacillus stearothermophilus

Sterilization procedures should be monitored through a combination of


mechanical, chemical, and biological techniques designed to evaluate the
sterilizing conditions and the procedure's effectiveness.

Mechanicaltechniques for monitoring sterilization include assessing the


cycle time, temperature, and pressure of sterilization equipment by
observing the gauges or displays on the sterilizer. Some tabletop
sterilizers have recording devices that print out these parameters. Correct
readings do not ensure sterilization, but incorrect readings could be the
first indication that a problem has occurred with the sterilization cycle.

Chemical indicators, internal and external, use sensitive chemicals to


assess physical conditions such as temperature during the sterilization
process. Chemical indicators such as heat sensitive tape change color
rapidly when a given parameter is reached. An internal chemical indicator
should be placed in every sterilization package to ensure the sterilization
agent has penetrated the packaging material and actually reached the
instruments inside. An external indicator should be used when the internal
indicator cannot be seen from outside the package. Single-parameter
internal indicators provide information on only one sterilization parameter
and are available for steam, dry heat, and unsaturated chemical vapor.
Multiparameter internal indicators measure 23 parameters and can
provide a more reliable indication that sterilization conditions have been
met. Multiparameter internal indicators are only available for steam
sterilizers (i.e., autoclaves). Refer to manufacturer instructions for proper
use and placement of chemical indicators.

Indicator test results are shown immediately after the sterilization cycle is
complete and could provide an early indication of a problem and where
the problem occurred in the process. If the internal or external indicator
suggests inadequate processing, the item that has been processed should
not be used. Because chemical indicators do not prove sterilization has
been achieved, a biological indicator (i.e., spore test) is required.

Biologicalindicators (BIs) are the most accepted means of monitoring


the sterilization process because they directly determine whether the
most resistant microorganisms (e.g., Geobacillus or Bacillus species)
are present rather than merely determine whether the physical and
chemical conditions necessary for sterilization are met. Because spores
used in BIs are more resistant and present in greater numbers than are the
common microbial contaminants found on patient care equipment, an
inactivated BI indicates that other potential pathogens in the load have
also been killed.

Geobacillus stearothermophilus(formallyBacillus stearothermophilus) is


a rod-shaped,Gram-positivebacteriumand a member of the
divisionFirmicutes. The bacteria is athermophileand is widely distributed
in soil, hot springs, ocean sediment, and is a cause of spoilage in food
products. It will grow within a temperature range of 30-75 degrees
Celsius. Some strains are capable of oxidizing carbon monoxide
aerobically. It is commonly used as a challenge organism
forsterilizationvalidation studies and periodic check of sterilization cycles.
Thebiological indicatorcontainssporesof the organism on filter paper
inside a vial. After sterilizing, the cap is closed, an ampule of growth
medium inside of the vial is crushed and the whole vial isincubated. A
color and/orturbiditychange indicates the results of the sterilization
process; no change indicates that the sterilization conditions were
achieved, otherwise the growth of the spores indicates that the
sterilization process has not been met. It was first described in 1920
asBacillus stearothermophilus, but, together withBacillus
thermoglucosidasius, it was reclassified as a member of the
genusGeobacillusin 2001

(Q.51) Which of the following has No Cystic stage?

(a) Trichomonas vaginalis

(b) Entamoeba histolytica

(c) Ascaris lumbricoides

(d) Giardia lamblia

Your Response :

Correct Answer : a

Exp: A. Trichomonas vaginalis


Ref. Panicker 5th Edition Pages 29 and 37, Textbook of Medical
Parasittology, P.Chakraborty Pg 43.

i Trichomonas vaginalis occurs only as trophozoite, there being no


cystic form in Trichomonas

ii Entamoeba histolytica produces a spherical cyst, about 10-15 mm


(micron) in size. The early cyst has single nucleus and the mature cyst is
quadrinucleate

iii Dientamoeba fragilis is the smallest of the intestinal amoeba, 5 to 8


mm (micron) in size and is seen only rarely. It has two nuclei and the
cytoplasm is fragile. It has no cystic phase and is non pathogenic

iv Giardia lamblia encysts in the colon and the cyst is ovoid about 12
mm (micron) by 8 mm (micron) and surrounded by a tough hyaline cyst
wall. The young cyst contains two and the mature cyst four nuclei situated
at one end.

IMPORTANT POINTS ABOUT Trichomonas vaginalis :

Shape and size Pear shaped, 7 -23 micrometer wide;6-8


microm wide

Motility Jerky, rapid

Nucleus One

Flagella 3-5 anterior; 1 curving as undulating


membrane extending half the body length

Infective form Trophozoite

Habitat Vagina (Male : urethra)

Clinical findings Leukorrhea, pruritis vulvae

Other sites of infection Urethra (prostrate in men and females)

Stage recovered during Trophozoite only (No cystic phase)


clinical phase

Other features Seen in urine, urethral discharges, vaginal


smears

(Q.52) Which of the followingare the special laboratory conditions needed to recover
Campylobacter jejuni?

(a) 37C (98.6F) aerobic on blood agar plates


(b) 37C (98.6F) anaerobic on blood agar plates

(c) 42C (107.6F) microaerophilic on Skirrow's medium

(d) 42C (107.6F) aerobic on Skirrow's medium

Your Response :

Correct Answer : c

Exp: C. 42C (107.6F) microaerophilic on Skirrow's medium

A. Campylobacter species are small motile, non-spore-forming,


comma-shaped, gram-negative bacilli, best grown in a microaerophilic
environment at 42C (107.6F).

B. Guillain-Barre is a rare neurological complication of C. jejuni


gastroenteritis.

C. C. jejuni gastroenteritis is usually self-limited; however, if


necessary, erythromycin is the drug of choice.

D. Campylobacter infection most often occurs several days after


consumption of undercooked chicken.

E. Symptom of fever and abdominal pain may initially mimic


appendicitis.

F. The isolation and identification of Campylobacter jejuni can be


achieved using special culture characteristics. Three requirements must be
met. First, a selective medium is needed. There are several selective
media in widespread use: Skirrow's medium uses vancomycin, polymyxin
B, and trimethoprim in 7% lysed blood agar; other selective media contain
cefoperazone, other antimicrobials, and inhibitory compounds. The
selective media are suitable for isolation of C. jejuni at 42C (107.6F);
when incubated at 36

G. -37C (96.8-98.6F), other Campylobacters and bacteria may be


isolated. Finally, incubation must be in an atmosphere with reduced
oxygen and added carbon dioxide. The colonies appear to be colorless or
gray and may be watery and spreading or round and convex.

ORGANSISM SPECIAL AGARS USED

Staph. aureus Salt milk agar, Salt broth, Ludlams


medium, Polymixin containing
media, Robertsons cooked meat
medium

N. gonorrhea Modified Thayer Martin agar


Salmonella and Shigella Deoxycholate citrate agar

B. anthrax PLET

B. cereus MYPA

Shigella Hektoen enteric agar

H. influenzae Levinthals medium

Fildes agar

Boiled blood agar

Bordetella pertusis Bordet-Gengou-Glycerine-Potato-


Blood agar

Laceys DFP

Regan Lowe medium

Charcoal blood agar

Transport media for Bordetella Modified Stuart medium


pertusis

Bacteriodes Brain heart infusion agar

Calymmatobacterium granulomatis Modified Levinthal agar

Campylobacter jejuni Skirrows media, Butzlers,


Campy BAP medium

Legionella BCYE agar

Borrelia burgdorferi Modified Kellys (BSK) medium

Leptospira Korthofs, Stuarts, Fletchers,


EMJH medium

(Q.53) To which group does the verotoxin producing E. coli 0157:H7serotype belong?

(a) Enteroaggregative E. coli (EAEC).

(b) Enterohemorrhagic E. coli (EHEC).

(c) Enteroinvasive E. coli (EIEC).

(d) Enteropathogenic E. coli (EPEC).

Your Response :
Correct Answer : b

Exp: B. Enterohemorrhagic E. coli (EHEC)

All of the above classes of E. coli cause diarrhea. However, only EHEC
produce a verotoxin that has many properties that are similar to Shiga
toxin.

EHEC has been associated with hemorrhagic colitis, a severe form of


diarrhea, and with hemolytic uremic syndrome (HUS). HUS is a disease
resulting in acute renal failure, microangiopathic hemolytic anemia, and
thrombocytopenia.

Extra Edge: Harrison, 17th edition, page 940

Shiga ToxinProducing and Enterohemorrhagic E. coli

i STEC/EHEC strains constitute an emerging group of pathogens that


can cause hemorrhagic colitis and the hemolytic-uremic syndrome (HUS).

ii Several large outbreaks resulting from the consumption of


undercooked ground beef and other foods (e.g., fresh spinach) have
received significant media attention.

iii O157:H7 is the most prominent serotype, but serogroups O6, O26,
O55, O91, O103, O111, O113, and OX3 have also been associated with
these syndromes.

iv The ability of STEC/EHEC to produce Shiga toxin (Stx2 and/or


Stx1) or related toxins is a critical factor in the expression of clinical
disease.

v Shigella dysenteriae strains that produce the closely related Shiga


toxin Stx can cause the same syndrome. Stx2 appears to be more
important than Stx1 in the development of HUS.

vi All Shiga toxins studied to date are multimers composing one


enzymatically active A subunit and five identical B subunits that mediate
binding to globoceramides.

vii The A subunit cleaves an adenine from the host cell's 28S rRNA,
thereby irreversibly inhibiting ribosomal function.

viii Therefore, Shiga toxins belong to the class of toxins known as


ribosome-inactivating proteins.

(Q.54) Which of the following is true regardingpasteurization

(a) A few sporing bacteria are get killed by pasteurization


(b) Milk is heated to 70 degree C and held for 30 minutes

(c) Milk is rapidly cooled to zero degrees centigrade

(d) It is done for checking milk-borne infections

Your Response :

Correct Answer : d

Exp: Ans D)It is done for checking milk-borne infections

ORGANSISM SPECIAL AGARS USED

Staph. aureus Salt milk agar, Salt broth, Ludlams


medium, Polymixin containing
media, Robertsons cooked meat
medium

N. gonorrhea Modified Thayer Martin agar

Salmonella and Shigella Deoxycholate citrate agar

B. anthrax PLET

B. cereus MYPA

Shigella Hektoen enteric agar

H. influenzae Levinthals medium

Fildes agar

Boiled blood agar

Bordetella pertusis Bordet-Gengou-Glycerine-Potato-


Blood agar

Laceys DFP

Regan Lowe medium

Charcoal blood agar

Transport media for Bordetella Modified Stuart medium


pertusis

Bacteriodes Brain heart infusion agar

Calymmatobacterium granulomatis Modified Levinthal agar


Campylobacter jejuni Skirrows media, Butzlers,
Campy BAP medium

Legionella BCYE agar

Borrelia burgdorferi Modified Kellys (BSK) medium

Leptospira Korthofs, Stuarts, Fletchers,


EMJH medium

(Q.55) Which of the following is true regardingexotoxin:

(a) They are very stable and resist most physical and chemical agents

(b) Highly antigenic

(c) Produced abundantly by gram negative bacilli

(d) Lipopolysaccharide-protein complex in nature

Your Response :

Correct Answer : b

Exp: Ans B Highly antigenic

ORGANSISM SPECIAL AGARS USED

Staph. aureus Salt milk agar, Salt broth, Ludlams


medium, Polymixin containing
media, Robertsons cooked meat
medium

N. gonorrhea Modified Thayer Martin agar

Salmonella and Shigella Deoxycholate citrate agar

B. anthrax PLET

B. cereus MYPA

Shigella Hektoen enteric agar

H. influenzae Levinthals medium

Fildes agar

Boiled blood agar

Bordetella pertusis Bordet-Gengou-Glycerine-Potato-


Blood agar

Laceys DFP

Regan Lowe medium

Charcoal blood agar

Transport media for Bordetella Modified Stuart medium


pertusis

Bacteriodes Brain heart infusion agar

Calymmatobacterium granulomatis Modified Levinthal agar

Campylobacter jejuni Skirrows media, Butzlers,


Campy BAP medium

Legionella BCYE agar

Borrelia burgdorferi Modified Kellys (BSK) medium

Leptospira Korthofs, Stuarts, Fletchers,


EMJH medium

(Q.56) Tranexamic acid is a

(a) Antithrombotic

(b) Antifibrinolytic

(c) Fibrinolytic

(d) Styptic

Your Response :

Correct Answer : b

Exp: Ans B) Antifibrinolytic

Tranexamic acid is an antifibrinolytic. It works by preventing blood clots


from breaking down too quickly. This helps to reduce excessive bleeding.

(Q.57) Which of the following is longest actingcorticosteroid ?

(a) Dexamethasone

(b) Prednisolone
(c) Triamcinolone

(d) Hydrocortisone

Your Response :

Correct Answer : a

Exp: Ans A) Dexamethasone

Short-Acting Injectable Corticosteroids

A. Cortisone

(Q.58) The most effective antidote for belladonna poisoning is

(a) Neostigmine

(b) Physostigmine

(c) Pilocarpine

(d) Methacholine

Your Response :

Correct Answer : b

Exp: Ans B) Physostigmine

Decontamination Use / Indication Comment / Dose

Activated Charcoal(without Consider use within 1 g/kg (up to


sorbitol) one hour of a life- 100g)
threatening ingestion

Other Decontamination Discuss specific


Techniques(e.g. multidose indications withyour
charcoal, dialysis etc) regional poison center

(Q.59) Which of the following is resistant to both true and pseudo cholinesteraseenzymes ?

(a) Bethanechol

(b) Acetylcholine

(c) Methacholine
(d) Pilocarpine

Your Response :

Correct Answer : a

Exp: Ans A) Bethanechol

Acetylcholinesterase

Sites of location

- Cholinergic neurons - Cholinergic synapses - Neuromuscular


junction - Red blood cells

Substrates

Acetylcholine is the best substrate

Methacholine is a substrate

Hydrolyzes ACh at greater velocity than choline esters with acyl groups
larger than acetate or proprionate

Esters that are not substrates

- Bethanechol - Carbachol - Succinylcholine

Its inhibition produces synergistic interaction with methacholine and


additive actions with bethanechol and carbachol

Drugs that block its hydrolysis of esters are called cholinesterase


inhibitors

Pseudocholinesterase

Sites of location

Plasma, liver, glial cells, other tissues

Substrates

- Butyrylcholine is the best - Acetylcholine - Succinylcholine -


Procaine

Esters that are not substrates

Methacholine, bethanechol, and carbachol

Is inhibited by carbamyl and organophosphate inhibitors of


acetylcholinesterase
(Q.60) Mecasermin is indicated in

(a) Active or suspected neoplasia

(b) Intracranial hypertension

(c) Impaired growth due to GH-deficiency

(d) Enlarged tonsils due to lymphoid hypertrophy

Your Response :

Correct Answer : c

Exp: Ans C) Impaired growth due to GH-deficiency

Mecasermin is recombinant human insulin-like growth factor-1 (rhIGF-1)


designed for use as replacement therapy in severe primary insulin-like
growth factor deficiency.

(Q.61) NSAIDS are commonly used as first line therapy in the pain of

(a) Malignancy

(b) Migraine

(c) Neuropathy

(d) Visceral pain except dysmenorrhea

Your Response :

Correct Answer : b

Exp: Ans B) Migraine

Treatment Approaches

Migraine treatment involves both treating acute attacks when they occur
and developing preventive strategies for reducing the frequency and
severity of attacks.

Treating Migraine Attacks

There are many medications for treating a migraine attack. Still, many
patients are treated with unapproved drugs, including opoids and
barbiturates that can be potentially addictive or dangerous.

The main types of medications for treating a migraine attack are:

Pain relievers [usually nonprescription nonsteroidal anti-inflammatory


drugs (NSAIDs) or acetaminophen]

Ergotamines

Triptans

It is best to treat a migraine attack as soon as symptoms first occur.


Doctors generally recommend:

Start with nonprescription pain relievers for mild-to-moderate attacks. If


migraine pain is severe, a prescription version of an NSAID may be
recommended.

A triptan is generally the next drug of choice.

Ergotamine drugs tend to be less effective than triptans but are helpful for
some patients.

Depending on the severity of the attacks, and accompanying symptoms,


the doctor may recommend taking a triptan or ergotamine drug in tablet,
injection, or suppository form. The doctor may also prescribe specific
medications for treating symptoms such as nausea.

Try to guard against medication overuse, which can cause a rebound


effect. Nearly all pain relief drugs used for migraine can cause rebound
headache, and patients should not take any of these drugs more than 2
days per week. This is especially true for OTC medications that contain
caffeine. If you find that you need to use acute migraine treatment more
frequently, talk to your doctor about preventive medications.

Preventing Migraine Attacks

Preventive strategies for migraine include both drug treatment and


behavioral therapy or lifestyle adjustments.

Patients should consider using preventive migraine drugs if they have:

Migraines that are not helped by acute treatment drugs

Frequent attacks (more than once per week)

Side effects from acute treatment drugs or contraindications to taking


them

The main preventive drug treatments for migraine are:

Beta-blocker drugs [usually propranolol (Inderal, generic) or timolol


(Blocadren)]

Anti-seizure drugs [usually divalproex (Depakote, generic), valproate


(Depacon, generic) or topiramate (Topamax, generic)]
Tricyclic antidepressants [usually amitriptyline (Elavil, generic)] or the
dual inhibitor antidepressant venlafaxine (Effexor, generic)

The triptan frovatriptan (Frova) for menstruation-associated migraine

(Q.62) Chemokine co-receptor 5 (CCR 5) inhibitor is

(a) Enfuvirtide

(b) Maraviroc

(c) Raltegavir

(d) Atazanavir

Your Response :

Correct Answer : b

Exp: Maraviroc

In August of 2007, the Food and Drug Administration (FDA) approved


the first chemokine (C-C motif) receptor 5 (CCR5) inhibitor, maraviroc,
for treatment-experienced patients infected with R5-using virus.
Chemokine receptor antagonists are the first antiretrovirals to bind a
cellular protein of the host and, as such, engender unique safety concerns.

ANTIRETROVIRAL Indication Toxicity


Drug

NRTI (NUCLEOSIDE REVERSE TRANSCRIPTASE INHIBITOR)

Zidovudine (AZT, Treatment of HIV Anemia,


azidothymidine, infection in granulocytopenia,
Retrovir, 3'azido-3'- combination with myopathy, lactic
deoxythymidine) other antiretroviral acidosis, hepatomegaly
agents with steatosis, headache,
nausea

Prevention of
maternal-fetal
HIVtransmission

Didanosine (ddI, For treatment of HIV Pancreatitis, peripheral


dideoxyinosine, 2',3'- infection in neuropathy,
dideoxyinosine) combination with abnormalities on liver
other antiretroviral function tests, lactic
agents acidosis, hepatomegaly
with steatosis
Zalcitabine (ddC, In combination with Peripheral neuropathy,
HIVID, 2'3'- other antiretroviral pancreatitis, lactic
dideoxycytidine) agents for the acidosis, hepatomegaly
treatment of HIV with steatosis, oral ulcers
infection

Stavudine (d4T, Zerit, Treatment of HIV- Peripheral neuropathy,


2'3'-didehydro-3'- infected patients in pancreatitis, lactic
dideoxythymidine) combination with acidosis, hepatomegaly
other antiretroviral with steatosis, ascending
agents neuromuscular
weakness, lipodystrophy

Lamivudine ( 2'3'- In combination with Hepatotoxicity


dideoxy-3'-thiacytidine, other antiretroviral
3TC) agents for the
treatment of HIV
infection

Emtricitabine (FTC) In combination with Hepatotoxicity


other antiretroviral
agents for the
treatment of HIV
infection

Abacavir For treatment of HIV Hypersensitivity reaction


infection in (can be fatal); fever,
combination with rash, nausea, vomiting,
other antiretroviral malaise or fatigue, and
agents loss of appetite

Tenofovir For use in Potential for renal


combination with toxicity
other antiretroviral
agents when
treatment is indicated

NNRTI (NONNUCLEOSIDE REVERSE TRANSCRIPTASE


INHIBITOR)

Delavirdine For use in Skin rash, abnormalities


combination with in liver function tests
appropriate
antiretrovirals when
treatment is
warranted

Nevirapine In combination with Skin rash, hepatotoxicity


other antiretroviral
agents for treatment
of progressive HIV
infection

Efavirenz For treatment of HIV Rash, dysphoria,


infection in elevated liver function
combination with tests, drowsiness,
other antiretroviral abnormal dreams,
agents depression

Etravirine Pending Rash, headache,


dizziness, nausea,
diarrhea

PROTEASE INHIBITORS

Saquinavir mesylate In combination with Diarrhea, nausea,


(Invirasehard gel other antiretroviral headaches,
capsule) agents when therapy hyperglycemia, fat
is warranted redistribution, lipid
abnormalities

(Fortovasesoft gel For use in Diarrhea, nausea,


capsule) combination with abdominal pain,
other antiretroviral headaches,
agents when hyperglycemia, fat
treatment is redistribution, lipid
warranted abnormalities

Ritonavir In combination with Nausea, abdominal pain,


other antiretroviral hyperglycemia, fat
agents for treatment redistribution, lipid
of HIV infection abnormalities, may alter
when treatment is levels of many other
warranted drugs, including
saquinavir

Indinavir sulfate For treatment of HIV Nephrolithiasis, indirect


infection in hyperbilirubinemia,
combination with hyperglycemia, fat
other antiretroviral redistribution, lipid
agents when abnormalities
antiretroviral
treatment is
warranted

Nelfinavir mesylate For treatment of HIV May contain traces of the


infection in potential
combination with carcinogen/teratogen
other antiretroviral ethyl methane sulfonate
agents when
antiretroviral therapy
is warranted

Amprenavir In combination with Nausea, vomiting,


other antiretroviral diarrhea, rash, oral
Fosamprenavir agents for treatment paresthesias, elevated
of HIV infection liver function tests,
hyperglycemia, fat
redistribution, lipid
abnormalities

Lopinavir/ritonavir For treatment of HIV Diarrhea, hyperglycemia,


infection in fat redistribution, lipid
combination with abnormalities
other antiretroviral
agents

Atazanavir For treatment of HIV Hyperbilirubinemia, PR


infection in prolongation, nausea,
combination with vomiting,
other antiretroviral hyperglycemia, fat
agents maldistribution

Tipranavir In combination with Diarrhea, nausea,


200 mg ritonavir for fatigue, headache, skin
combination therapy rash, hepatotoxicity,
in treatment- intracranial hemorrhage
experienced adults

Darunavir In combination with Diarrhea, nausea,


100 mg ritonavir for headache
combination therapy
in treatment-
experienced adults

ENTRY INHIBITORS

EnfuvirtideQ In combination with Local injection reactions,


other agents in hypersensitivity
treatment- reactions, increased rate
experienced patients of bacterial pneumonia
with evidence of
HIV-1 replication
despite ongoing
antiretroviral therapy

Maraviroc In combination with Hepatotoxicity,


other antiretroviral nasopharyngitis, fever,
agents in treatment cough, rash, abdominal
experienced adults pain, dizziness, fever,
infected with only musculoskeletal
CCR5-tropic HIV-1 symptoms
that is resistant to
multiple antiretroviral
agents

INTEGRASE INHIBITOR

Raltegravir In combination with Nausea, rash


other antiretroviral
agents in treatment
experienced patients
with evidence of
ongoing HIV-1
replication

Adverse effect Culprit Anti-HIV drug

Anti-HIV drug causes severe hepatotoxicity Zidovudine.


with steatohepatitis

Anemia is the most common side effect of Zidovudine.

Anti-HIV drug known to cause myopathy Zidovudine.


resembling Mitochondrial myopathy with
ragged red Fibers

Local injection reactions, hypersensitivity Enfuvirtide (fusion


reactions, increased rate of bacterial inhibitor).
pneumonia

Anti-retroviral drug is contraindicated in Efavirenz, Abacavir.


pregnancy

Anti-HIV drug that should never be given as re- Abacavir.


challenge once history of producing allergic
reaction with it is known

Peripheral neuropathy in common side effect of Didanosine,


zalcitabine, and
stavudine, indinavir.
Non-nucleoside reverse transcriptase inhibitor Nevirapine.
anti-retroviral drug with mild side effects like
rash and hepatotoxicity

Pancreatitis Didanosine,
Zalcitabine.

Nephrotoxic Tenofovir

Abnormal dreams Efavirenz

Hyperglycemia, fat redistribution, and lipid Saquinavir,


abnormalities Ritonavir, Nelfinavir,
Indinavir.

Nephrolithiasis Indinavir

(Q.63) Tiprolisant is

(a) H3 Receptor agonist

(b) H3 Receptor antagonist

(c) Partial H3 Receptor agonist

(d) Inverse H3 Receptor agonist

Your Response :

Correct Answer : d

Exp: Ans D) Inverse H3 Receptor agonist

Pitolisant or tiprolisant is a histamine receptor inverse agonist/antagonist


selective for the H3 subtype. It has stimulant and nootropic effects in
animal studies, and may have several medical applications, having been
researched for the treatment of narcolepsy, for which it has been granted
orphan drug status in the EU and US. It is currently in clinical trials for
schizophrenia and Parkinson's disease

(Q.64) Cyproheptadine has all of the following actions; EXCEPT

(a) Antimuscarinic actions

(b) Antihistaminic actions

(c) Antiserotonergic actions

(d) Antiadrenergic actions


Your Response :

Correct Answer : d

Exp: Ans D) Antiadrenergic actions

Cyproheptadine, is a first-generation antihistamine with additional


anticholinergic, antiserotonergic, and local anesthetic properties.

Medical uses

Cyproheptadine is used to treat allergic reactions

Has shown effectiveness in the treatment of nightmares including


nightmares related to post traumatic stress disorder

Has been used in the management of moderate to severe cases


of serotonin syndrome, a complex of symptoms associated with the use
of serotonergic drugs, such as selective serotonin reuptake
inhibitors and monoamine oxidase inhibitors), and in cases of
hyperserotoninaemia resulting from a serotonin-producing carcinoid
tumor

Can also be used as a preventive measure against migraine in children and


adolescents.

Can relieve SSRI-induced sexual dysfunction and drug-


induced hyperhidrosis

Also used in the treatment of cyclical vomiting syndrome

Short-term use of the drug (up to 12 weeks) can stimulate the appetite and
may lead to weight gain, which is helpful forunderweight people.

According to a small study, cyproheptadine hydrochloride has been found


to improve sleep, calmness, as well as mood and energy levels, and to
improve both negative and (sometimes even) positive psychotic
symptoms in a subgroup of chronic schizophrenics who do not respond
(either completely or sufficiently) to other therapies.

Cyproheptadine may improve akathisia in patients on antipsychotic


medications.

In clinical trials in which cyproheptadine was used as an adjunct to


antipsychotic treatment for patients with schizophrenia, an improvement
in negative symptoms was seen.

(Q.65) Proton Pump inhibitors are most effective when given

(a) Halfhour before breakfast


(b) With meals

(c) After prolonged fasting

(d) Along with H2 blockers

Your Response :

Correct Answer : a

Exp: Ans A) half hour before breakfast

Proton pump inhibitor (PPI) is a prodrug which is activated by acid.


Activated PPI binds covalently to the gastric H+, K+-ATPase via
disulfide bond. Cys813 is the primary site responsible for the inhibition of
acid pump enzyme, where PPIs bind. The rate of omeprazole absorption is
decreased by concomitant food intake. In addition, the absorption of
lansoprazole and esomeprazole is decreased and delayed by food

(Q.66) Which of the following drug is useful in motionsickness'

(a) Hyoscine

(b) Metoclopramide

(c) Prochlorperazine

(d) Ondansetron

Your Response :

Correct Answer : a

Exp: (A). Hyoscine

Atropine and hyoscine have anti-motion sickness action because they


depress vestibular excitation.

Hyosine is more potent than atropine in this regard.

(Q.67) Misoprostol, prostaglandin analogue is useful as:

(a) Uterine relaxant

(b) Antiulcer drug

(c) Bronchodilator

(d) Vasodilator
Your Response :

Correct Answer : b

Exp: (B). Antiulcer drug

Misoprostol, a prostaglandin analogue used as an anti-ulcer drug, acts


by:

(Q.68) Which of the following antitubercular drug is associated with red green blindness?

(a) Cycloserine

(b) Isoniazid

(c) Pyrazinamide

(d) Ethambutol

Your Response :

Correct Answer : d

Exp: (D). Ethambutol

Ethambutol can cause optic neuritis leading to loss of colour vision (red
green blindness) and visual field defects.

Ethambutol should not be used in children below 6 years of age because


young children are unable to report early visual impairment.

(Q.69) Cardio Vocal Syndrome is also called

(a) Ortners Syndrome

(b) Lermoyez Syndrome

(c) Pendreds Syndrome

(d) Jaccouds Syndrome

Your Response :

Correct Answer : a

Exp: A) Ortners Syndrome

Ortner's syndrome is a rare cardiovocal syndrome and refers to recurrent


laryngeal nerve palsy from cardiovascular disease. The most common
historical cause is a dilated left atrium due to mitral stenosis, but other
causes, including pulmonary hypertension, thoracic aortic aneurysms and
aberrant subclavian artery syndrome have been reported

Some of the Eponyms in Otolaryngology

Abrikossoff tumor

A benign neoplasm that may occur in a variety of visceral, mucosal, and


cutaneous sites but is most frequently seen in the tongue. It usually
presents as a small, firm, circumscribed nodule. Excision is the treatment
of choice. It is also known as agranular cell tumor.

Ackerman tumor

Another name forverrucous carcinoma.

Albright syndrome

Characterized by polyostotic fibrous dysplasia. Scattered melanotic


macular spots and, in females, precocious puberty also are typical. The
bony lesions may occasionally occur in single bones; therefore, this
syndrome should be in the differential diagnosis of any patient presenting
with a bony lesion in the face or jaw.

Alexander syndrome

The least severe type of inner ear dysplasia. The cochlear duct and basilar
turn of the cochlea are usually the only structures affected, resulting in a
high-frequency, rather than a total, hearing loss.

Alport syndrome

An autosomal-dominant syndrome that consists of progressive


glomerulonephritis (usually starting in the mid-teens) and progressive
sensorineural hearing loss. The auditory deficit is symmetric, worse in the
high frequencies, and primarily due to a loss of hair cells.

Apert syndrome

A rare craniofacial disorder occasionally seen by the otolaryngologist


because of the associated hypoplastic maxilla or cleft palate (25%). Some
of these patients reportedly have a conductive hearing loss secondary to
stapes footplate fixation.

Arnold nerve

The auricular branch of the vagus. It arises from the jugular ganglion,
passes through the temporal bone via the mastoid canaliculus, and exits
the skull through the tympanomastoid fissure. It supplies the skin of the
posterior external canal and posterior auricle.
Ascher syndrome

A rare, possibly autosomal-dominant disorder. Manifestations are a loss


of elasticity in the skin of the eyelids (blepharochalasis), goiter, and
edema and thickening of the gingivobuccal mucosa, which gives the
appearance of double lips.

Avellis syndrome

One of a number of syndromes caused by thrombosis of the vertebral


artery. The nucleus ambiguus and the spinothalamic tract are affected,
often with involvement of the bulbar nucleus of the spinal accessory
nerve. This results in an ipsilateral paralysis of the soft palate, pharynx,
and larynx, with loss of pain and temperature sensation on the
contralateral trunk and extremities.

Babinski-Nageotte syndrome

Thought to be caused by multiple infarcts in the distribution of the


vertebral artery. Almost any cranial nerve may be affected; the most
common motor deficits are paralysis of the ipsilateral pharynx, larynx,
soft palate, and sometimes tongue. Possible sensory deficits include loss
of taste and loss of pain and temperature sensation around the face.
Horner syndrome also may be present.

Baelz syndrome

Characterized by onset in childhood or early adolescence of thickening of


the lower lip and hyperplasia of the minor salivary glands in this region.
Papules sometimes form at the duct openings, and saliva can be easily
expressed. Some forms may predispose toward an increased risk of
squamous cell cancer.

Brny syndrome

A combination of unilateral headache in the back of the head, periodic


ipsilateral deafness (alternating with periods of unaffected hearing),
vertigo, and tinnitus. It is also known asBrny benign positional vertigo.

Barre-Lieou syndrome

Consisting of sharp pain beginning in the neck and radiating up to the


occiput and then forward. It is most common in patients between 40 and
60 years of age. Pain is usually on one side and aggravated by certain
movements of the head. Sensory disturbances, including vertigo, tinnitus,
and cloudy vision, may accompany the pain. The cause is not known. It is
also known ascervical migraine.

Barrett esophagitis

Replacement of the squamous epithelium of the distal esophagus by


columnar epithelium, similar to that which lines the stomach. The most
common cause is chronic gastroesophageal reflux, and 2% to 5% of cases
may progress to adenocarcinoma.

Bartholin duct

The major duct of the sublingual gland. It is formed by the confluence of


several of the more anterior small sublingual ducts (ducts of Rivinus) and
empties into the submandibular duct. It is variably present in hum

Battle sign

Ecchymosis over the mastoid process. It is indicative of a temporal bone


or posterior fossa fracture.

(Q.70) Hitzelbergers sign is seen in

(a) Acoustic Neuroma

(b) Glomus Jugulare

(c) Glomus Tympanicum

(d) Menieres disease

Your Response :

Correct Answer : a

Exp: A) Acoustic Neuroma

Hitzelberger sign

anesthesia of medial, posterior, or superior areas of the external auditory


canal caused by an acoustic neuroma that is pressing against the facial
nerve

MC CP angle tumour of 40-60 Acoustic neuroma

MC symptom of acoustic neuroma U/L SNHL

MC presenting sign of advanced loss of corneal reflex(CN5)


acoustic neuroma

Hitzelbergers sign Hypoaethesia of posterior meatal


wall(CN7)

PTA signs of acoustic neuroma dip at 4000, roll over


phenomenon
Gold standard for acoustic neuroma MRI
imaging

Rx of choice for acoustic neuroma stereotactic radiosurgery, gamma


knife

(Q.71) Anterior pillar of tonsil is formed by

(a) Palatopharyngeus muscle

(b) Palatoglossus muscle

(c) Genioglossus muscle

(d) Hyoglossus muscle

Your Response :

Correct Answer : b

Exp: B) Palatoglossus muscle

A ring of lympnoid tissue surround the naso pharynx and oro pharynx.
These lymphoid tissue are collectively known as the waldayer's ring.
Waldayer's ring has two components, namely the inner and outer rings.
The cervical lymph nodes constitute the outer ring, while the inner ring is
constituted by 1. adenoid at the roof of nasopharynx, 2. tubal tonsils or
tonsil of Gerlac which surround the pharyngeal ends of eustachean tube.
These lymphoid tissue surround the naso pharynx.

Anatomy of palatine tonsils: Palatine tonsils are the largest member of


the inner waldayer's ring. It is almond shaped and lie on either side of the
oropharynx. Develomentally tonsils arise from the ventral portion of the
second pharyngeal pouch, i.e. ideally named as sinus tonsillaris. The
trace of this sinus is present in tonsil as supra tonsillar cleft.

Tonsil is lodged in the tonsillar fossa on either side of orophayrnx. The


tonsillar fossa lies between two pillars, anterior and the posterior
pillars. The anterior pillar is formed by palato glossus muscle, the
posterior pillar is formed by palato pharyngeus muscle. The outer aspect
of tonsil is lined by condensed capsule formed by the pharyngobasilar
fascia a specialised portion (it is also known to course the surface of the
tonsil and extend into it to form septa that conduct nerves and vessels),
deep to which lie the superior constrictor muscle, lateral to which is the
bucco pharyngeal fascia. The glossopharyngeal nerve and the stylohoid
ligament pass downwards and forwards beneath the lower edge of the
superior constrictor in the lower part of the tonsillar fossa. These
structures collectively constitute the tonsillar bed. The tonsil is virtually
inseparable from its capsule, but the capsule is united by loose connective
tissue to pharyngeal muscles, hence the tonsillar dissection is carried out
in this plane.

The medial surface of the tonsil is free and faces the oropharynx. It is
covered by non-keratinizing stratified squamous epithelium which is
continuous with that of the lining of the oropharynx. A triangluar fold of
mucous membrane extends back from the paltoglossal fold to cover the
anteroinferior part of the tonsil. This fold of mucous membrane is known
as plica triangularis. In childhood, this fold is usually invaded by
lymphoid tissue and becomes incorporated into the tonsil.A semilunar
fold of mucous membrane passes from the upper aspect of the
palatopharyngeal arch towards the upper pole of tonsil, thus separating it
from the base of the uvula

(Q.72) Tripod fracture involves

(a) Ethmoid complex

(b) Zygomatic complex

(c) Orbital floor

(d) Mandible

Your Response :

Correct Answer : b

Exp: B) Zygomatic complex

Zygomaticomaxillary complex fractures (which are also known as a


tripod, tetrapod or quadripod fracture, trimalar fracture or malar fracture)
are seen in the setting of traumatic injury to the face.

On radiographic evaluation, typically with dedicated CT imaging with


multiplanar reformats, the following three fracture components are
generally identified

(Q.73) Which of the below is an example of affective learning?

(a) Measuring pulse rate

(b) Enumerating causes of obesity

(c) Motivating a person for blood donation

(d) Arriving at differential diagnosis


Your Response :

Correct Answer : c

Exp: Ans C) Motivating a person for blood donation

Learning domains, sometimes referred to as categories of learning


outcomes, are critical to consider as you plan your lessons. These domains
include cognitive, affective, psychomotor, and interpersonal.

Learning Activities Delivery Assessment


Domain Considerations

Cognitive Self-check Web-enhanced Project based for higher


quizzes materials cognitive skills
supplementing
Case studies classroom lectures Multiple choice or short
essay questions
Drill and Hybrid course with
practice cognitive content on Case Studies

Short answer the web


essay Multimedia
Project or simulations of
problem-based challenging and key
activities concepts

Affective Goal setting Face-to-face meetings Self-assessment using


check-list
Self-reflective Motivational videos
writing in a Pre/post attitude survey
journal Streaming audio related to course content
explanations and
Practice encouragement Retention/success in
tutorials course
designed for Interactive video, web
student success casts, conference calls

Psycho- Practice of Face-to-face Performance of skill


motor desired skill demonstrations matches set standard as
with feedback observed by an
Demonstration videos instructor or designee
Arranging
sequences of Pictures with audio
an activity in and text explanations
correct order
Interactive video
demonstrations

Interper- Structured Face-to-face small Team, instructor and


sonal team projects group coaching and self assessment
with debriefing feedback sessions measures

Analyzing Check lists, examples, Analysis of video taped


video models videos and other student performance of
and identifying cognitive support desired interpersonal
correct from material presented skill
incorrect online
performance

(Q.74) In which phase of demographic cycle is India now?

(a) High stationary

(b) Early expanding

(c) Late expanding

(d) Low stationary

Your Response :

Correct Answer : c

Exp: Ans C) Late expanding

Demographic transition model is a model used to represent the process of


explaining the transformation of countries from high birth rates and high
death rates to low birth rates and low death rates as part of the
economic development of a country from a pre-industrial to an
industrialized economy. It is based on an interpretation begun in 1929 by
the American demographer Warren Thompson of prior observed changes,
or transitions, in birth and death rates in industrialized societies over
the past two hundred years.

Stage one ( high stationary)

In stage one, pre-industrial society, death rates and birth rates are high
and roughly in balance.

the birth rate is constant, while the death rate fluctuates due to manmade
and natural disasters as famines, floods and wars.

India was in this phase till 1920

Stage two ( early expanding )

In stage two, that of a developing country, the death rates drop rapidly
due to improvements in food supply and sanitation, which increase life
spans and reduce disease. These changes usually come about due to
improvements in farming techniques, access to technology, basic
healthcare, and education. Without a corresponding fall in birth rates this
produces an imbalance, and the countries in this stage experience a large
increase in population.

Many countries in South Asia and Africa are in this stage

Stage three ( late expanding)

In stage three, birth rates fall due to access to contraception, increases in


wages, urbanization, a reduction in subsistence agriculture, an increase in
the status and education of women, a reduction in the value of children's
work, an increase in parental investment in the education of children and
other social changes. Population growth begins to level off.

India is currently in this stage

Stage four ( low stationary)

During stage four there are both low birth rates and low death rates.

As the large group born during stage two ages, it creates an economic
burden on the shrinking working population. Death rates may remain
consistently low or increase slightly due to increases in lifestyle diseases
due to low exercise levels and high obesity and an aging population
in developed countries.

Birth rates fluctuate, indicative of fertility control as people alter their


reproduction according to socioeconomic changes.

Stage five (declining)

The population begins to decline due to lower birth rate as compared to


death rate

some of east European countries like Germany and Hungary are in this
phase

England was the first country to pass through the demographic transition.
This took approximately 200 years. Some other countries, such as Japan,
which started the process rather later than England, completed their
passage through the transition in less than half that time.

(Q.75) In patient with Malaria, if fever has periodicity of 72 hours, which one of the
following is likely to be causative agent ?

(a) P. falciparum

(b) P. vivax
(c) P. ovale

(d) P. malariae

Your Response :

Correct Answer : d

Exp: Ans D) P. malariae

The incubation period for malaria is around 7-30 day. There is a brief
prodromal period with symptoms of fever, headache, and myalgia.
Symptoms begin with a cold stage (a shaking chill), following by a fever
stage (4041C) that lasts about 24 hours, and finally a wet stage. The wet
stage occurs several hours after the fever, when the body temperature
drops quickly to normal and profuse sweating begins. The patient is
exhausted but well until the next cycle of fever begins. Other symptoms
include splenomegaly and anemia.

Three basic types of malaria

(Q.76) In clinical trials one can take care of the effects of unknown confounders by

(a) Matching cases and controls

(b) Randomization of study subjects

(c) Proper selection of cases and controls

(d) Properly measuring exposure and outcome

Your Response :

Correct Answer : b

Exp: Ans B) Randomization of study subjects

Research studies typically fall into one of two main categories:

Observational studies

Here researchers observe the effect of a risk factor, diagnostic test or


treatment without trying to influence what happens. Such studies are
usually "retrospective" the data are based on events that have already
happened. Most workplace health research falls into this category.

Cohort study:For research purposes, a cohort is any group of people who


are linked in some way and followed over time. Researchers observe what
happens to one group that's been exposed to a particular variable for
example, the effect of company downsizing on the health of office
workers. This group is then compared to a similar group that hasn't been
exposed to the variable.

Case control study: Here researchers use existing records to identify


people with a certain health problem (cases) and a similar group
without the problem (controls). Example: To learn whether a certain
drug causes birth defects, one might collect data about children with
defects (cases) and about those without defects (controls). The data are
compared to see whether cases are more likely than controls to have
mothers who took the drug during pregnancy.

Some strengths of observational studies

This may be the only way researchers can explore certain questions. For
example, it would be unethical to design a randomized controlled trial
deliberately exposing workers to a potentially harmful situation.

But...

The results of observational studies are, by their nature, open to dispute.


Example: A cohort study might find that people who meditated regularly
were less prone to heart disease than those who didn't. But the link may be
explained by the fact that people who meditate also exercise more and
follow healthier diets.

Experimental studies

Here researchers introduce an intervention and study the effects.


Experimental studies are usually randomized, meaning the subjects are
grouped by chance. While not all controlled studies are randomized, all
randomized trials are controlled.

Randomized Controlled Trial (RCT)

Eligible people are randomly assigned to two or more groups. One group
receives the intervention (such as a new drug) while the control group
receives nothing or an inactive placebo. The researchers then study what
happens to people in each group. Any difference in outcomes can then be
linked to the intervention.

Controlled Clinical Trial (CCT)

This is similar to an RCT, except that subjects are not randomly assigned
to the treatment or control groups. This increases the chance for bias
that is, that people with similar qualities ended up in each of the groups
which could influence the final results.

Some strengths of experimental studies

The RCT is still considered the gold standard for producing reliable
evidence because little is left to chance.

But...

There's a growing realization that such research is not perfect, and that
many questions simply can't be studied using this approach. Such research
is time-consuming and expensive it may take years before results are
available.

(Q.77) If a disease has three times more incidence in females as compared to males and same
prevalence in both males and females, true statement will be

(a) It is highly fatal in females

(b) More survival in females

(c) Better prognosis in males

(d) Less fatal in males

Your Response :

Correct Answer : a

Exp: Ans A) It is highly fatal in females

Incidence

Incidence is the rate of new (or newly diagnosed) cases of the disease. It
is generally reported as the number of new cases occurring within a period
of time (e.g., per month, per year). It is more meaningful when the
incidence rate is reported as a fraction of the population at risk of
developing the disease (e.g., per 100,000 or per million population).
Obviously, the accuracy of incidence data depends upon the accuracy of
diagnosis and reporting of the disease. In some cases (including ESRD) it
may be more appropriate to report the rate of treatment of new cases since
these are known, whereas the actual incidence of untreated cases is not.

Incidence rates can be further categorized according to different subsets of


the population e.g., by gender, by racial origin, by age group or by
diagnostic category.

Prevalence

Prevalence is the actual number of cases alive, with the disease either
during a period of time (period prevalence) or at a particular date in time
(point prevalence). Period prevalence provides the better measure of the
disease load since it includes all new cases and all deaths between two
dates, whereas point prevalence only counts those alive on a particular
date.

Prevalence is also most meaningfully reported as the number of cases as a


fraction of the total population at risk and can be further categorized
according to different subsets of the population.

Incidence to Prevalence

The relationship between incidence and prevalence depends greatly on the


natural history of the disease state being reported. In the case of an
influenza epidemic, the incidence may be high but not contribute to much
growth of prevalence because of the high, spontaneous rate of disease
resolution. In the case of a disease that has a low (or zero) cure rate, but
where maintenance treatment permits sustained survival, then incidence
contributes to continuous growth of prevalence. In such cases, the
limitation on prevalence growth is the mortality which occurs in the
population. Obviously, prevalence will continue to grow until mortality
equals or exceeds the incidence rate.

(Q.78) Prophylaxis with vitamin C is helpful in preventing

(a) Fluorosis

(b) Neuro Lathyrism

(c) Iodine deficiency

(d) Botulism

Your Response :

Correct Answer : b

Exp: Ans B) Neuro Lathyrism

Vitamin C has a protective role against neurolathyrism

Neurolathyrism is a disease that has crippled many thousands in the


Indian subcontinent and elsewhere during the past several centuries. It
struck poor people who used the seeds of Lathyrus sativus as the principal
item in their diet continuously for a period of at least two to three months.
The disease appeared to affect humans almost exclusively. No
experimental animal model was available to facilitate biomedical research
on the subject until recently, when we produced the condition in guinea
pigs. When guinea pigs having a subacute deficiency of L-ascorbic acid
were fed for four to five weeks on a diet of cooked L. sativus seeds
supplemented with minerals and the usual vitamins except for L-ascorbic
acid, they developed typical symptoms of neurolathyrism, affecting the
hind limbs.
The same condition also appeared within two to three hours following
intraperitoneal administration of an extract of L. sativus in guinea pigs
previously made deficient in ascorbic acid by omitting it from a normal
diet that did not include any L. sativus seeds. We further found that L-
ascorbic acid not only protected the animals from the neurotoxic effect of
L. sativus, but also had a curative effect on those that had earlier become
paralysed by the toxicity of the seeds.

We subsequently produced the same symptoms of neurolathyrism in


monkeys made deficient in ascorbic acid by feeding them cooked L.
sativus seeds or by administering extracts of L sativus intraperitoneally.
Thus, a subacute deficiency of L-ascorbic acid that was not severe enough
to produce scorbutic symptoms was found to be a precondition for the
appearance of neurolathyrism attributed to L. sativus seeds. This also
explained the failure to produce experimental neurolathyrism in
laboratory animals that do not require any dietary supply of L-ascorbic
acid or that were provided this vitamin in the experimental diet before we
suspected that its deficiency could be a predisposing factor.

(Q.79) Which of the following is NOT a source of infection inplague ?

(a) Case of bubonic plague

(b) Case of pneumonic plague

(c) Infected rodents

(d) Infected fleas

Your Response :

Correct Answer : a

Exp: Ans A) Case of bubonic plague

There three most common forms of plague are:

Bubonic plague, an infection of the lymph nodes

Pneumonic plague, an infection of the lungs

Septicemic plague, an infection of the blood

The time between being infected and developing symptoms is typically 2


to 8 days. But the time can be as short as 1 day for pneumonic plague.

Risk factors for plague include a recent flea bite and exposure to rodents,
especially rabbits, squirrels, or prairie dogs, or scratches or bites from
infected domestic cats.
Humans bitten by an infected flea usually develop a bubonic form of
plague, which is characterized by a bubo, i.e. a swelling of the lymph
node draining the flea bite site.

If the bacteria reach the lungs, the patient develops pneumonia


(pneumonic plague), which is then transmissible from person to person
through infected droplets spread by coughing.

(Q.80) The base of the Food Guide Pyramid is formed by

(a) Cereals and pulses

(b) Fruits and vegetables

(c) Milk and milk products

(d) Meat, poultry and fish

Your Response :

Correct Answer : a

Exp: Ans A) Cereals and pulses

(Q.81) Which of the following antitubercular drugs can be used in presence of renal disease?

(a) isoniazid

(b) Rifampicin
(c) Pyrazinamide

(d) All

Your Response :

Correct Answer : d

Exp: Ans D) All

In the presence of renal disease isoniazide(H) , Rifampicin (R),


Pyrazinamide (Z) are relatively safe for administration, while
Streptomycin(S) and Ethambutol (E)are avoided.

In liver disease, the reverse procedure is followed i.e. S and E are


considered

to be safe, whereas H, R and Z should be stopped.

(Q.82) The overall burden of diseases is best detected by

(a) Sullivans Index

(b) Quality adjusted life years (QALY)

(c) Human Developmental Index

(d) Disability adjusted life years (DALY)

Your Response :

Correct Answer : d

Exp: Ans D) Disability adjusted life years (DALY)

DALYs for a disease or health condition are calculated as the sum of the
Years of Life Lost (YLL) due to premature mortality in the population
and the Years Lost due to Disability (YLD) for people living with the
health condition or its consequences.

It is used for quantifying the Burden of Disease from mortality and


morbidity.

The Sullivan method or Sullivan's method is a very simple method to


compute health expectancies. It combines data issued from a regular
period of life table on the one hand and from cross-sectional survey
providing prevalence of given health dimension on the other hand, for
instance disability. The age-specific prevalence is directly applied to the
person-year of the life table: it provides the total number of years spent
with disability, the total number of years lived without disability, and
summing both, the total number of years lived.

The main advantage of the Sullivan's method lies in the separate


collection of mortality and disability data and in the ready availability of
the data necessary for the calculation. Basic cross-sectional surveys are
sufficient to collect the observed prevalence of disability within the
population. However the problem with this method lies in approximating
the period prevalence by the observed prevalence of disability

Health-adjusted life expectancy (HALE) is a measurement developed by


the World Health Organization that attempts to capture a more complete
estimate of health than standard life expectancy rates (note that HALE
was previously referred to as Disability-Adjusted Life Expectancy or
DALE). HALE estimates the number of healthy years an individual is
expected to live at birth by subtracting the years of ill health weighted
according to severity from overall life expectancy . HALE is also
calculated at age 65 to provide a measurement of the quality of life of
seniors. By moving beyond mortality data, HALE is meant to measure not
just how long people live, but the quality of their health through their
lives.

A quality-adjusted life-year (QALY) takes into account both the quantity


and quality of life generated by healthcare interventions. It is the
arithmetic product of life expectancy and a measure of the quality of the
remaining life-years. A QALY places a weight on time in different health
states. A year of perfect health is worth 1 and a year of less than perfect
health is worth less than 1. Death is considered to be equivalent to 0;
however, some health states may be considered worse than death and have
negative scores.

(Q.83) All the following features contribute to High Risk Babies EXCEPT

(a) Babies of working mother

(b) Birth order of 4 or more

(c) Artificial feeding

(d) Weight less than 70% of the expected weight

Your Response :

Correct Answer : b

Exp: Ans B) Birth order of 4 or more

The following are some factors that can place a baby at high risk and
increase the chances of being admitted to the NICU.
maternal factors:

age younger than 16 or older than 40 years

drug or alcohol exposure

diabetes

hypertension (high blood pressure)

bleeding

sexually transmitted diseases

multiple pregnancy (twins, triplets, or more)

too little or too much amniotic fluid

premature rupture of membranes (also called the amniotic sac or bag of


waters)

single working mother

delivery factors:

fetal distress/birth asphyxia (changes in organ systems due to lack of


oxygen)

breech delivery presentation (buttocks delivered first) or other abnormal


presentation

meconium (the baby's first stool passed during pregnancy into the
amniotic fluid)

nuchal cord (cord around the baby's neck)

forceps or cesarean delivery

baby factors:

birth at gestational age less than 37 weeks or more than 42 weeks

birthweight less than 2,500 grams (5 pounds, 8 ounces) or over 4,000


grams (8 pounds, 13 ounces)

small for gestational age

medication or resuscitation in the delivery room

birth defects

respiratory distress including rapid breathing, grunting, or apnea (stopping


breathing)
infection such as herpes, group B streptococcus, chlamydia

seizures

hypoglycemia (low blood sugar)

need for extra oxygen or monitoring, intravenous (IV) therapy, or


medications

need for special treatment or procedures such as a blood transfusion

(Q.84) Acculturation is defined as

(a) Loss of cultural values

(b) Opposition of two cultures on contact

(c) Fading away of a culture

(d) Diffusion of two cultures on both ways

Your Response :

Correct Answer : d

Exp: Ans D) Diffusion of two cultures on both ways

Acculturation is a process in which members of one cultural group adopt


the beliefs and behaviors of another group. Although acculturation is
usually in the direction of a minority group adopting habits and language
patterns of the dominant group, acculturation can be reciprocal--that is,
the dominant group also adopts patterns typical of the minority group.
Assimilation of one cultural group into another may be evidenced by
changes in language preference, adoption of common attitudes and values,
members hip in common social groups and institutions, and loss of
separate political or ethnic identification.

(Q.85) Which of the following is NOT included as fundamental aspects of disaster


management ?

(a) Disaster prevention

(b) Disaster Response

(c) Disaster preparedness

(d) Disaster mitigation

Your Response :
Correct Answer : a

Exp: Ans A) Disaster prevention

Fundamental Aspects of Disaster Management

(Q.86) The difference in incidence rates of disease (or deaths) between an exposed group
and non exposed group is known as

(a) Relative Risk

(b) Attributable Risk

(c) PopulationAttributable Risk

(d) Odds Ratio

Your Response :

Correct Answer : b

Exp: Ans B)Attributable Risk

In epidemiology, attributable risk is the difference in rate of a condition


between an exposed population and an unexposed population.
Attributable risk is mostly calculated in cohort studies, where individuals
are assembled on exposure status and followed over a period of time.

The Population Attributable Risk (or Population Attributable Fraction)


indicates the number (or proportion) of cases that would not occur in a
population if the factor were eliminated (e.g. how many lives would be
saved if people no longer smoked?)

Another term for the relative risk is the risk ratio because it is the ratio of
the risk in the exposed divided by the risk in the unexposed.

An odds ratio (OR) is a measure of association between an exposure and


an outcome

(Q.87) Outbreaks of leptospirosis are usually expected after

(a) Earthquakes

(b) Floods

(c) Mud slides

(d) Avalanche

Your Response :
Correct Answer : b

Exp: Ans B) Floods

The cause of leptospirosis is bacteria, Leptospira interrogans, a Gram-


negative spirochete (spiral-shaped bacteria). The bacteria infect many
types of animals (many wild animals, rodents, dogs, cats, pigs, horses,
cattle, for example) that subsequently contaminate water, soil, and crops
when they urinate because the bacteria are present in urine.

(Q.88) In Chronic non-communicable diseases which of the following is least important?

(a) Specificity of the association

(b) Sensitivity of the association

(c) Temporal association

(d) Dose response relationship

Your Response :

Correct Answer : b

Exp: B Sensitivity of the association (Ref. Clinical Epidemiology by


Rletcher 3rd ed. - 242)

CAUSAL ASSOCIATION:

a. Temporal relationship:

Causal attribute must precede disease or unfavourable out come

Fulfillment of this criterion is not easy as starting point and magnitude of


exposure cannot be established.

b. Consistency: Occurrence of association repeatedly at some time and


place or other.

E.g. we now accept casual association between smoking and Ca lung

c. Dose response relationship: : Higher exposure to the cause should


have higher possibility of occurrence of disease.

E.g. Occupational exposure to higher noise level and hearing loss.

Higher prevalence of Ca Lung in heavy smokers than in non or light


smokers

d. Strength of association: Can be known by calculating relative,


attributable risk.
e. Reversibility: Removal of suspected cause should decrease risk of
unfavourable out come

E.g. Risk of Ca Lung in Ex-smokers, than in continuing smoking.

f. Plausibility: Association must be consistent with other knowledge


(Mechanism of action, etc)

g. Specificity of the association

(Q.89) By giving equal time for interviewing the control and the cases, which of the
following bias is avoided?

(a) Selection bias

(b) Berkesonian bias

(c) Interviewer bias

(d) Confounding bias

Your Response :

Correct Answer : c

Exp: C. Interviewer bias. Ref : Park 20th Ed Pg 70.

Interviewers Bias : Bias may occur when the interviewer knows the
hypothesis and also know who the cases are. This prior information may
lead him to question the cases more thouroghly than tha controls
regarding a positive history of the suspected causal factor. A useful check
on this kind of bias can be made by noting the length of time taken to
interview the average case and the average control. This kind of bias can
be eliminated by double blinding.

Confounding Memory Selection bias Berkesonian Interviewer


bias bias bias bias

Removed by - Best controlled - Double


matching by prevention blinding

Seen in case Patients Cases Due to different Interviwer


control study with past controlled may rates of knows
history of not be admission to hypothesis
disease representative hospital for and cases
recall better of general different
than population purposes
normal

(Q.90) All are true aboutKwashiorkor except ?


(a) Child has a well nourished appearance

(b) Develops in few months to years

(c) Edema is a feature

(d) Has a high mortality.

Your Response :

Correct Answer : b

Exp: Ans B.Develops in few months to years.

Comparison of Marasmus and Kwashiorkor

Marasmus Kwashiorkor

Clinical Energy intake Protein intake during stress


setting state

Time course to Months or years Weeks


develop

Clinical Starved appearance Well-nourished appearance


features

Weight <80% standard for Easy hair pluckabilityb


height

Triceps skinfold <3 mm Edema

Mid-arm muscle
circumference <15 cm

Laboratory Creatinine-height index Serum albumin <2.8 g/dL


findings <60% standard

Total iron-binding capacity


<200 g/dL

Lymphocytes <1500/L

Anergy

Clinical Reasonably preserved Infections


course responsiveness to short-
term stress
Poor wound healing,
decubitus ulcers, skin
breakdown

Mortality Low unless related to High


underlying disease

Diagnostic Triceps skinfold <3 mm Serum albumin <2.8 g/dL


criteria

Mid-arm muscle At least one of the


circumference <15 cm following:

Poor wound healing,


decubitus ulcers, or skin
breakdown

Easy hair pluckabilityb

Edema

(Q.91) Cobblestone appearance of the conjunctiva is seen in

(a) Spring catarrh

(b) Angular conjunctivitis

(c) Eczematous conjunctivitis

(d) Trachoma

Your Response :

Correct Answer : a

Exp: Spring catarrh

Aetiology :: Uncommon allergic disorder of children. Complex immune


response with raised IgE levels in the tears and serum, and mast cells and
eosinophils in the conjunctival epithelium

Predisposing factors :: Onset usually before 10 years of age; M >>F,


Seasonal exacerbations (hence name) but condition may be year-round. if
severe Patients usually atopic with a history of eczema and asthma. Often
a family history of atopic disease

Symptoms:: Ocular itching, Watering, Mucoid stringy discharge, Blurred


vision, Photophobia, Difficulty opening eyes on waking, the severity of
symptoms is often asymmetrical

Signs:: Stringy white mucous exudate

Palpebral, limbal and corneal manifestations:

Palpebral:: Hyperaemia, oedema (chemosis) and cellular infiltration of


conjunctiva when active Giant papillary hypertrophy (papillae 1mm or
greater in diameter) of upper tarsus (cobblestone appearance)

Limbal:: Hyperaemic, oedematous, thickened limbus Trantas Dots


(discrete white superficial accumulations of eosinophils and degenerating
epithelial cells)

Corneal:: (usually in upper third), Punctate epithelial keratopathy, Macro-


erosion (coalescent epithelial loss), Plaque (deposited on Bowmans layer,
preventing re-epithelialisation), Subepithelial scarring (often ring-shaped)

(Q.92) Distant vision is measured at a distance of (Metres)

(a) 1

(b) 2

(c) 3

(d) 6

Your Response :

Correct Answer : d

Exp: D) 6 metres

Visual acuity is a measure of the spatial resolution of the visual


processing system. VA is tested by requiring the person whose vision is
being tested to identify so-called optotypes stylized letters, Landolt
rings, Lea symbols, or other patterns on a printed chart (or some other
means) from a set viewing distance. Optotypes are represented as black
symbols against a white background (i.e. at maximum contrast). The
distance between the person's eyes and the testing chart is set at a
sufficient distance to approximate infinity in the way the lens attempts to
focus (far acuity), or at a defined reading distance (near acuity).

Normal visual acuity is commonly referred to as 20/20 vision (even


though acuity in normally sighted people is generally higher), the metric
equivalent of which is 6/6 vision. At 20 feet or 6 meters, a human eye
with nominal performance is able to separate contours that are
approximately 1.75 mm apart.A vision of 20/40 corresponds to lower than
nominal performance, a vision of 20/10 to better performance.

(Q.9 Relative afferent pupillary defect is pathognomonic of


3)
(a) Optic neuritis

(b) Retinal Holes

(c) Papilledema

(d) Macular Edema

Your Response :

Correct Answer : a

Ex A) Optic neuritis
p:
The swinging light test is used to detect a relative afferent pupil defect (RAPD): a means
of detecting differences between the two eyes in how they respond to a light shone in one
eye at a time.

CONDITIONS LEADING TO A RELATIVE AFFERENT PUPILLARY DEFECT :


Optic nerve disorders:

Unilateral optic neuropathies are common causes of an RAPD. If a condition is bilaterally


symmetrical, there will not be an RAPD.

Optic neuritis - Even very mild optic neuritis with a minimal loss of vision can lead to a
very strong RAPD.

Ischemic optic neuropathies

Glaucoma

Traumatic optic

Optic nerve tumor

Orbital disease

Radiation optic nerve damage

Miscellaneous optic neuropathies, such as Leber's optic neuropathy (usually eventually


bilateral) and other inheritable optic neuropathies.

Optic nerve infections or inflammations

Optic atrophy status-post papilledema - This is usually bilateral.

Surgical damage to the optic nerve.


Retinal Causes of a Relative Afferent Pupillary Defect

Again, symmetrically bilateral retinal disease will not show an RAPD. Usually retinal
disease has to be quite severe for an RAPD to be clinically evident.

Ischemic retinal disease.

Ischemic ocular disease (Ocular ischemic syndrome)

Retinal detachment

Severe macular degeneration

Intraocular tumor

Retinal infection

Other Causes of a Relative Afferent Pupillary Defect

Amblyopia, if severe, can lead to a relative afferent pupillary defect. Usually the visual
acuity would be 20/400, or worse.

Cerebral vascular disease - Usually, it is an optic nerve disorder that leads to an RAPD,
rather than an optic tract or visual cortex disorder.

CONDITIONS WHICH WILL NOT CAUSE A RELATIVE AFFERENT PUPILLARY


DEFECT INCLUDE:

Refractive Error

Media Opacity (a bright enough light will indicate NO RAPD)

- Cataract (even if completely opaque) - Corneal scar - Hyphema - Vitreous


hemorrhage

Previous eye surgery (unless there is a complication, previous disease, or a new problem)

Strabismus

Conditions with anEfferentPupillary Defect

- Third Cranial Nerve Palsy - Adie's Pupil - Horner's Syndrome

Mild retinal problems, including:

- Mild background diabetic retinopathy - Central serous choroidopathy

- Non-ischemic vein occlusions - Mild macular degeneration

Conditions which are typically bilaterally symmetrical will not show an RAPD:

Bilateral retinitis pigmentosa

Bilateral nutritional or metabolic optic neuropathies


Cerebral infarct usually will not cause an RAPD

(Q.94) Foster Fuchs spots are seen in

(a) Hypermetropia

(b) Iridocyclitis

(c) Corneal ulcer

(d) Myopia

Your Response :

Correct Answer : d

Exp: D) Myopia

Fuchs' Spots are also known as:

Forster-Fuchs Spots,

Fuchs' retinal spots,

Disciform degeneration (in myopia), and

Choroidal neovascularisation (in myopia)

Fuch's Spots are initially caused by tiny breaks in the Bruch's membrane
(which is the transparent innermost layer of the choroid of the eye).

These tiny breaks lead to the appearance of pigmented lesions in the


macular part of theretina.

These pigmented scars, called Fuchs' Spots, are the result of


neovascularization (that is, the abnormal formation of new, fragile, blood
vessels) in the choroid.

Fuch's Spots generally, though not necessarily, occur in severely short-


sighted (myopic) individuals.

Typical features of degenerative myopia demonstrating:

Peripapillary atrophy

Tilted disc

Tesselated or tigroid fundus

Multiple areas of pigment epithelium and choriocapillaris atrophy


involving the macular area
Lacquer cracks

Pigment epithelium proliferation produces Foerster-Fuchs' spots

(Q.95) Satellite lesions are seen in

(a) Mycotic corneal ulcer

(b) Bacterial corneal ulcer

(c) Viral corneal ulcer

(d) Protozoal keratitis

Your Response :

Correct Answer : a

Exp: A) Mycotic corneal ulcer

Bacterial keratitis is caused by Staphylococcus aureus, Streptococcus


viridans, Escherichia coli, Enterococci, Pseudomonas, Nocardia, N.
Gonorrhoea and many other bacteria.

Fungal keratitis causes deep and severe corneal ulcer. It is caused


by Aspergillus sp., Fusarium sp., Candida sp., as also Rhizopus, Mucor,
and other fungi. The typical feature of fungal keratitis is slow onset and
gradual progression, where signs are much more than the symptoms.
Small satellite lesions around the ulcer are a common feature of fungal
keratitis and hypopyon is usually seen.

Viral keratitis causes corneal ulceration. It is caused most commonly


by Herpes simplex, Herpes Zoster and Adenoviruses. Also it can be
caused by coronaviruses & many other viruses. Herpes virus cause
a dendritic ulcer, which can recur and relapse over the lifetime of an
individual.

Protozoa infection like Acanthamoeba keratitis is characterized by severe


pain and is associated with contact lens users swimming in pools.

(Q.96) In absence of clinical evidence of thyroid disease, thyroid function tests showing low
TSH, and normal or low T3 and T4 indicates

(a) Subclinical thyrotoxicosis

(b) subclinical hypothyroidism

(c) Sick euthyroidism


(d) transient thyroiditis

Your Response :

Correct Answer : c

Exp: Ans C) sick euthyroidism

Euthyroid sick syndrome is low serum levels of thyroid hormones in


clinically euthyroid patients with nonthyroidal systemic illness. Diagnosis
is based on excluding hypothyroidism. Treatment is of the underlying
illness; thyroid hormone replacement is not indicated. Patients with
various acute or chronic nonthyroid disorders may have abnormal thyroid
function test results. Such disorders include acute and chronic illness,
particularly fasting, starvation, protein-energy undernutrition, major
trauma, MI, chronic renal failure, diabetic ketoacidosis, anorexia nervosa,
cirrhosis, thermal injury, and sepsis.

(Q.97) Co-infection with which virus leads to change in antiretroviral therapy in people
living with HIV ?

(a) Cytomegalo virus

(b) Varicella virus

(c) Hepatitis B virus

(d) JC virus

Your Response :

Correct Answer : c

Exp: Ans C) Hepatitis B virus

Prior to initiation of antiretroviral therapy (ART), all patients who test


positive for hepatitis B surface antigen (HBsAg) should be tested for
hepatitis B virus (HBV) DNA using a quantitative assay to determine the
level of HBV replication

Because emtricitabine (FTC), lamivudine (3TC), and tenofovir (TDF)


have activity against both HIV and HBV, if HBV or HIV treatment is
needed, ART should be initiated with the combination of TDF + FTC or
TDF + 3TC as the nucleoside reverse transcriptase inhibitor (NRTI)
backbone of a fully suppressive antiretroviral (ARV) regimen

If HBV treatment is needed and TDF cannot safely be used, the


alternative recommended HBV therapy is entecavir in addition to a fully
suppressive ARV regimen . Other HBV treatment regimens include
peginterferon alfa monotherapy or adefovir in combination with 3TC or
FTC or telbivudine in addition to a fully suppressive ARV regimen .

Entecavir has activity against HIV; its use for HBV treatment without
ART in patients with dual infection may result in the selection of the
M184V mutation that confers HIV resistance to 3TC and FTC. Therefore,
entecavir must be used in addition to a fully suppressive ARV regimen
when used in HIV/HBV-coinfected patients .

Discontinuation of agents with anti-HBV activity may cause serious


hepatocellular damage resulting from reactivation of HBV; patients
should be advised against self-discontinuation and carefully monitored
during interruptions in HBV treatment .

If ART needs to be modified due to HIV virologic failure and the patient
has adequate HBV suppression, the ARV drugs active against HBV
should be continued for HBV treatment in combination with other suitable
ARV agents to achieve HIV suppression

(Q.98) Periodic, deep, retroorbital, excruciating, nonfluctuant and explosive headache with
ipsilateral lacrimation or rhinorrhoea is characteristic of

(a) Cluster headache

(b) Migraine headache

(c) Hypnic headache

(d) Tension headache

Your Response :

Correct Answer : a

Exp: Ans A) cluster headache

Cluster headache (CH) is a neurological disorder characterized by


recurrent, severe headaches on one side of the head, typically around the
eye. There are often accompanying autonomic symptoms during the
headache such as eye watering, nasal congestion and swelling around the
eye, typically confined to the side of the head with the pain.

Cluster headache belongs to a group of primary headache disorders,


classified as the trigeminal autonomic cephalalgias or (TACs). Cluster
Headache is named after the demonstrated grouping of headache attacks
occurring together (cluster). Individuals typically experience repeated
attacks of excruciatingly severe unilateral headache pain. Cluster
Headache attacks often occur periodically; spontaneous remissions may
interrupt active periods of pain, though about 1015% of chronic CH
never remit. The cause of Cluster Headache has not been identified.
(Q.99) The study of alteration in chromatin and histone proteins and methylation of DNA
sequence that influence gene expression is called as

(a) Proteomics

(b) Metagenomics

(c) Epigenomics

(d) Metabolomics

Your Response :

Correct Answer : c

Exp: Ans C) Epigenomics

Epigenomics is the study of the complete set of epigenetic modifications


on the genetic material of a cell, known as the epigenome. Epigenetic
modifications are reversible modifications on a cells DNA or histones
that affect gene expression without altering the DNA sequence .Two of
the most characterized epigenetic modifications are DNA methylation and
histone modification.

Proteomics is the large-scale study of proteins, particularly their structures


and functions.

Metagenomics is the study of genetic material recovered directly from


environmental samples.

Metabolomics is the scientific study of chemical processes involving


metabolites.

(Q.100) The circulating neutrophils are usually mature and not clonally derived in

(a) Leucocytosis

(b) Neutropenia

(c) Leukemoid reactions

(d) Neutrophilia

Your Response :

Correct Answer : c

Exp: Ans C) Leukemoid reactions

Persistent neutrophilia with WBC count of 30000-50000 / microliter or


more is called leukemoid reaction. The circulating neutrophils are
usually mature and not clonally derived in The circulating neutrophils are
usually mature and not clonally derived in leukemoid reaction.

(Q.101) Omalizumab is a blocking antibody that neutralises circulating

(a) IgE

(b) Leukotrines

(c) Cromones

(d) TNF and antibodies

Your Response :

Correct Answer : a

Exp: Ans A) IgE

Omalizumab is a recombinant DNA-derived humanized IgG1k


monoclonal antibody that specifically binds to free human
immunoglobulin E (IgE) in the blood and interstitial fluid and to
membrane-bound form of IgE (mIgE) on the surface of mIgE-expressing
B lymphocytes.

(Q.102) Which is not a potential complications of gene therapy

(a) Genotoxicity

(b) Genome integration

(c) Gene silencing

(d) Immunotoxicity

Your Response :

Correct Answer : b

Exp: Ans B) Genome integration

Despite a strong record of success in small and large animal models, a


range of problems has limited the scope of successful application of gene
therapy in hum Clinical studies over the past decade have identified
several major obstacles to gene therapy, as listed below. Typically, for
each specific combination of vector, transgene and target tissue, one or
two of these problems predominate as the major clinical obstacle(s).

Gene silencing: Gene silencing can result, for example, from promoter
methylation leading to loss of therapeutic transgene expression.

Insertional mutagenesis:This results from the random integration of the


therapeutic vector genome within the host DNA.

Phenotoxicity: Phenotoxicity can result from the overexpression or


ectopic expression of the donated gene.

Immunotoxicity: This refers to harmful immune responses to either the


vector or the transgene product.

Horizontal transmission of the donated DNA: The risk of vector


shedding after in vivo gene transfer is proportional to the time since
administration and to the vector dose administered.

Vertical transmission: This refers to inadvertent germline transmission of


the donated DNA

Many viruses are able to insert their genetic material into host
chromosomes. Three kind of integration exist:

- Mandatory integration: viruses for which this is an obligatory event


during viral replication.

- Occasional integration: Viral genome can be integrated under certain


conditions. This occasional integration can confer an advantage to the
virus, for example for HHV-6 or AAV-2. It can also provide an
advantage to the host cell,often leading to oncogenesis in multicellular
organism, for example with HBV or HPV-16 or 18.

- Rare integration: Viruses integrated long time ago into a host genome,
but through a very rare and presumably accidental process.

(Q.103) The most common site for Amebiasis among following is

(a) Sigmoid colon

(b) Transverse colon

(c) Cecum

(d) Hepatic flexure

Your Response :

Correct Answer : c

Exp: Ans C) Cecum

The most common site for Amebiasis is caecum.


Amoebiasis is usually a ''medical' disease, but it does have some surgical
complications, ranging from the very acute to the very chronic, which
you should be able to treat. They usually involve a patient's gut, but they
can involve his liver, or occasionally his lungs, or even his skin.
Amoebiasis is more common in older patients, but no age is immune,
and amoebae may invade the gut of babies. It is less often seen in
women, but in pregnancy it can be fulminating.

Entamoeba histolytica normally lives harmlessly in the colon, but


trophozoites occasionally invade its mucosa to cause shallow discrete
circular or oval ulcers, with yellow sloughs in their bases, and sometimes
red edges. These ulcers are most common in the caecum and ascending
colon, the sigmoid colon, and the rectum. They cause diarrhoea, with or
without blood, pus, and mucus.

The lesions in the gut are usually quite superficial, but, if a patient's
resistance is low, amoebae may invade it more deeply, especially if he is
diabetic, or alcoholic, or has recently been severely injured. Invasive
intestinal amoebiasis takes several acute forms: (1) Amoebae can cause
massive necrosis of the mucosa of his colon, so that large pieces of it
separate as casts, and are passed rectally. (2) They can invade its
muscular wall to cause gangrene, sloughing, and perforation[md]acute
necrotizing amoebic colitis. Bacterial infection may then spread as
generalized peritonitis, or it may remain localized as a pericolic abscess
which you can feel as a tender mass. He can also develop peritonitis,
without actual perforation, or his gut can perforate extraperitoneally. As
the result of this suppuration, it may obstruct, or he may develop ileus.
Occasionally, his colon bleeds severely.

If the pathological process is more chronic, he may have: (1) An


amoeboma; this is a diffuse, oedematous, hyperplastic granulomatous
swelling anywhere in his colon or rectum, which is often multiple, and
may be palpable, and may obstruct his gut (usually temporarily).
Although an amoeboma may form anywhere, a mass in the caecum is
more easily palpable. If you do feel a mass in a patient with amoebiasis,
it is more likely to be a paracolic abscess than an amoeboma. (2) A
fibrous postamoebic stricture, which is one of the end results of an
amoeboma. An amoeboma and a stricture are two stages in the same
process, and he may have a lesion with some of the features of both.
Both are common in some areas (Durban), and are the late, chronic
complications of amoebic colitis; they occur years after the initial bloody
diarrhoea, and are less serious than acute invasive amoebiasis. They
usually involve the rectum (where you can feel them rectally), the
sigmoid, and the descending colon, in that order. Both can: (1) cause
diarrhoea and other abdominal symptoms, and (2) obstruct the large gut,
usually incompletely.

(Q.104) Gitelmans syndrome is differentiated from Barters syndrome by


(a) Hypocalciuria and severe hypomagnesemia

(b) Hypercalciuria and severe hypomagnesemia

(c) Hypocalciuria and severe hypokalemia

(d) hypercalciuria and severe hyperkalemia

Your Response :

Correct Answer : a

Exp: A) Hypocalciuria and severe hypomagnesemia

Gitelman's syndrome, also known as familial hypokalaemic


hypomagnesaemia, is a rare autosomal recessive hereditary salt-losing
tubulopathy, characterised by hypokalaemic metabolic alkalosis,
hypomagnesaemia, and hypocalciuria, which is usually caused by mutations
in the SLC12A3 gene encoding the thiazide-sensitive sodium chloride
transporter.

Bartter's syndrome refers to a group of autosomal recessive disorders with


impaired salt reabsorption in the thick ascending loop of Henle with
pronounced salt wasting, hypokalaemic metabolic alkalosis, and
hypercalciuria.

Bartter's syndrome or Gitelman's syndrome can be differentiated by:

Hypokalaemic alkalosis with hypercalciuria (true Bartter's syndrome).

Hypokalaemic alkalosis with hypocalciuria (Gitelman's syndrome).

Some Differences Between Bartter Syndrome and Gitelman Syndrome

Feature Bartter Syndrome Gitelman


Syndrome

Location of kidney Ascending loop of Henle Distal tubule


defect (mimics effects of loop (mimics effects of
diuretics) thiazides)

Urinary Ca excretion Normal or increased, Decreased


commonly with
nephrocalcinosis

Serum Mg level Normal or decreased Decreased,


sometimes greatly

Renal prostaglandin Increased Normal


E2 production

Usual age at Before birth to early Late childhood to


presentation childhood, often with adulthood
intellectual disability and
growth disturbance

Neuromuscular Uncommon or mild Common


symptoms (eg, muscle
spasms, weakness)

(Q.105) Ipsilateraloculomotor palsy with contralateral ataxia is seen in

(a) Nothnagels syndrome

(b) Claude syndrome

(c) Benedicts syndrome

(d) Webers syndrome

Your Response :

Correct Answer : b

Exp: B. Claude syndrome

Lateral medullary Occlusion of any of five vessels may be


syndrome responsiblevertebral, posterior inferior cerebellar,
superior, middle, or inferior lateral medullary
arteries.

On side of lesion

Pain, numbness, impaired sensation over one-half


the face: Descending tract and nucleus fifth nerve

Ataxia of limbs, falling to side of lesion:


Uncertainrestiform body, cerebellar hemisphere,
cerebellar fibers, spinocerebellar tract (?)

Nystagmus, diplopia, oscillopsia, vertigo, nausea,


vomiting: Vestibular nucleus

Horner's syndrome (miosis, ptosis, decreased


sweating): Descending sympathetic tract

Dysphagia, hoarseness, paralysis of palate, paralysis


of vocal cord, diminished gag reflex: Issuing fibers
ninth and tenth nerves

Loss of taste: Nucleus and tractus solitarius

Numbness of ipsilateral arm, trunk, or leg: Cuneate


and gracile nuclei

Weakness of lower face: Genuflected upper motor


neuron fibers to ipsilateral facial nucleus

On side opposite lesion

Impaired pain and thermal sense over half the body,


sometimes face: Spinothalamic tract

Medial medullary Occlusion of vertebral artery or of branch of


syndrome vertebral or lower basilar artery.

On side of lesion

Paralysis with atrophy of one-half half the tongue:


Ipsilateral twelfth nerve

On side opposite lesion

Paralysis of arm and leg, sparing face; impaired


tactile and proprioceptive sense over one-half the
body: Contralateral pyramidal tract and medial
lemniscus

Total unilateral Occlusion of vertebral artery.


medullary
syndrome Combination of medial and lateral syndromes

Lateral Occlusion of vertebral artery.


pontomedullary
syndrome Combination of lateral medullary and lateral inferior
pontine syndrome

Basilar artery The syndrome of the lone vertebral artery is


equivalent. A combination of the various brainstem
syndrome syndromes plus those arising in the posterior
cerebral artery distribution.

Bilateral long tract signs (sensory and motor;


cerebellar and peripheral cranial nerve
abnormalities): Bilateral long tract; cerebellar and
peripheral cranial nerves

Paralysis or weakness of all extremities, plus all


bulbar musculature: Corticobulbar and
corticospinal tracts bilaterally

Medial inferior Occlusion of paramedian branch of basilar artery.


pontine syndrome
On side of lesion

Paralysis of conjugate gaze to side of lesion


(preservation of convergence): Center for conjugate
lateral gaze

Nystagmus: Vestibular nucleus

Ataxia of limbs and gait: Likely middle cerebellar


peduncle

Diplopia on lateral gaze: Abducens nerve

On side opposite lesion

Paralysis of face, arm, and leg: Corticobulbar and


corticospinal tract in lower pons

Impaired tactile and proprioceptive sense over one-


half of the body: Medial lemniscus

Lateral inferior Occlusion of anterior inferior cerebellar artery.


pontine syndrome
On side of lesion

Horizontal and vertical nystagmus, vertigo, nausea,


vomiting, oscillopsia: Vestibular nerve or nucleus

Facial paralysis: Seventh nerve

Paralysis of conjugate gaze to side of lesion: Center


for conjugate lateral gaze

Deafness, tinnitus: Auditory nerve or cochlear


nucleus

Ataxia: Middle cerebellar peduncle and cerebellar


hemisphere

Impaired sensation over face: Descending tract and


nucleus fifth nerve

On side opposite lesion

Impaired pain and thermal sense over one-half the


body (may include face): Spinothalamic tract

Medial Paramedian branch of midbasilar artery.


midpontine
syndrome On side of lesion

Ataxia of limbs and gait (more prominent in


bilateral involvement): Pontine nuclei

On side opposite lesion

Paralysis of face, arm, and leg: Corticobulbar and


corticospinal tract

Variable impaired touch and proprioception when


lesion extends posteriorly: Medial lemniscus

Lateral Short circumferential arter occlusion.


midpontine
syndrome On side of lesion

Ataxia of limbs: Middle cerebellar peduncle

Paralysis of muscles of mastication: Motor fibers or


nucleus of fifth nerve

Impaired sensation over side of face: Sensory fibers


or nucleus of fifth nerve

On side opposite lesion

Impaired pain and thermal sense on limbs and trunk:


Spinothalamic tract

Medial superior Paramedian branches of upper basilar artery.


pontine syndrome
On side of lesion

Cerebellar ataxia (probably): Superior and/or


middle cerebellar peduncle

Internuclear ophthalmoplegia: Medial longitudinal


fasciculus

Myoclonic syndrome, palate, pharynx, vocal cords,


respiratory apparatus, face, oculomotor apparatus,
etc.: Localization uncertaincentral tegmental
bundle, dentate projection, inferior olivary nucleus

On side opposite lesion

Paralysis of face, arm, and leg: Corticobulbar and


corticospinal tract

Rarely touch, vibration, and position are affected:


Medial lemniscus

Lateral superior Syndrome of superior cerebellar artery.


pontine syndrome
On side of lesion

Ataxia of limbs and gait, falling to side of lesion:


Middle and superior cerebellar peduncles, superior
surface of cerebellum, dentate nucleus

Dizziness, nausea, vomiting; horizontal nystagmus:


Vestibular nucleus

Paresis of conjugate gaze (ipsilateral): Pontine


contralateral gaze

Skew deviation: Uncertain

Miosis, ptosis, decreased sweating over face


(Horner's syndrome): Descending sympathetic fibers

Tremor: Localization unclearDentate nucleus,


superior cerebellar peduncle

On side opposite lesion

Impaired pain and thermal sense on face, limbs, and


trunk: Spinothalamic tract

Impaired touch, vibration, and position sense, more


in leg than arm (there is a tendency to incongruity of
pain and touch deficits): Medial lemniscus (lateral
portion)

Weber's P1 segment of PCA occlusion


syndrome
Infarction usually occurs in the ipsilateral
subthalamus and medial thalamus and in the
ipsilateral cerebral peduncle and midbrain.

A third nerve palsy with contralateral hemiplegia


Benedikt Caused by alesion(infarction,hemorrhage, tumor,
syndrome ortuberculosis) in thetegmentumof the midbrain and
cerebellum. Specifically, the median zone is
impaired. It can result from occlusion of
theposterior cerebral arteryor paramedian
penetrating branches of the basilar artery.

It is characterized by the presence of anoculomotor


nerve (CN III) palsyand cerebellarataxiaincluding
tremor.Neuroanatomicalstructures affected include
CNIII nucleus, Red nucleus, corticospinal tracts,
brachium conjunctivum, and the superior cerebellar
peduncle decussation. It is very similar in etiology,
morphology and clinical presentation toWeber's
syndrome; the main difference between the two
being that Weber's is more associated
withhemiplegia(i.e. paralysis), and Benedikt's
withhemiataxia(i.e. disturbed coordination of
movements).

The thalamic Occlusion of the penetrating branches of thalamic


Djrine-Roussy and thalamogeniculate arteries produces less
syndrome extensive thalamic and thalamocapsular lacunar
syndromes.

The syndrome consists of contralateral hemisensory


loss followed later by an agonizing, searing or
burning pain in the affected areas. It is persistent
and responds poorly to analgesics. Anticonvulsants
(carbamazepine or gabapentin) or tricyclic
antidepressants may be beneficial

Blint's syndrome Involves deficits in the orderly visuomotor scanning


of the environment (oculomotor apraxia) and in
accurate manual reaching toward visual targets
(optic ataxia). The third and most dramatic
component of Blint's syndrome is known as
simultanagnosia and reflects an inability to integrate
visual information in the center of gaze with more
peripheral information. The patient gets stuck on the
detail that falls in the center of gaze without
attempting to scan the visual environment for
additional information. A patient with
simultanagnosia "misses the forest for the trees."
Complex visual scenes cannot be grasped in their
entirety, leading to severe limitations in the visual
identification of objects and scenes. For example, a
patient who is shown a table lamp and asked to
name the object may look at its circular base and
call it an ashtray. Some patients with
simultanagnosia report that objects they look at may
vanish suddenly, probably indicating an inability to
look back at the original point of gaze after brief
saccadic displacements. Movement and distracting
stimuli greatly exacerbate the difficulties of visual
perception. Simultanagnosia sometimes can occur
without the other two components of Blint's
syndrome.

Frontal abulic The patient shows a loss of initiative, creativity, and


syndrome curiosity and displays a pervasive emotional
blandness and apathy.

Frontal The patient becomes socially disinhibited and shows


disinhibition severe impairments of judgment, insight, and
syndrome foresight. The dissociation between intact
intellectual function and a total lack of even
rudimentary common sense is striking. Despite the
preservation of all essential memory functions, the
patient cannot learn from experience and continues
to display inappropriate behaviors without appearing
to feel emotional pain, guilt, or regret when those
behaviors repeatedly lead to disastrous
consequences. The impairments may emerge only in
real-life situations when behavior is under minimal
external control and may not be apparent within the
structured environment of the medical office.

Millard-Gubler Involvement of Pons: Basis pontis and fascicles of


Syndrome CN VI and VII due to involvement of Short
circumferential and Paramedian branches of basilar
artery.

A unilateral lesion of the ventrocaudal pons may


invovle the basis pontis and the fascicles of cranial
nerves VI and VII. Symptoms include:

Contralateral hemiplegia (sparing the face) due to


pyramidal tract involvement

Ipsilateral lateral rectus palsy with diplopia that is


accentuated when the patient looks toward the
lesion, due to cranial nerve VI involvement.

Ipsilateral peripheral facial paresis, due to cranial


nerve VII involvemen
Claude's Caused bymidbraininfarctionas a result of occlusion
syndrome of a branch of theposterior cerebral artery.[1]This
lesion is usually a unilateral infarction of thered
nucleusandcerebral peduncle, affecting several
structures in the midbrain including:

Dentato-rubral fibres = Contralateral ataxia

Cortocospinal tract = Contralateral hemiparesis

Corticobulbar fibres = Contralateral hemiplegia of


lower facial muscles, tongue & shoulder.

Oculomotor nerve fibres = Ipsilateral oculomotor


nerve fibres with a drooping eyelid & fixed wide
pupil pointed down and out, probabale diplopia.

(Q.106) Which of following ischaracterized by enlarged gastric rugae, acid hypersecretion


and protein loss?

(a) Granulomatousgastritis

(b) Eosinophilic gastritis

(c) Varioliform gastritis

(d) Mntriers disease

Your Response :

Correct Answer : d

Exp: d) menetriers disease

Mntrier disease is a rare, acquired, premalignant disease of the


stomach characterized by massive gastric folds, excessive mucous
production with resultant protein loss, and little or no acid production.
The disorder is associated with excessive secretion of transforming
growth factor alpha (TGF-)

(Q.107) PAIR technique is used in treatment of

(a) Pyogenic liver abscess

(b) Liver tumours

(c) Hydatid cyst

(d) Portal hypertension


Your Response :

Correct Answer : c

Exp: c) Hydatid cyst

The technique of PAIR (Puncture Aspiration Instillation of scolicidal


agent and Reaspiration )

It was initially discouraged because of risk of anaphylaxis. However


studies have shown the risk of spillage to be minimal.

Indications for percutaneous treatment include univesicular cysts (WHO


classification type CE 1), univesicular cysts with detached membrane
(WHO type CE 3), and some multiple cysts (WHO type CE 2).

Other ill defined indications are in

a) Recurrent cystic collections after surgery for hydatid cyst

b) Hydatid cyst in pregnancy as a first line

PAIR contraindicated

in cysts that are inaccessible to puncture, in cysts in which puncture may


damage important vascular structures, and in peripheral cysts that do not
have a sufficient layer of hepatic tissue that permits transhepatic
puncture.

3 types of PAIR tecniques in Hydatid cyst liver

(Q.108) Kussmauls sign is not positive in

(a) Cardiac tamponade

(b) Constrictive pericarditis

(c) Restrictive cardiomyopathy

(d) RVMI

Your Response :

Correct Answer : a

Exp: a) Cardiac tamponade

Difference between cardiac tamponade and constrictive pericarditis

Cardiac tamponade Constrictive


pericarditis

Paradoxical pulse Present Only in one third of


patients

Equal right and left Yes Yes


filling pressure

JVP Absent Y descent Rapid Y descent

Kussmaul sign :- Absent (normal fall of Present (increase or no


Inspiratory change in systemic venous change in systemic
systemic venous pressure) venous pressure)
pressure

Square root sign in Absent Present


ventricular pressure
tracing

Signs commonly associated with Kussmaul.

Constrictive Restrictive Pericardial


pericarditis cardiomyopathy tamponade

Kussmauls Yes Yes No


sign

Pulsus Yes Yes Yes


paradoxus

(Q.109) Bacillary angiomatosis in HIV is caused by

(a) EBV

(b) Borellia burgdorferi

(c) B. henselae

(d) HHV8

Your Response :

Correct Answer : c

Exp: c) B. henselae

Bacillary angiomatosis is a systemic illness characterised by lesions


similar to those of Kaposi sarcoma in the skin, mucosal surfaces, liver,
spleen and other org It is caused by bacterial infection with Bartonella
quintana and Bartonella henselae (cause of catscratch disease and also
known as Rochalimaea henselae). The disease is only rarely seen in
healthy immunocompetent people. It mostly affects patients with
immunodeficiency, particularly those with AIDS or HIV.

(Q.110) In hypopituitarism all are trueexcept?

(a) Selective gonadotrophin deficiency may be present

Concomitant diabetes insipidus (DI) may be masked by anterior


(b)
pituitary failure

Adrenal steroid replacement must be started before thyroid


(c)
replacement

(d) Mineralocorticoid replacement is usually necessary

Your Response :

Correct Answer : d

Exp: (D) mineralocorticoid replacement is usually necessary

Isolated deficiencies of gonadotrophins or growth hormone are well


described and may be due to failure of secretion of their respective
hypothalamic releasing hormones. Concomitant cortisol deficiency
reduces the severity of and may even conceal diabetes insipidus, possibly
because it lowers glomerular filtration rate. DI may therefore be revealed
by adrenal replacement therapy. In combined adrenal and thyroid failure,
an adrenal crisis may be precipitated by starting thyroxine replacement
therapy before corticosteroids. Aldosterone synthesis and secretion occur
in the zona glomerulosa of the adrenal cortex and are largely ACTH
independent. Mineralocorticoid deficiency sufficient to require
replacement therapy is therefore rare in hypopituitarism. Fertility
depends on the gonadotrophins, which must be replaced for fertility to be
achieved.

(Q.111) In anorexia nervosa

(a) Loss of pubic hair occurs

(b) Patients may present with primary amenorrhea

(c) Luteinizing hormone (LH) levels are elevated

(d) ESR is high

Your Response :
Correct Answer : b

Exp: (B). Patients may present with primary amenorrhoea

Anorexia nervosa is associated with a number of endocrine


abnormalities, most apparently due to malnutrition or weight loss. These
include low gonadotrophin levels with loss of the normal pulsatile
pattern of secretion (a profile typical of prepubertal girls), high-normal or
supranormal cortisol levels (possibly related to depression), and high
growth hormone levels (possibly due to reduced negative feedback on
the pituitary by low circulating levels of insulin-like growth factors). The
eating disorder and weight loss may begin before menarche and so
prevent pubertal gonadotrophin release and the appearance of puberty
itself. An elevated ESR suggests organic disease (the ESR is normal or
low in anorexia), whereas loss of pubic hair suggests hypopituitarism (in
anorexia, pubic hair is conserved and fine, dark lanugo hair may appear
on the body and face).

This is a very high yield topic and the following table from HARRISON
17th Ed Chapter 76 is very helpful

Common Characteristics of Anorexia Nervosa and Bulimia Nervosa

Anorexia Nervosaa Bulimia Nervosa

Clinical Characteristics

Onset Mid-adolescence Late adolescence/early


adulthood

Female:male 10:1 10:1

Lifetime prevalence 1% 13%


in women

Weight Markedly decreased Usually normal

Menstruation Absent Usually normal

Binge eating 2550% Required for diagnosis

Mortality Low
5% per decade

Physical and Laboratory Findingsa

Skin/extremities Lanugo
Acrocyanosis

Edema

Cardiovascular Bradycardia

Hypotension

Gastrointestinal Salivary gland Salivary gland


enlargement enlargement

Slow gastric emptying Dental erosion

Constipation

Elevated liver enzymes

Hematopoietic Normochromic,
normocyctic anemia

Leukopenia

Fluid/Electrolyte Increased BUN, Hypokalemia


creatinine
Hypochloremia
Hypokalemia
Alkalosis

Endocrine Hypoglycemia ,

Low estrogen or
testosterone

Low LH and FSH, Low-


normal thyroxine

Normal TSH, Increased


cortisol

Bone Osteopenia

(Q.112) Features of primary autoimmune hypothyroidism include all except?

(a) Increased incidence of type1 DM & Addison's disease

(b) Brisk deep tendon reflexes

(c) Ataxia

(d) Paranoia and delusions


Your Response :

Correct Answer : b

Exp: (B). Brisk deep tendon reflexes

Autoimmune thyroid failure is associated with other organ-specific


autoimmune disease; the combination with Addison's disease is referred
to as 'Schmidt's syndrome'. Pericardial and pleural effusions and ascites,
reversible with thyroid replacement, may occur. Rare neurological
complications include bilateral cerebellar damage and 'myxoedema
madness' (first described by Richard Asher), which includes
hallucinations, agitation, delusions and paranoia. Pretibial 'myxoedema'
consisting of cutaneous hyaluronic acid deposits and most often
appearing on the shins, is a feature of Graves' disease. In hypothyroid
deep tendon reflexes are not brisk. rather deep tendon reflexes relaxation
is delayed.

(Q.113) Most common extra renal manifestation of nephronophthisis is?

(a) Retinitis pigmentosa

(b) VSD

(c) Cerebellar ataxia

(d) Hepatic fibrosis

Your Response :

Correct Answer : a

Exp: (A). Retinitis pigmentosa

Up to 15% of patients with juvenile NPHP have extrarenal


manifestations, most commonly retinitis pigmentosa (Senior-Loken
syndrome). Other abnormalities include blindness from amaurosis,
oculomotor apraxia, cerebellar ataxia, mental retardation, hepatic
fibrosis, and ventricular septal defect. Situs inversus is seen in some
cases of infantile NPHP.

(Q.114) Isovolumetric relaxation ends with:

(a) Beginning of "C'' wave in JVP

(b) Opening of AV valves

(c) Closure of semilunar valves


(d) T waves on ECG

Your Response :

Correct Answer : b

Exp: B (Opening of AV valves)

Phases Of Cardiac cycle: (Total duration = 0.8 minutes)

(Q.115) True about Pulsus bisferiens:

(a) Double peak, one is systole and another in diastole

(b) Double peak, both in systole

(c) Seen in patients with dilated cardiomyopathy

(d) Seen only when aortic stenosis is associated with aortic regurgitation

Your Response :

Correct Answer : b

Exp: B (Double peak, both in systole)

VARIOUS ARTERIAL PULSES

-------------------------------------------------------------------------------------------------------

Arterial pulse Mechanism Etiology

-------------------------------------------------------------------------------------------------------

Pulsus paradoxus Decrease in SBP (<10mmHg) that normally Pericardial


temponade

decrease in arterial pulse Airway obstruction

amplitude during inspiration SVC obstruction

accentuated

Pulsus bigeminus Regular alteration of pressure pulse amplitude PVCs

Pulsus alternans Regular alteration of pressure pulse amplitude LVF, PAT


after premature beats

Pulsus bisferiens Two systolic peaks AR/HOCM

(Percussion and tidal wave)


Dicrotic pulse 2 palpable waves, 1 in systole 1 in diastole DCM

Pulsus parvus Small weak pulse LVF, MS, Restrictive

(Hypokinetic) pericardial disease

Pulsus tardus Delayed systolic peak AS

Hyperkinetic CHB, anemia, fever,

Exercise, hyperthyroid

Bounding pulse VSD, MR

-------------------------------------------------------------------------------------------------------

(Q.116) This type of hand deformitiesare seen in which condition?

(a) Systemic sclerosis

(b) RA

(c) OA

(d) Psoriatic arthritis

Your Response :

Correct Answer : b
Exp: B. RA

The hand in rheumatoid arthritis. (RA)

a. Ulnar deviation of the fingers with wasting of the small muscles of the
hands and synovial swelling at the wrists, the extensor tendon sheaths,
the metacarpophalangeal and proximal interphalangeal joints.

b. 'Swan neck' deformity of the fingers.

(Q.117) Pulsus bisferiens is best felt in:

(a) Carotid artery

(b) Brachial artery

(c) Radial artery

(d) Femoral artery

Your Response :

Correct Answer : c

Exp: : C (Radial artery)

Usually palpation of peripheral arterial pulses such as radial artery gives


less information than examination of a more central pulse (Carotid pulse)
regarding alterations in left ventricular ejection or Aortic valve function.
However certain findings such as Bisferiens pulse of Aortic regurgitation
or pulses alternans are more evident in peripheral arteries.

(Q.118) Which is the correct statement regarding JVP:

(a) Cannon wave: Complete heart block

(b) Slow vy descent: Tricuspid regurgitation

(c) Giant c wave: Tricuspid stenosis

(d) Increased NP with prominent pulsations: SVC obstruction

Your Response :

Correct Answer : a

Exp: : A (Cannon wave in complete heart block)

Cannon waves are large a waves. They may be seen in complete heart
block.
Slow v-y descent in the JVP suggests an obstruction to right ventricular
filling, as occurs with Tricuspid Stenosis or Right Atrial Myxoma.

Tricuspid Regurgitation causes the v-waves to be prominent, when TR


becomes severe, the combination of a prominent v-wave and obliteration
of x-descent results in a single large positive systolic wave.

Tricuspid stenosis- Positive presystolic a-wave is enlarged when the


right atrium is contracting against an increased resistance.

SVC obstruction causes raised JVP with absent pulsation.

(Q.119) Ventilation mode used primarily for weaning patients on mechanicalventilation ?

(a) Pressure support ventilation.

(b) Pressure control ventilation.

(c) Synchronized intermittent mandatory ventilation.

(d) Assist control mode ventilation.

Your Response :

Correct Answer : a

Exp: Pressure support ventilation.

HARRISON 17th Ed Chapter 263.

Pressure-Support Ventilation (PSV)

This form of ventilation is patient triggered, flow cycled, and pressure


limited; it is specifically designed for use in the weaning process. During
PSV, the inspiration is terminated when inspiratory airflow falls below a
certain level; in most ventilators this flow rate cannot be adjusted by the
operator. When PSV is used, patients receive ventilator assist only when
the ventilator detects an inspiratory effort (Fig. 263-2B). PSV can also be
used in combination with SIMV to ensure volume-cycled backup for
patients whose respiratory drive is depressed.

PSV is well tolerated by most patients who are being weaned; PSV
parameters can be set to provide full or nearly full ventilatory support
and can be withdrawn slowly over a period of days in a systematic
fashion to gradually load the respiratory muscles.

(Q.120) The Ig A anti TG 2 (Transglutaminase) is a sensitive marker for diagnosis of

(a) Celiac Disease


(b) Eosinophilic Gastroenteritis

(c) Primary Immunodeficiency

(d) Crohns Disease

Your Response :

Correct Answer : a

Exp: Ans A) Celiac Disease

Anti-tissue transglutaminase

Antibodies to tissue transglutaminase (ATA or anti-tTG) are found in


patients with several conditions, including coeliac disease, juvenile
diabetes,inflammatory bowel disease,and various forms of arthritis.

In Coeliac Disease, ATA are involved in the destruction of the villous


extracellular matrix and target the destruction of intestinal villous
epithelial cells by killer cells. Deposits of anti-tTG in the intestinal
epithelium predict coeliac disease.

Anti-endomysial reactivity

The endomysium is a layer of connective tissue that ensheaths a muscle


fiber. The endomysium contains a form of transglutaminase called
"tissue transglutaminase" or "tTG" for short, and antibodies that bind to
this form of transglutaminase are called anti-endomysial antibodies
(EMA).The antiendomysial antibody test is a histological assay for
patient serum binding to esophageal tissue from primate. EMA are
present in celiac disease. They do not cause any direct symptoms to
muscles, but detection of EMA is useful in the diagnosis of the disease.

Anti-epidermal transglutaminase

Antibodies to epidermal transglutaminase (eTG, also keratinocyte


transglutaminase) are the autoantibodies believed to cause dermatitis
herpetiformis

(Q.121) The maternal uniparental disomy for chromosome 15 results in

(a) Angelman Syndrome

(b) Prader Willi Syndrome

(c) Pallister Killian Syndrome

(d) Hypomelanosis of ITO


Your Response :

Correct Answer : b

Exp: Ans B) Prader Willi Syndrome

Angelman syndrome: In most cases (about 70%)people with Angelman


syndrome have a deletion in the maternal copy of chromosome 15. This
chromosomal change deletes the region of chromosome 15 that includes
the UBE3A gene. Because the copy of the UBE3A gene inherited from a
person's father (the paternal copy) is normally inactive in the brain, a
deletion in the maternal chromosome 15 results in no active copies of the
UBE3A gene in the brain.

Prader Willi Syndrome: In about 70% of cases,Prader-Willi syndrome


occurs when the 15q11-q13 region of the paternal chromosome 15 is
deleted. The genes in this region are normally active on the paternal copy
of the chromosome and are inactive on the maternal copy. Therefore, a
person with a deletion in the paternal chromosome 15 will have no active
genes in this region.

(Q.122) The average whey/casein ratio in Breast milk is

(a) 60 : 40

(b) 80 : 20

(c) 20 : 80

(d) 40 : 60

Your Response :

Correct Answer : a

Exp: Ans A)60 : 40

Composition of human breast


milkhttp://en.wikipedia.org/wiki/Breast_milk - cite_note-30

Fat

total (g/100 ml) 4.2

fatty acids - length 8C (% ) trace

polyunsaturated fatty acids (%) 14

Protein(g/100 ml)
total 1.1

casein 0.4 0.3

a-lactalbumin 0.3

lactoferrin (apo-lactoferrin) 0.2

IgA 0.1

IgG 0.001

lysozyme 0.05

serum albumin 0.05

-lactoglobulin -

Carbohydrate(g/100 ml)

lactose 7

oligosaccharides 0.5

Minerals(g/100 ml)

calcium 0.03

phosphorus 0.014

sodium 0.015

potassium 0.055

chlorine 0.043

Human milk contains two types of proteins: whey and


casein. Approximately 60% is whey, while 40% is casein. This balance
of the proteins allows for quick and easy digestion.

(Q.123) Sweaty feet odour to urine occurs in

(a) Homocystinuria

(b) Isovaleric acidemia

(c) Phenylketonuria

(d) Alkaptonuria

Your Response :
Correct Answer : b

Exp: Ans B) Isovaleric academia

Odor originating from Metabolic Disorder

a. Trimethylaminuria

Trimethylaminuria also called fish odor syndrome or TMAU is a genetic


disease. It is due to abnormal excretion of trimethylamine in the breath,
urine, sweat, saliva and vaginal secretions. The odor produced is similar
to decaying fish. The smell consists of sulfur compounds, plus nitrogen
compounds (amines).

b. Maple Syrup Urine Disease

Babies that suffer from the mild form have a sugary smell. Adults may
have a burned sugar smell to urine. There second form responds well to
the vitamin thiamine. It is reported that the patients smell like caramel,
maple syrup or have a malty odor.

c. Phenylketunuria

The person may present a musty, mousy, wolflike, barny, horsey or stale
smell.

d. Multiple Acyl-CoA Dehydrogenase Deficiency

The person presents variable body odor of sweaty feet.

e. Isovaleric Acidaemia

Isovaleric acidaemia is a rare genetic condition in which the body is


unable to process certain proteins properly. People with this disorder
have abnormal levels of an enzyme that helps break down the amino acid
leucine, a building block of proteins. It is related to the genetic metabolic
disorder Maple syrup syndrome. A symptom of isovaleric acidemia is a
odor of cheesy, acrid,sweaty feet. This odor is result of the buildup of
isovaleric acid compound.

f. Tyrosinaemia

One of the symptoms of Tyrodinaemia type 1 is a odor like cabbage or


rancid butter.

g. Diabetes Mellitus

Fruity breath is present in people with diabetes.

h. Diabetic Ketoacidosis

The patient may have a fruity breath, a sweet taste on the skin, or
emanate a distinctive, chemical smell.

i. 3-Methylcrotonylglycinuria

The patient presents an odor like male cat urine.

j. Cystinuria

Because cystine is one of the sulfur-containing amino acids, the urine


may have a characteristic "rotten egg" odor.

k. Hypermethioninemia

Hypermethioninemia this condition can happen when methionine (amino


acid) is not metabolized correctly in the body. Individuals with this
condition may experience a fishy, sweety and fruity, rancid butter or
boiled cabbage odor. It has been said that tyrosinosis-tyrosinemia is the
same as hypermethioninemia.

l. Oast-House Syndrome

The urine has an odor similar to that of dried celery, yeast or malt, or an
oasthouse (a building for drying hops).

m. Congenital Adrenal Hyperplasia

The non classical or milder form of CAH can produce body odor during
childhood because of the premature puberty.

(Q.124) Which of the following is a class II anti-arrhythmicdrug ?

(a) Propranolol

(b) Digoxin

(c) Amiodarone

(d) Lidocaine

Your Response :

Correct Answer : a

Exp: Ans A) Propranolol

Carvedilol, Propranolol, Esmolol, Timolol, Metoprolol, Atenolol,


Bisoprolol are class II anti arrhtymic drugs

(Q.125) Acute febrile illness with vasculitis having predilection for the coronary arteries is
seen in
(a) Kawasaki disease

(b) Adenovirus infection

(c) Diphtheria

(d) Measles

Your Response :

Correct Answer : a

Exp: Ans A) Kawasaki disease

Kawasaki disease is a condition that causes inflammation in the walls of


medium-sized arteries throughout the body, including the coronary
arteries, which supply blood to the heart muscle. Kawasaki disease is
also called mucocutaneous lymph node syndrome because it also affects
lymph nodes, skin, and the mucous membranes inside the mouth, nose
and throat.

(Q.126) Microangiopathic hemolytic anemia with thrombocytopenia and renal insufficiency


is commonly seen in young children with

(a) Sickle cell anemia

(b) Hemolytic uremic syndrome

(c) Malaria

(d) S.L.E (systemic lupus erythematosus)

Your Response :

Correct Answer : b

Exp: Ans B) Hemolytic uremic syndrome

Hemolytic-uremic syndrome (HUS) is a clinical syndrome characterized


by progressive renal failure that is associated with microangiopathic
(nonimmune, Coombs-negative) hemolytic anemia and
thrombocytopenia. HUS is the most common cause of acute renal failure
in children and is increasingly recognized in adults.

Hemolytic-Uremic Syndrome

There are at least four variants of HUS.

The most common is D+ HUS referring to its association with bacterial


gastroenteritis.
This typically affects young children (<5 years), but adults are also
susceptible.

More than 80% of cases are preceded within a week by diarrhea, often
bloody.

Gastrointestinal symptoms include abdominal pain, cramping, and


vomiting.

(Q.127) Blueberry muffins are seen in

(a) Congenital rubella

(b) Acquired rubella

(c) Syphilis

(d) Rubeola

Your Response :

Correct Answer : a

Exp: a) Congenital rubella

Blueberry muffin syndrome is the descriptive term used when an infant


is born with multiple blue/purple marks or nodules in the skin. These are
due to the presence of clusters of blood-producing cells in the skin
(extramedullary erythropoiesis), or bleeding into the skin (purpura) or
spreading cancer (metastases).

Causes of blueberry muffin syndrome

Tumours such as::

Congenital leukaemia cutis Langherhans cell histiocytosis

Neuroblastoma Congenital rhabdomyosarcoma

Blood disorders such as:

- Haemolytic disease of the newborn rhesus or ABO incompatibility


- Hereditary spherocytosis

- Twin-twin transfusion syndrome

Congenital infections such as:

Rubella Toxoplasmosis Cytomegalovirus

Herpes simplex Coxsackie virus Parvovirus


Epstein Barr virus Syphilis

The TORCH complex is a medical acronym used for these serious


congenital infections: Toxoplasmosis, Other infections, Rubella,
Cytomegalovirus, Herpes simplex virus. The Other infections are
hepatitis B, coxsackie virus, syphilis, varicella-zoster virus and
parvovirus B19.

(Q.128) What is the genetic defect of Downs syndrome?

(a) Trisomy 21

(b) Mosaicism

(c) Robertsonian translocation

(d) All

Your Response :

Correct Answer : d

Exp: D) All

Down syndrome results when abnormal cell division involving


chromosome 21 occurs. These cell division abnormalities result in extra
genetic material from chromosome 21, which is responsible for the
characteristic features and developmental problems of Down syndrome.
Any one of three genetic variations can cause Down syndrome:

Trisomy 21.About 95 percent of the time, Down syndrome is caused by


trisomy 21 the child has three copies of chromosome 21 (instead of
the usual two copies) in all cells. This is caused by abnormal cell
division during the development of the sperm cell or the egg cell.

Mosaic Down syndrome.In this rare form of Down syndrome, children


have some cells with an extra copy of chromosome 21. This mosaic of
normal and abnormal cells is caused by abnormal cell division after
fertilization.

Translocation Down syndrome.Down syndrome can also occur when


part of chromosome 21 becomes attached (translocated) onto another
chromosome, before or at conception. These children have the usual two
copies of chromosome 21, but they also have additional material from
chromosome 21 attached to the translocated chromosome.

(Q.1 Most common cause of nephrotic syndrome inpediatric population?


29)
(a) Minimal change disease
(b) Mesangial proliferation

(c) FSGS

(d) MPGN

Your Response :

Correct Answer : a

Ex A. Minimal change ds
p:
The most common cause of neprotic synd in children is MCD.

Summary of Primary Renal Diseases That Present as Idiopathic Nephrotic


Syndrome

MINIMA FOCAL MEMBRAN MEMBRANOPROLIF


L SEGMENTAL OUS ERATIVE
CHANG GLOMERULOSCL NEPHROPA GLOMERULONEPHR
E EROSIS THY ITIS
NEPHRO
TIC
SYNDRO
ME

Type I Type II

FREQUENC
Y[*]

Children 75% 10% <5% 10% 10%

Adults 15% 15% 50% 10% 10%

Clinical
Manifestation
s

Age (yr) 26, some 210, some adults 4050 515 515
adults

Sex 2 : 1 male 1.3 : 1 male 2 : 1 male Male-female Male-


female

Nephrotic 100% 90% 80% 60% 60%


syndrome

Asymptomatic 0 10% 20% 40% 40%


proteinuria
Hematuria 1020% 6080% 60% 80% 80%

Hypertension 10% 20% early Infrequent 35% 35%

Rate of Does not 10 yr 50% in 1020 1020 yr 515 yr


progression to progress yr
renal failure

Associated Allergy? None Renal vein None Partial


conditions Hodgkin thrombosis, lipodystrop
disease, cancer, SLE, hy
usually hepatitis B
none

Laboratory Manifestat Manifestations of Manifestation


Findings ions of nephrotic syndrome s of nephrotic
nephrotic
syndrome

BUN in BUN in 2040% syndrome Low C1, C4, Normal C1,


1530% C3C9 C4, low

C3C9

Immunogeneti HLA-B8, Mutations in podocin, HLA-DRw3 Not C3 nephritic


cs B12 -actinin-4, other (1232)[] established factor Not
[]
(3.5) genes established

Renal
Pathology

Light Normal Focal sclerotic lesions Thickened Thickened Lobulation


microscopy GBM, spikes GBM,
proliferation

Immunofluore Negative IgM, C3 in lesions Fine granular Granular C3 only


scence IgG, C3 IgG, C3

Electron Foot Foot process fusion Subepithelial Mesangial Dense


microscopy process deposits and deposits
fusion subendotheli
al deposits

Response to 90% 1520% May be slow Not Not


Steroids progression established established

(Q.130) A 3.8 kg baby of a diabetic mother developed seizures 16 hours after birth. The
most probable cause would be?
(a) Hypoglycemia

(b) Hypocalcemia

(c) Birth asphyxia

(d) Intraventricular hemorrhage

Your Response :

Correct Answer : a

Exp: A. Hypoglycemia (Ref. OP Ghai 7th ed. 156, 177; Nelson


paediatrics 18th ed. 784)

The neonate has seizures developed at around 16 hrs after birth. There
are two possibilities in this neonate i.e. hypoglycemia and hypocalcemia.
Hypoglycemia in first 18-36 hrs is usually due to transient neonatal
hypoglycemia and after 36 hrs metabolic and other causes are likely.

Hypoglycemia develops in about 2550% of infants of diabetic mothers


and 1525% of infants of mothers with gestational diabetes, but only a
small percentage of these infants become symptomatic.

The probability of hypoglycemia developing in the infant increases, and


glucose levels are likely to be lower at higher cord or maternal fasting
blood glucose levels.

In those infants with HIE who have seizures, onset of seizures is


generally within the first 24 hours after birth. However, the timing of
onset is not a reliable indicator of the timing of the neurologic injury.

Seizures due to intracranial hemorrhage may also be associated with


hypoxic-ischemic or traumatic injury since these events are frequently
associated with each other. Onset of seizures due to subarachnoid
hemorrhage or subdural hemorrhage is usually the second or third day of
life, while those due to germinal matrix-intraventricular hemorrhage
present after the third day.

(Q.131) The best predictor of prognosis in congenital diaphragmatic hernia?

(a) Pulmonary hypertension

(b) Gestational age at which baby is born

(c) Time of surgery

(d) Size of the herniated sac

Your Response :
Correct Answer : a

Exp: Pulmonary hypertension (Ref. Nelson paediatrics 18th


ed. 751)

The infant born with congenital diaphragmatic hernia (CDH) remains


one of the most complex patients to manage.

Pulmonary hypoplasia and immaturity of the lungs remain the leading


cause of death, from pulmonary hypertension (right-to-left shunting)
with resultant hypoxemia.

The best predictor of prognosis in congenital diaphragmatic hernia


Pulmonary hypertension.

Pulmonary problems continue to be a source of morbidity for long-term


survivors of CDH.

Those without severe pulmonary hypertension and barotrauma do the


best.

The ideal time to repair the diaphragmatic defect is under debate. Most
centers will wait at least 48 hr after stabilization and resolution of the
pulmonary hypertension.

Whenever possible, a primary repair using native tissue is performed. If


the defect is too large, a Gore-Tex patch is use.

(Q.132 Increased risk of liver adenoma is found in?


)
(a) Lipid storage disorder

(b) Mitochondrial disease

(c) Glycogen storage disorder

(d) Lysosomal storage disorder

Your Response :

Correct Answer : c

Exp C Glycogen storage disorder.


:
Increased risk of hepatic adenomas is a feature of glycogen storage disorder type 1
(Von Geirke's disease).

Features of the Disorders of Carbohydrate Metabolism

DISORDERS BASIC DEFECTS CLINICAL COMMENTS


PRESENTATION

LIVER
GLYCOGENOSES

Type/Common Name

Ia/Von Gierke Glucose-6- Growth retardation, Common,


phosphatase hepatomegaly, severe
hypoglycemia; elevated hypoglycemia
blood lactate,
cholesterol, triglyceride,
and uric acid levels

Ib Glucose-6- Same as type Ia, with 10% of type Ia


phosphate additional findings of
translocase neutropenia and
impaired neutrophil
function

IIIa/Cori or Forbes Liver and muscle Childhood:hepatomegal Common,


debrancher y, growth retardation, intermediate
deficiency (amylo, muscle weakness, severity of
1,6 glucosidase) hypoglycemia, hypoglycemia
hyperlipidemia, elevated
transaminase levels;
liver symptoms improve
with age

IIIb Liver debrancher Liver symptoms same as 15% of type III


deficiency; normal in type IIIa;no muscle
muscle enzyme symptoms
activity

IV/Andersen Branching enzyme Failure to thrive, Rare


hypotonia, neuromuscular
hepatomegaly, variants exist
splenomegaly,
progressive cirrhosis
(death usually before
5th yr), elevated
transaminase levels

VI/Hers Liver phosphorylase Hepatomegaly, mild Rare, benign


hypoglycemia, glycogenosis
hyperlipidemia, and
ketosis
Phosphorylase kinase Phosphorylase Hepatomegaly, mild Common,
deficiency kinase hypoglycemia, benign
hyperlipidemia, and glycogenosis
ketosis

Glycogen synthetase Glycogen synthetase Early morning Decreased liver


deficiency drowsiness and fatigue, glycogen store
fasting hypoglycemia,
and ketosis

Fanconi-Bickel Glucose transporter Failure to thrive, rickets, GLUT-2


syndrome 2 (GLUT-2) hepatorenomegaly, expressed in
proximal renal tubular liver, kidney,
dysfunction, impaired pancreas, and
glucose and galactose intestine
utilization

MUSCLE
GLYCOGENOSES

Type/Common Name

II/Pompe Infantile Acid -glucosidase Cardiomegaly, Common,


(acid maltase) hypotonia, cardiorespirator
hepatomegaly; y failure leading
onset:birth6 mo to death by age
2 yr

Juvenile Acid -glucosidase Myopathy, variable Residual


(acid maltase) cardiomyopathy; enzyme activity
onset:childhood

Adult Acid -glucosidase Myopathy, respiratory Residual


(acid maltase) insufficiency; enzyme activity
onset:adulthood

Danon disease Lysosome- Hypertrophic Rare, X-linked


associated cardiomyopathy
membrane protein 2
(LAMP2)

PRKAG2 deficiency AMP-activated Hypertrophic Autosomal


protein kinase cardiomyopathy dominant

V/McArdle Myophosphorylase Exercise intolerance, Common, male


muscle cramps, predominance
increased fatigability
VII/Tarui Phosphofructokinas Exercise intolerance, Prevalent in
e muscle cramps, Japanese and
hemolytic anemia, Ashkenazi Jews
myoglobinuria

Phosphoglycerate Phosphoglycerate As with type V Rare, X-linked


kinase deficiency kinase

Phosphoglycerate M subunite of As with type V Rare, majority


mutase deficiency phosphoglycerate of patients are
mutase African-
American

Lactate dehydrogenase M subunit of lactate As with type V Rare


deficiency dehydrogenase

GALACTOSE
DISORDERS

Galactosemia with Galactose-1- Vomiting, African-


transferase deficiency phosphate hepatomegaly, cataracts, American
uridyltransferase aminoaciduria, failure to patients tend to
thrive have milder
symptoms

Galactokinase Galactokinase Cataracts Benign


deficiency

Generalized uridine Uridine diphosphate Similar to transferase A benign


diphosphate galactose- galactose-4- deficiency with variant also
4-epimerase deficiency epimerase additional findings of exists
hypotonia and nerve
deafness

FRUCTOSE
DISORDERS

Essential fructosuria Fructokinase Urine reducing Benign


substance

Fructose-1- Acute:vomiting,
phosphate aldolase sweating, lethargy

Hereditary fructose Chronic:failure to Prognosis good


intolerance thrive, hepatic failure with fructose
restriction

DISORDERS OF
GLUCONEOGENESI
S

Fructose-1,6- Fructose-1,6- Episodic hypoglycemia, Good


diphosphatase diphosphatase apnea, acidosis prognosis,
deficiency avoid fasting

Phosphoenolpyruvate Phosphoenolpyruvat Hypoglycemia, Rare


carboxykinase e carboxykinase hepatomegaly,
deficiency hypotonia, failure to
thrive

DISORDERS OF
PYRUVATE
METABOLISM

Pyruvate Pyruvate Severe fatal neonatal to Most


dehydrogenase complex dehydrogenase mild late onset, lactic commonly due
defect acidosis, psychomotor to E1 subunit,
retardation, and failure defect X-linked
to thrive

Pyruvate carboxylase Pyruvate Same as above Rare, autosomal


deficiency carboxylase recessive

Respiratory chain Complex I to V, Heterogeneous with Mitochondrial


defects (oxidative many mitochondrial multisystem inheritance
phosphorylation DNA mutations involvement
disease)

DISORDERS IN
PENTOSE
METABOLISM

Pentosuria L-xylulose Urine reducing Benign


reductase substance

Transaldolase Transaldolase Liver cirrhosis and Autosomal


deficiency failure, cardiomyopathy recessive

Ribose-5-phosphate Ribose-5-phosphate Progressive


isomerase deficiency isomerase leukoencephalopathy
and peripheral
neuropathy

(Q.133) Most common cause of meningoencephalitis is children?

(a) Enteroviruses
(b) Arboviruses

(c) Herpes virus

(d) Cytomegalovirus

Your Response :

Correct Answer : c

Exp: C. Herpes virus.

Most common cause of meningoencephalitis is children is Herpes virus

Encephalitis is also called meningoencephalitis or encephalomyelitis, the


latter indicating .

In contrast to viral meningitis, where the infectious process and


associated inflammatory response are limited largely to the meninges, in
encephalitis the brain parenchyma is also involved. Many patients with
encephalitis also have evidence of associated meningitis
(meningoencephalitis)

The most important viruses causing sporadic cases of encephalitis in


immunocompetent adults are herpesviruses (HSV, VZV, EBV).

Epidemics of encephalitis are caused by arboviruses, which belong to


several different viral taxonomic groups including Alphaviruses (e.g.,
EEE virus, western equine encephalitis virus), Flaviviruses (e.g., WNV,
St. Louis encephalitis virus, Japanese encephalitis virus, Powassan
virus), and Bunyaviruses (e.g., California encephalitis virus serogroup,
Lacrosse virus).

Arbovirus = epidemic encephalitis

HSV-I; = sporadic encephalitis

Can occurs in any age group, yet, is most common in children. Q

Hemorrhagic Necrotizing temporal encephalitis.

PCR based detection of viral particles aides in the diagnosis.

HSV-II:

Generalized encephalitis (50% neonates born by VD with active primary


HSV vaginal infections)

Most common cause of viral meningitis = enterovirus.

(Q.134) True about Weaning of breast feeding is allexcept ?


(a) Begin at 6 mo of age

(b) Avoid foods with high allergenic potential

(c) Phytate intake should be maximized to enhance mineral absorption

(d) All of above

Your Response :

Correct Answer : c

Exp: C. Phytate intake should be maximized to enhance mineral


absorption Ref : NELSON 18th Ed Chap 42

WEANING FROM BREAST-FEEDING

Between 6 and 12 mo of age, after they become accustomed to solid


foods and liquids by bottle and/or cup, most infants decrease the volume
and frequency of breast-feeding. As the infant demands less milk, the
mother's supply gradually diminishes without causing discomfort from
engorgement. Weaning can be initiated when mutually desired by the
mother and infant by substituting formula by bottle or cup for part and,
subsequently, all of a breast-feeding. Breast-feeding is eventually
replaced with formula-feeding, at which time the infant is weaned
completely. Occasionally, an infant takes a cup as readily as a bottle. If
so, the intermediate transfer from breast to bottle before transferring
from bottle to cup can be avoided. These changes should be made
gradually and should be a pleasant experience, not a conflict, for both the
mother and the infant.

Important Principles for Weaning

Begin at 6 mo of age

Avoid foods with high allergenic potential (cow's milk, eggs, fish,
nuts, soybeans).

At the proper age, encourage a cup rather than a bottle.

Introduce 1 food at a time.

Energy density should exceed that of breast milk.

Iron-containing foods (meat, iron-supplemented cereals) are


required.

Zinc intake should be encouraged with foods such as meat, dairy


products, wheat, and rice.
Phytate intake should be low to enhance mineral absorption.

Breast milk should continue to 12 mo; formula or cow's milk is


then substituted.Give no more than 24 oz/day of cow's milk.

Fluids other than breast milk, formula, and water should be


discouraged. Give no more than 46 oz/day of fruit juices. No
soda.

When cessation of nursing is necessary at an early age, use of a tight


breast binder and application of ice bags may help decrease milk
production. Restriction of the mother's fluid intake and small doses of
estrogen for 12 days also may help decrease milk production.

CONTRAINDICATIONS TO BREAST-FEEDING.

Provided the mother's milk supply is ample, her diet is adequate, and she
is not infected with HIV, there are no disadvantages of breast-feeding for
the healthy term infant (see Chapter 94 ). Allergens to which the infant is
sensitized can be conveyed in the milk, but the presence of such
allergens is rarely a valid reason to stop breast-feeding. Rather, an
attempt should be made to identify the allergen and remove it from the
mother's diet.

There also are few maternal contraindications to breast-feeding.


Markedly inverted nipples may be troublesome, as may fissuring or
cracking of the nipples, but the latter can usually be avoided by
preventing engorgement. Mastitis usually can be alleviated by continued
and frequent nursing on the affected breast to keep it from becoming
engorged, but local heat applications and antibiotics may occasionally be
necessary. Acute maternal infection may contraindicate breast-feeding if
the infant does not have the same infection; otherwise, there is no need to
stop nursing unless the condition of either the mother or the infant
necessitates it. When the infant is unaffected, the breast may be emptied
and the milk given to the infant by bottle or cup. Mothers with
septicemia, active tuberculosis, typhoid fever, breast cancer, or malaria
should not breast-feed. Substance abuse and severe neuroses or
psychoses also are contraindications to breast-feeding. Infants with
galactosemia should not be breast-fed, but should receive a nonlactose-
containing formula.

(Q.135) True about pleural exudative collection is allexcept ?

(a) PMNs are predominant cells

(b) LDH < 200 U/L

(c) Pleural/serum LDH ration > 0.6


(d) Pleural/serum protein ratio >0.5

Your Response :

Correct Answer : b

Exp: B. LDH < 200 U/L Ref : NELSON 18th Ed. Chap 397

Differentiation of Pleural Fluid

TRANSUDATE EXUDATE COMPLICATED


EMPYEMA

Appearance Clear Cloudy Purulent

Cell count <1000 >1000 >5000

Cell type Lymphocytes, PMNs PMNs


monocytes

LDH <200 U/L >200 U/L >1000 U/L

Pleural/serum <0.6 >0.6 >0.6


LDH ration

Protein >3 g Unusual Common Common

Pleural/serum <0.5 >0.5 >0.5


protein ratio

Glucose[*] Normal Low Very low[*](<40 mg/dL)

pH[*] Normal (7.40 7.207.40 <7.20, chest tube


7.60) placement required

Gram stain Negative Usually >85% positive unless


positive patient received prior
antibiotics

(Q.136) Modified Walker criteria is applied for diagnosisof ?

(a) Mitochondrial diseases

(b) Glycogen disorders

(c) Lysosomal disorders

(d) Peroxisomal disorders

Your Response :
Correct Answer : a

Exp: A. Mitochondrial diseases

Modified Walker Criteria Applied to Children Referred for Evaluation of


Mitochondrial Disease

MAJOR CRITERIA MINOR CRITERIA

Clinical Clinically complete RC Symptoms compatible


encephalomyopathy[] or a with an RC defect[]
mitochondrial cytopathy
defined as fulfilling all 3 of the
following

Histology >2% ragged red fibers (RRF) in Smaller numbers of RRF,


skeletal muscle SSAM, or widespread
electron microscopy
abnormalities of
mitochondria

Enzymology Cytochrome c oxidase negative Antibody-based


fibers or residual activity of an demonstration of an RC
RC complex <20% in a tissue; defect or residual activity
<30% in a cell line, or <30% in of an RC complex 2030%
2 or more tissues in a tissue, 3040% in a
cell line, or 3040% in 2 or
more tissues

Functional Fibroblast ATP synthesis rates Fibroblast ATP synthesis


>3 SD below mean rates 23 SD below mean,
or fibroblasts unable to
grow in galactose media

Molecular Nuclear or mtDNA mutation of Nuclear or mtDNA


undisputed pathogenicity mutation of probable
pathogenicity

Metabolic One or more metabolic


indicators of impaired
metabolic function

(Q.137) Drug of choice in dermatitis herpetiformis

(a) Steroid

(b) Phototherapy
(c) Dapsone

(d) Methotrexate

Your Response :

Correct Answer : c

Exp: Ans C) Dapsone

Immunologically Mediated Blistering Diseases

Disease Clinical Histology Immunopathology Autoantigensa

Pemphigus Crusts and Acantholytic Cell surface Dsg1


foliaceus shallow erosions blister deposits of IgG on
on scalp, central formed in keratinocytes
face, upper superficial
chest, and back layer of
epidermis

Pemphigus Flaccid blisters, Acantholytic Cell surface Dsg3 (plus Dsg1


vulgaris denuded skin, blister deposits of IgG on in patients with
oromucosal formed in keratinocytes skin
lesions suprabasal involvement)
layer of
epidermis

Paraneoplastic Painful Acantholysis, Cell surface Plakin protein


pemphigus stomatitis with keratinocyte deposits of IgG and family members
papulosquamous necrosis and C3 on and desmosomal
or lichenoid vacuolar keratinocytes and cadherins (see
eruptions that interface (variably) similar text for details)
progress to dermatitis immunoreactants
blisters in epidermal BMZ

Bullous Large tense Subepidermal Linear band of IgG BPAG1,


pemphigoid blisters on blister with and/or C3 in BPAG2
flexor surfaces eosinophil- epidermal BMZ
and trunk rich infiltrate

Pemphigoid Pruritic, Teardrop- Linear band of C3 BPAG2 (plus


gestationis urticarial shaped, in epidermal BMZ BPAG1 in some
plaques, rimmed subepidermal patients)
by vesicles and blisters in
bullae on the dermal
trunk and papillae;
eosinophil-
extremities rich infiltrate

Linear IgA Pruritic small Subepidermal Linear band of IgA BPAG2 (see text
disease papules on blister with in epidermal BMZ for specific
extensor neutrophil- details)
surfaces; rich infiltrate
occasionally
larger, arciform
blisters

Cicatricial Erosive and/or Subepidermal Linear band of BPAG2,


pemphigoid blistering blister that IgG, IgA, and/or laminin-332, or
lesions of may or may C3 in epidermal others
mucous not include a BMZ
membranes and leukocytic
possibly the infiltrate
skin; scarring of
some sites

Epidermolysis Blisters, Subepidermal Linear band of IgG Type VII


bullosa erosions, scars, blister that and/or C3 in collagen
acquisita and milia on may or may epidermal BMZ
sites exposed to not include a
trauma; leukocytic
widespread, infiltrate
inflammatory,
tense blisters
may be seen
initially

Dermatitis Extremely Subepidermal Granular deposits Epidermal


herpetiformis pruritic small blister with of IgA in dermal transglutaminase
papules and neutrophils in papillae
vesicles on dermal
elbows, knees, papillae
buttocks, and
posterior neck

Dapsone is an effective treatment for most patients. DH responds to dapsone so


quickly (itching is significantly reduced within 23 days that this response may
almost be considered diagnostic. However, dapsone treatment has no effect on any
intestinal damage (see coeliac disease) that might be present

(Q.138) Scraping the surface of the lesions in pityriasis versicolor will accentuate the scaling.
This sign is known as
(a) Nikolsky sign

(b) Auspitz sign

(c) Coup Dongle sign

(d) Carpet tack sign

Your Response :

Correct Answer : c

Exp: Ans C) Coup Dongle sign

Scaly signs

Auspitz sign

Heinrich Auspitz (1835-1886) was the early star among Ferdinand Ritter
von Hebras pupils. But this sign was already described by number of
authors including Hebra, Robert Willan and Daniel Turner. However, the
Auspitz phenomenon is eponymously linked to him because his
extraordinary treatise on general pathology and therapeutics of the skin
was translated into English in 1885 and thereby constituted an early
harbinger of central European dermatopathology.

When the scales are completely scraped off, the stratum mucosum
(basement membrane) is exposed and is seen as a moist red surface
(membrane of Bulkeley) through which dilated capillaries at the tip of
elongated dermal papillae are torn, leading to multiple bleeding points .
This is a characteristic feature of psoriasis and is known as Auspitz sign.
It is attributed to parakeratosis, suprapapillary thinning of the stratum
malphighii, elongation of dermal papillae and dilatation and tortuosity of
the papillary capillaries.

However, Auspitz sign is not sensitive or specific for psoriasis. Also it is


not seen in inverse psoriasis; pustular, erythrodermic psoriasis; guttate
psoriasis. Not specific because it is also seen in nonpsoriatic scaling
disorders, including Dariers disease and actinic keratosis.

Carpet tack sign (cats tongue sign, tin tack sign)

In DLE, characteristic lesions are well-defined erythematous plaques


with partially adherent scales entering a patulous follicle. When the scale
is removed, its undersurface shows horny plugs that had occupied
follicles. This is called the carpet tack or tintack sign.

However, carpet tack sign is not diagnostic of DLE. It is also seen in


seborrheic dermatitis and pemphigus foliaceous. But in DLE, on removal
of scale, bleeding may be seen due to adherent scales unlike in
pemphigus foliaceous/seborrheic dermatitis, where the scales are loose.

Scratch sign (coup dongle sign, besniers sign, stroke of the nail)

Pityriasis versicolor is characterized by asymptomatic hypopigmented or


hyperpigmented macules and patches and produces fine scales
(branny/furfuraceous). Often the scale is not visible. An important
diagnostic clue may be the loosing of barely perceptible scale with a
fingernail, which is called as the scratch sign . This sign may be negative
if patient has taken recent bath or in case of treated lesion, in which case,
only hypopigmentation persists.

(Q.139) Which of the following can lead to ear lobule deformity, loss of nails, resorption of
distal phalanges?

(a) Tuberculoid leprosy

(b) Lepromatous leprosy

(c) Indeterminate leprosy

(d) Any of the above

Your Response :

Correct Answer : b

Exp: b) lepromatous leprosy

Leprosy type Features

Tuberculoid Can be either one large red patch with well-defined


raised borders or

large hypopigmented asymmetrical spot

Lesions become dry and hairless

Loss of sensation may occur at site of some lesions

Tender, thickened nerves with subsequent loss of


function are common

Spontaneous resolution may occur in a few years or


it may progress to orderline or rarely lepromatous
borderline or rarely lepromatous types

Borderline Similar to tuberculoid type except that lesions are


tuberculoid smaller and more
numerous

Disease may stay in this stage or convert back to


tuberculoid form, or

progress

Borderline Numerous, red, irregularly shaped plaques


borderline
Sensory loss is moderate

Disease may stay in this stage, improve or worsen

Borderline Numerous lesions of all kinds, plaques, macules,


lepromatous papules and nodules. Lesions looking like inverted
saucers are common

Hair growth and sensation are usually not impaired


over the lesions

Lepromatous Early nerve involvement may go unnoticed

Numerous lesions of all kinds, plaques, macules,


papules and nodules

Early symptoms include nasal stuffiness, discharge


and bleeding, and

swelling of the legs and ankles

Left untreated, the following problems may occur:

Skin thickens over forehead (leonine facies),


eyebrows and

eyelashes are lost, nose becomes misshapen or


collapses, ear lobes thicken, upper incisor teeth fall
out

Eye involvement causing photophobia (light


sensitivity),

glaucoma and blindness

Skin on legs thickens and forms ulcers when


nodules break down

Testicles shrivel causing sterility and enlarged


breasts (males)

Internal organ infection causing enlarged liver and


lymph nodes
Voice becomes hoarse due to involvement of the
larynx

Slow scarring of peripheral nerves resulting in nerve


thickening

and sensory loss. Fingers and toes become deformed


due to painless repeated trauma.

(Q.140) Which of the following is the most common cause of metal allergy due to jewellery?

(a) Silver

(b) Gold

(c) Iron

(d) Nickel

Your Response :

Correct Answer : d

Exp: d) nickel

jewellery allergy is a common cause of contact allergic dermatitis. Most


jewelry allergy is caused by the metal nickel which is used in the
manufacture of precious metal alloys.

(Q.141) Suprabasal acantholytic blisters are seen in

(a) Pemphigus vulgaris

(b) Dermatitis herpetiformis

(c) Bullous pemphigoid

(d) Pemphigus foliaceus

Your Response :

Correct Answer : a

Exp: a) Pemphigus vulgaris

This group of diseases is usually classified according to the location and


way of origin of the blisters:

Subcorneal: in pemphigus foliaceus, staphylococcal impetigo...

Intraepidermal: within stratum spinosum; further classification


according to the way of origin:

Spongiotic: caused by excessive spongiosis; usually some inflammatory


cells are present. Examples: allergic dermatitis, dyshidrotic dermatitis

Cell ballooning: often caused by viruses (herpes)

Acantholytic: cellular cohesivness is lost, stratum spinosum


desintegrates: pemphigus vulgaris, Darier's disease...

Subepidermal:

Epidermolysis bullosa, porphyria cutanea tarda, lichen sclerosus et


atrophicus, dermatitis herpetiformis Duhring and others. Subepidermal
bullae appear after epidermal necrosis (vascular occlusion, phlegmone
etc.).

(Q.142) Exclamation mark alopecia is seen in

(a) Androgenic alopecia

(b) Telogen effluvium

(c) Anagen effluvium

(d) Alopecia areata

Your Response :

Correct Answer : d

Exp: d) alopecia areata

The characteristic patch of alopecia areata is usually round or oval, and is


completely bald and smooth. "Exclamation-mark" hairs may be seen at
the margin of the patch. These are broken, short hairs that taper at the
base. Pulling slightly on these hairs causes them to fall out.

(Q.143) Most confirmatory sign of endotracheal intubation is

(a) Chest rise

(b) Auscultation

(c) Spirometry

(d) Capnography

Your Response :
Correct Answer : d

Exp: Ans D) Capnography

Methods Used to Confirm Endotracheal Tube Placement

Direct visualization

Observation of chest movement

Auscultation of breath sounds

Absence of epigastric sounds with respiration

Presence of an exhaled tidal volume

Reservoir bag compliance

Endotracheal cuff maneuvers

Absence of air escape

Tube condensation with exhalation

Absence of gastric contents within the tube Measured

Pulse oximetry

End-tidal carbon dioxide measurement

Esophageal detector device

ETCO2 monitoring is a very reliable and rapid method of confirmation.


It can be positive with esophageal intubation in early stages as air can be
forced into the esophagus during mask ventilation. CO2 may not appear
despite correct placement in cases with severe bronchospasm and
conditions in which there is no pulmonary blood flow. ETCO2 is the
gold standard method in detecting the correct placement of endotracheal
tube

(Q.144) The pin index safety system of oxygen is

(a) 1, 5

(b) 2, 5

(c) 3, 5

(d) 4, 5

Your Response :
Correct Answer : b

Exp: Ans B) 2, 5

Gas Color US Service Capacity L Pin Position


(internatal) Pressure psi

Oxygen green (white) 1,900 660 2-5

Nitrous blue (blue) 745 1,590 3-5


Oxide

Air yellow(black & 1,900 625 1-5


white)

(Q.145) An Aplastic crisis and sequestration crisis during Anaesthesia can occur in which
one of the following disease?

(a) Thalassemia

(b) Sickle cell disease

(c) Aplastic anaemia

(d) Glucose 6 phosphate dehydrogenase deficiency

Your Response :

Correct Answer : b

Exp: Ans B) Sickle cell disease

Acute crises in sickle cell disease may occur spontaneously or


precipitated by

- Infection - Dehydration - Hypoxia - Sedatives and local


anaesthetics

(Q.146) Which anaesthetic causes diffusion hypoxia?

(a) N2O

(b) Halothane

(c) Ether

(d) Etoxide

Your Response :
Correct Answer : a

Exp: a) N2O

The concentration effect

The concentration effect describes how the concentration of the gas in


the remaining alveolar volume can increase after some of the gas has
been transferred into the blood. Thus, the concentration effect states that
with higher inspired concentrations of an anaesthetic, the rate of rise in
arterial tension is greater.

N2O has a low blood:gas partition coefficient and therefore a rapid onset
and offset of action. N2O is about 20 times more soluble than O2 and
N2. During induction the volume of N2O entering the pulmonary
capillaries is greater than the N2 leaving the blood and entering the
alveolus. As a result the volume of the alveolus decreases, thereby
increasing the fractional concentration of the remaining gases. This
process augments ventilation as bronchial and tracheal gas is drawn into
the alveolus to make good the diminished alveolar volume.

The second gas effect

The second gas effect usually refers to nitrous oxide combined with an
inhalational agent. Because nitrous oxide is not soluble in blood, its'
rapid absorption from alveoli causes an abrupt rise in the alveolar
concentration of the other inhalational anaesthetic agent.

Diffusion Hypoxia

First described by Fink in 1955, this is effectively the reverse of the


above. The elimination of a poorly soluble gas, such as N2O, from the
alveoli may proceed at as greater rate as its uptake, (The volume of N2O
entering the alveolus is greater than the volume of N2 entering the
pulmonary capillary blood.), thereby adding as much as 1 l/min to
alveolar air. This gas effectively dilutes alveolar air, and available
oxygen, so that when room air is inspired hypoxia may result. This is
usually only mild and rarely clinically significant although this may
occur with any anaesthetic agent, its magnitude is insignificant unless an
insoluble agent, such as nitrous oxide, has been inhaled for some time.

(Q.147) Ebb phase ischaracterized by all except:

(a) Hypothermia

(b) Leucocytosis

(c) Low BMR


(d) Lactic acidosis

Your Response :

Correct Answer : b

Exp: b) leukocytosis

Phases of Metabolic Stress

o Ebb phase: metabolic stress begins; energy is conserved immediately


following injury

o Acute flow phase: energy requirements increase, sometimes markedly

o Adaptive flow phase: healing process begins

Ebb Phase of Metabolic Stress

o Begins immediately after injury; may last up to 24 hrs

o Metabolic rate and energy requirements decrease

o Blood pressure, cardiac output, and body temperature may drop

o Enteral feeding usually not initiated until patient is hemodynamically


stable

Acute Flow Phase

o Cardiac output and blood pressure increase

o Fat stores and muscles are catabolized, yielding energy

o Protein in muscle tissue breaks down, providing amino acids for


gluconeogenesis in

the liver

o Stress hormones (glucagon, catecholamines, and glucocorticoids) are


rapidly produced

o Levels of circulating glucose rise

(Q.148) True about the oculocardiac reflex includes following except:

(a) Mediated by trigeminal route

(b) Preoperative glycopyrrolate is not helpful for prevention

(c) Induced by pressure on eye ball


(d) Mediated by traction on medial rectus

Your Response :

Correct Answer : b

Exp: b) preoperative glycopyrolate is not helpful for prevention

Oculocardiac reflex is one of the trigemino-vagal reflexes, and is


frequently observed during anesthesia for pediatric strabismus surgery.
The reflex is greatly exaggerated in the presence of hypoventilation,
hypoxemia and acidosis. The reflex is important as a cause of cardiac
arrest during eye surgery. Several related reflexes are known, such as
blepharocardiac reflex, oculorespiratory reflex, and the sudden infant
death syndrome. Although the reflex can be prevented by a retrobulbar
block or the administration of parasympatholytic drugs, well conducted
anesthesia and cooperation with the surgeon is much more important

(Q.149) Ayers T piece is

(a) Mapleson A circuit

(b) Mapleson B circuit

(c) Mapleson D circuit

(d) Mapleson E circuit

Your Response :

Correct Answer : d

Exp: D) Mapelson E circuit

Mapleson A - the Magill and Lack circuits

Mapleson B and C - Rebreathing of exhaled gases occurs even when


very high fresh gas flow rates are used, since inspiration is taken from
the same space into which the previous breath was expired. These are
unsatisfactory for anaesthesia, but may be used in emergency for
resuscitation.

Mapleson D - the modified Bain circuit.

Mapleson E - Ayre's T piece and the Bain circuit.

"Mapleson F" - not originally classified by Mapleson, but is used to refer


to Jackson-Rees' modification of Ayre's T-piece.

The Humphrey ADE is a circuit that provides the ability to switch


between the Mapleson A, D and E arrangements.

(Q.150) Hallucinations experienced outside the limits of ones sensory fields are

(a) Functional hallucinations

(b) Extracampine hallucinations

(c) Pseudo hallucination

(d) Reflex hallucinations

Your Response :

Correct Answer : b

Exp: Ans B) Extracampine hallucinations

The stimulus can be perceived as the corresponding object, but not


accurately. For example an object could be perceived as being the wrong
size; this is called micropsia or macropsia.

The stimulus is perceived as an object, but not corresponding to the


source. That is to say, both the stimulus and object are present, but
different from each other. This is an illusion

There is no stimulus, but a perception occurs. This is a hallucination.

There is a stimulus, but no perception occurs. This is a negative


hallucination.

Hallucinations can occur in any modality and there are many different
types:

Elementary hallucinations are the simplest kind and they are unstructured
hallucinations and bear no relation to anything in the natural world.

Auditory hallucinations often occur with psychiatric illness, and auditory


hallucinations of voices are one of the first rank symptoms of
schizophrenia. Visual hallucinations on the other hand are much more
common with organic illness and are very uncommon in
schizophrenia. Organic causes for hallucinations include occipital lobe
tumours, post concussional states, hepatic failure and dementia.

Elderly patients with normal consciousness and no brain pathology, but


with reduced visual acuity due to ocular problems experience vivid,
distinct formed hallucinations, often of men wearing hats. This is called
Charles Bonnet syndrome. Lilliputian hallucinations involve seeing tiny
people or animals. These can occur with alcohol withdrawal.

Autoscopic hallucinations are the experience of seeing oneself. This is


different from an out of body experience, as with the latter the person
sees the world and his own body from a vantage point that is other than
his physical body. In autoscopy, the person remains in their own body.

Extracampine hallucinations occur outside the field of normal


perception. An example of this would be hearing someone discussing
you down the shops which are a mile away.

Functional Hallucinations is where an external stimulus provokes


hallucination, but both hallucination and stimulus are in the same
modality but individually perceived. On the other hand, Reflex
hallucinations are when hallucinations in one modality are provoked by a
stimulus in another modality.

Formication is a type of haptic hallucination where there is the sensation


of animals crawling under the skin. This is seen in cocaine
intoxication. A character in the beginning of the film A Scanner
Darkly has a similar problem.

A pseudohallucination is like a hallucination, but lacks the quality of a


perception. It is a form of vivid imagery. If someone feels that they are
hearing voices in their head, this is a pseudohallucination as it does not
have the same qualities as a normal perception.

Synaesthesia is the perceiving of a stimulus in one modality in a different


modality, for example, hearing the colour red. This can happen on
taking LSD

Hypnagogic and hypnopompic hallucinations are hallucinations on


falling asleep and waking up, respectively. They may be normal
phenomena and are particularly seen in narcolepsy

(Q.151) Following diagnostic test can be used in identifying person with alcohol related
disorders EXCEPT

(a) SGOT, SGPT

(b) Mean Corpuscular Volume (MCV)

(c) Hemoglobin

(d) Gamma Glutamyl Transferase (GGT)

Your Response :

Correct Answer : c

Exp: Ans C) Hemoglobin

There liver enzymes are commonly used in screening for ALDs:


aspartate amino transferase (AST),alanine amino transferase (ALT), and
gamma glutamyl transferase (GGT). An AST : ALT ratio of >2 in a
patient with liver disease diagnosed on clinical grounds is highly
sugestive of alcohol as a cause. GGT adds little information to AST and
alkaline phosphatase (ALP) in the diagnosis of liver disease, however it
can help to locate the origin of elevated ALP to the liver. GGT is more
sensitve to enzyme induction by alcohol than AST or ALT in excesive
drinkers, but can also be elevated due to liver damage. False positve
results can be due to enzyme-inducing drugs.

Mean cell volume is commonly used as a marker for excessive drinking


in screening. The precise mechanism for macrocytosis (increased
corpuscular volume) in alcohol misuse is unclear, but is believed to be
related to a toxic effect of alcohol on the bone marow, leading to the
release of immature and abnormaly large cells.

Carbohydrate-deficient transferin (CDT) is an isoform of transfferin and,


in comparison to liver enzymes, appears to be relatively unaffected by
liver disease. CDT is more sensitive than GGT in detecting relapse in
alcohol-dependent patients in treatment than as a screening method to
detect ALDs in moderately heavy drinkers

(Q.152) All of the following are true about Lithium (Li), EXCEPT

(a) Propranolol is effective in Li induced tremors

Li is drug of choice for treatment of mania in 1st trimester of


(b)
pregnancy

Li has been effective in treatment of depression associated with


(c)
Bipolar disorder

(d) Most common adverse renal effect of Li is polyuria

Your Response :

Correct Answer : b

Exp: Ans B) Li is drug of choice for treatment of mania in 1st trimester


of pregnancy

Lithium is used primarily for bipolar disorder.It is sometimes used when


other treatments are not effective in a number of other conditions
including: major depression, schizophrenia, and some psychiatric
disorders in children In mood disorders, of which bipolar is one, it
decreases the risk of suicide This benefit is not seen with other
medications.

Lithium treatment was previously considered to be unsuitable for


children; however, more recent studies show its effectiveness for
treatment of early-onset bipolar disorder in children as young as eight.
The required dosage is slightly less than the toxic level, requiring blood
levels of lithium to be monitored closely during treatment. High doses of
haloperidol, fluphenazine, or flupenthixol may be hazardous when used
with lithium; irreversible toxic encephalopathy has been reported.

Very Common (>10% incidence) adverse effects of lithium


include: Leukocytosis, Polyuria/polydypsia , Dry mouth, tremor ,
Headache, Decreased memory, Confusion, Muscle weakness , ECG
changes usually benign changes in T waves. Nausea ,Vomiting ,
Diarrhea , Constipation .

Lithium is also a teratogen, causing birth defects in a small number of


newborn babies.Case reports and several retrospective studies have
demonstrated possible increases in the rate of a congenital heart defect
known as Ebstein's anomaly, if taken during a woman's pregnancy.

(Q.153) Personality changes are commonly seen with lesion of which of thefollowing ?

(a) Limbic system

(b) Temporal lobe

(c) Frontal lobe

(d) Occipital lobe

Your Response :

Correct Answer : c

Exp: Ans C) Frontal lobe

The frontal lobe of the brain plays a key role in higher mental functions
such as motivation, planning, social behaviour, and speech production. A
frontal lobe syndrome can be caused by a range of conditions including
head trauma, tumours, degenerative diseases, neurosurgery and
cerebrovascular disease.

(Q.154) Delusion is an abnormal variant of whichcondition?

(a) Perception

(b) Thinking

(c) Memory

(d) Intelligence
Your Response :

Correct Answer : b

Exp: Ans B) Thinking

A delusion is a belief held with strong conviction despite superior


evidence to the contrary.As a pathology, it is distinct from a belief based
on false or incomplete information, confabulation, dogma, illusion, or
other effects of perception.

(Q.155) In a child with ADHD, while treating with drugs, the first choice of medicine is
usually

(a) Donepezil

(b) Atomoxetine

(c) Clozapine

(d) Methylphenidate

Your Response :

Correct Answer : d

Exp: Methylphenidate

Methylphenidate (trade names Concerta, Methylin, Ritalin, Equasym


XL, Quillivant XR) is a substituted phenethylamine and psychostimulant
drug used for the treatment of attention-deficit hyperactivity disorder
(ADHD), postural orthostatic tachycardia syndrome and narcolepsy.
Methylphenidate has been studied and researched for over 50 years and
has a very good efficacy and safety record for the treatment of ADHD

(Q.156) The commonest psychiatric sequelae following traumatic brain injury is

(a) Psychotic disorder

(b) Mood disorder

(c) Anxiety disorder

(d) Substance use disorder

Your Response :

Correct Answer : b
Exp: Ans B) Mood disorder

Common new disorders following head injury were major depression


(27%), alcohol abuse or dependence (12%), panic disorder (8%), specific
phobia (8%) and psychotic disorders (7%). Depression occurs more
frequently with left dorsolateral frontal and left basal ganglia lesions.
The mechanism of depression following head injury is probably due to
disruption of biogenic amine-containing neurons as they pass through the
basal ganglia or frontal-subcortical white matter.

(Q.157) Which condition is called as Pseudo-dementia?

(a) Confabulations

(b) Normal aging

(c) Delirium

(d) Depression

Your Response :

Correct Answer : d

Exp: Ans D) Depression

Pseudodementia is a phenotype approximated by a wide variety of


underlying disorders . Data indicate that some of the disorders that can
convert to a pseudodementia-like presentation include depression
(mood), schizophrenia, mania, dissociative disorders, Ganser syndrome,
conversion reaction, and psychoactive drugs . Although the frequency
distribution of disorders presenting as pseudodementia remains unclear,
what is clear is that depressive pseudodementia, synonymously referred
to as depressive dementia or major depression with depressive dementia ,
represents a major subclass of the overarching category of
pseudodementia

(Q.158) Hallucinations occurring while falling asleep are called as

(a) Hypnogogic hallucination

(b) Reflex hallucination

(c) Hypnopompic hallucination

(d) Functional hallucination

Your Response :
Correct Answer : a

Exp: a) Hypnogogic hallucination

These hallucinations occur just before falling asleep, and affect a high
proportion of the population (in one survey 37% of the respondents
experienced them twice a week. The hallucinations can last from seconds
to minutes; all the while, the subject usually remains aware of the true
nature of the images. These may be associated with narcolepsy.
Hypnagogic hallucinations are sometimes associated with brainstem
abnormalities, but this is rare

(Q.159) Flexion, Adduction, Internal Rotation deformity at hip is seen in

(a) Fracture neck of femur

(b) Posterior dislocation of hip

(c) Inter trochanteric fracture

(d) Anterior dislocation of hip

Your Response :

Correct Answer : b

Exp: B) Posterior dislocation of hip

Simple vs. Complex

simple

pure dislocation without associated fracture

complex

dislocation associated with fracture of acetabulum or proximal femur

Anatomic classification

posterior dislocation (90%)

occur with axial load on femur, typically with hip flexed and adducted

axial load through flexed knee (dashboard injury)

position of hip determines associated acetabular injury

increasing flexion and adduction favors simple dislocation

associated with
osteonecrosis

posterior wall acetabular fracture

femoral head fractures

sciatic nerve injuries

ipsilateral knee injuries (up to 25%)

anterior dislocation

associated with femoral head impaction or chondral injury

occurs with the hip in abduction and external rotation

inferior vs. superior

hip extension results in a superior (pubic) dislocation

flexion results in inferior (obturator) dislocation

(Q.160) Test to diagnose Anterior Cruciate Ligament (ACL) injury is

(a) Valgus test

(b) Varus test

(c) Lachman test

(d) Posterior Drawer test

Your Response :

Correct Answer : c

Exp: This is a variant of the anterior drawer test, in which the examination is
carried out w/ the knee in 15 deg of flexion, and external rotation
(relaxes IT band);

- Definition of Severity:

- clinical exam:

- mild: 0 to 5 mm laxity (greater than the uninvolved side)

- moderate: 6 to 10 mm laxity (greater than the uninvolved side)

- severe: 11 to 15 mm laxity (greater than the uninvolved side)

- when exam reveals such a large amount of translation


always consider concomitant MCL tear / meniscal tear;
- KT-1000:

- greater than 11 mm of anterior translation of the injured side or


greater than 3 mm of side to side difference (delta lachman) indicates
ACL tear;

- following ACL reconstruction, laxity should be less than 3 mm


compared to opposite side

Patellofemoral evaluation

- Quadriceps Femoris Muscle Angle (Q Angle) - Patella tracking


with quadriceps contraction

- Evaluate for smoothness of motion and crepitation - Patellar


Apprehension Test

- Evaluates for Patella Subluxation

Anterior Cruciate Ligament (ACL) Stability Tests

- Lachman Test (most sensitive) - Knee Anterior Drawer Test -


Pivot Shift Test (MacIntosh Test)

Posterior Cruciate Ligament (PCL) Tests

- Knee Posterior Drawer Test - PCL Sulcus Test - PCL Sag


Test

Collateral ligament evaluation

Knee Valgus Stress Test (Medial collateral ligament)

Knee Varus Stress Test (Lateral collateral ligament)

Meniscus Evaluation

- McMurray's Test - Apley's Compression Test and Apley's


Distraction Test

- Knee Bounce Test - Thessaly Test

- Inability to fully extend knee may suggest "bucket-handle" meniscal


tear

- Joint line tenderness is 76% sensitive for meniscal tear, but not specific

(Q.161) Sacroiliac joint involvement is seen in

(a) Rheumatoid Arthritis

(b) Ankylosing spondylitis


(c) Reiters syndrome

(d) Psoriatic arthritis

Your Response :

Correct Answer : b

Exp: Ankylosing Spondylitis

Chronic inflammatory disease of unknown etiology primarily affecting


spine

Most common spondyloarthropathy

Age-young adults

15-35 years

Mostly male

M:F = 4:1

Mostly Caucasian

Caucasians to Blacks = 3:1

Location

Axial skeleton and large, usually central, appendicular joints

Sacroiliac joint involvement

Hallmark of disease

Only synovial portion of SI joint is involved

Inferior and anterior portion of joint

Other enthesopathies like DISH can cause bridging of upper, non-


synovial part of joint

Usually site of initial involvement

Bilaterally symmetric

Widened with erosions at first

Then ankylosis

Spine

Usually begins at either thoracolumbar or lumbosacral junctions


Extends symmetrically without skip areas

Reiters and psoriasis characteristically are asymmetric and have skip


areas

Marginalsyndesmophyteformation = thin vertical dense spicules bridging


the vertebral bodies

Ossification of outer fibers of annulus fibrosus

Not anterior longitudinal ligament

Trolley-track signon AP view = central line of ossification (supraspinous


and interspinous ligaments) with two lateral lines of ossification
(apophyseal joints)

Bamboo spineon AP view = undulating contour due to syndesmophytes

Prone to fracture resulting in pseudarthrosis

Straightening /squaringof anterior vertebral margins

Osteitis of anterior corners

Reactive sclerosis of corners of vertebral bodies =shiny-corner sign

Symmetric erosions of laminar and spinous process at level of lumbar


spine

Apophyseal and costovertebral ankylosis

Periostealwhiskering

(Q.162) Treatment of choice in Intra-capsular fracture neck femur in 35 yr. old male,
presenting after 3 days is

(a) Plaster and Cast

(b) Hemiarthroplasty

(c) Closed Reduction and Internal fixation by cancellous screws

(d) Closed Reduction and internal fixation by Austin Moores pins

Your Response :

Correct Answer : c

Exp: From treatment point of view following points are considered.

1. Age of the patient:Based on the age the following groups were made.
a) 1-16 years before the closure of upper femoral epiphysis b) 16-50
years young adults

c) 50-60 years middle age group d) above


60 years (old age)

2. Site of fracture

a) Sub-capital b) Transcervical c) Basal type

3. Displacement of fragments : For displacement of fragments Garden's


classification/staging was discussed I, II, III, IV.

A simplification of this adopted: i) Undisplaced fracture. ii)


Displaced fracture

4. Duration of fracture : 1-21 days - fresh, more than 21 days - neglected


fracture

FRESH FRACTURE

Age 16-50 years :

1. Sub-capital fracture:

Undisplaced:Internal fixation with 2-3 cancellous or cannulated screws


should be used.

Displaced:Closed reduction and internal fixation with cancellous or


cannulated screws is recommended. Abduction osteotomy and internal
fixation with modified DHS or 135 degree blade plate or double angled
blade plate converts shearing force into compression force can be used.
Closed reduction internal fixation with 2 screws and one free fibular
graft has also been used.

2. Transcervical fracture:

Undisplaced:Internal fixation with cancellous or cannulated screws


should be used.

Displaced fractures:Closed reduction and internal fixation with


cancellous or cannulated screws, 2-3 screws are used If closed reduction
fails then

1. Open reduction and internal fixation with cancellous or cannulated


screws is done.

2. Open reduction and internal fixation with screws and free fibular graft
or muscle pedicle bone graft based on quadratus femoris or sartorius or
tensor fascia femoris are useful.
3. Basal fracture:

Undisplaced:Internal fixation with DHS should be used.

Displaced fractures:Closed reduction and internal fixation with DHS or


cancellous or cannulated screws is done. If closed reduction fails then
open reduction and internal fixation with screws DHS.

(Q.163) Froments sign is positive in:

(a) Median nerve injury

(b) Ulnar nerve injury

(c) Radial nerve injury

(d) Musculocutaneous nerve injury

Your Response :

Correct Answer : b

Exp: Sign Cause

1 Ape thumb Median nerve injury

2 Claw hand Ulnar nerve injury

3 Foot drop Common peroneal nerve injury

4 Froments sign Ulnar nerve injury

5 Galleazzis sign DDH

6 Kanavels sign Infection of ulnar bursa/Tenosynovitis

7 Ludloffs sign Lesser trochanter avulsion

8 ODonoghue triad MCL, ACL, and medial meniscus injury

9 Sulcus sign Inferior shoulder dislocation

10 Tinels sign Regeneration/ improvement of nerve

(Q.164) Most commonly used tendon for tendon transfer:

(a) Plantaris longus

(b) Palmaris longus

(c) Extensor Digitorum


(d) Flexor carpi radialis

Your Response :

Correct Answer : b

Exp: n A tendon transfer is an operation is an operation in which insertion of


the tendon Of a healthy functioning muscle is moved to a new site, so
that the muscle henceforth, has a different action.

Indications:

n Muscle paralysis n Muscle imbalance (e.g., polio)

n Tendon rupture n Jones operation is commonly done tendon


transfer for radial nerve palsy.

n Palmaris longus is most commonly used tendon for tendon transfer.

(Q.165) Three point relation is used to differentiate Supracondylar fracture from fracture
of:

(a) Medial epicondyle

(b) Lateral epicondyle

(c) Elbow dislocation

(d) All of the above

Your Response :

Correct Answer : d

Exp: The Three-point relation of both epicondyles & olecranon at elbow joint
is maintained in supracondylar fracture of humerus.

(Q.166) Modified Radical neck dissection does NOT preserve

(a) Spinal accessory nerve

(b) External Jugular vein

(c) Sternocleidomastoid muscle

(d) Internal Jugular vein

Your Response :

Correct Answer : b
Exp: B. External jugular vein

Radical neck dissection (RND) refers to the removal of all ipsilateral


cervical lymph node groups extending from the inferior border of the
mandible superiorly to the clavicle inferiorly and from the lateral border
of the sternohyoid muscle, hyoid bone, and contralateral anterior belly of
the digastric muscle anteriorly to the anterior border of the trapezius
muscle posteriorly.

Modified radical neck dissection (MRND) is defined as the excision


of all lymph nodes routinely removed in a radical neck dissection
with preservation of one or more nonlymphatic structures (SAN,
IJV, SCM). The preserved nonlymphatic structures should be
specifically mentioned (eg, modified radical neck dissection with
preservation of the IJV and SCM).

A selective neck dissection refers to any neck dissection in which one or


more lymph node groups removed in RND is preserved.

The most widely accepted terminology for categorizing the lymph node
groups in the neck was originally described by head and neck surgeons at
Memorial Sloan-Kettering Cancer Center, dividing the neck into 5
regions. A sixth region was later added to characterize the lymph nodes
in the anterior neck (see Table 1). Levels I, II, and V have also been
divided into sublevels, and the anatomic and radiographic boundaries of
each level and sublevel have been clarified. Lymph Node Groups of the
Neck

Level Lymph Node Group

I Submental and submandibular


nodes

II Upper jugular nodes

III Middle jugular nodes

IV Lower jugular nodes

V Posterior triangle nodes

VI Anterior compartment lymph


nodes

No formalized indications have been created for modified radical neck


dissection (MRND), and indications vary widely from one clinician to
another. In general, MRND is indicated whenever preservation of the
spinal accessory nerve (SAN), internal jugular vein (IJV), or
sternocleidomastoid muscle (SCM) is possible without compromising
oncologic efficacy, and when a selective neck dissection would not be
considered an adequate oncologic procedure. An en bloc approach
should be pursued, and nodal metastases encapsulated by reactive
connective tissue must be resected atraumatically.

For patients who are clinically staged N0 or N1, a selective neck


dissection or MRND would be appropriate. In patients with N2 disease,
MRND is reasonable if any of the aforementioned nonlymphatic
structures can be safely preserved. Many patients with N3 disease (nodal
metastases > 6 cm) require RND, but MRND can be considered when
dissection is feasible.

MRND is contraindicated whenever preservation of the nonlymphatic


structures of the neck would compromise complete resection of the
cervical metastatic disease. Because outcome data are limited, the precise
role of MRND remains undefined, especially in N2 and N3 nodal
disease. Although a growing body of evidence supports the performance
of selective neck dissection in carefully selected patients with clinically
positive nodal disease, no prospective randomized clinical trials have
been conducted. Consequently, the indications for MRND and selective
neck dissection in these patients also remains poorly defined.

(Q.167) The loss of tension in vocal cord with diminished power and range of voice occurs in

(a) Unilateral recurrent laryngeal nerve palsy

(b) Bilateral recurrent laryngeal nerves paralysis

(c) Ext. branch of superior laryngeal nerve palsy

(d) Tracheomalacia

Your Response :

Correct Answer : c

Exp: Ext. branch of superior laryngeal nerve palsy

Nerve involvement Cord position/ Clinical Features


Laryngeal findings

Unilateral Recurrent Incomplete paralysis One third asymptomatic


laryngeal nerve Median
paralysis Mild change invoice
Complete paralysis
Paramedian No aspiration / obstruction

Voice improves gradually


due to compensation by
healthy cord
Bilateral Recurrent Median or paramedian Inadequate airway
laryngeal nerve Dyspneoa, Stridor
paralysis worsens on exertion

Voice is good

Unilateral Superior Ant. commissure Weak voice


laryngeal nerve rotated to healthy
paralysis side Askew position Unable to raise pitch

Shortening of cord Occaisional aspiration


with loss of tension

Flapping of cord

Bilateral Superior Weak and Husky voice


laryngeal nerve
paralysis Aspiration

Unilateral Combined Ipsilateral cord in Hoarseness


Complete Paralysis Cadaveric position(3.5
(Recurrent and mm from midline) Aspiration
Superior laryngeal
nerves) Glottic incompetance Ineffective cough

Bilateral Combined Bilateral cords in Aphonia


Complete Paralysis cadaveric position
(Recurrent and Aspiration
Superior laryngeal Total anaesthesia of
larynx Inability to cough
nerves)
Bronchopneumonia

(Q.168) Melanocytes migrates from the neural crest to basal epidermis during

(a) Neonatal life

(b) Childhood

(c) Embryogenesis

(d) After birth

Your Response :

Correct Answer : c

Exp: Melanocytes are pigment-producing dendritic cells derived from the


neural crest. During embryogenesis, pluripotent neural crest cells
develop into lineage-restricted melanocyte precursors (melanoblasts) as
they migrate along the dorsolateral pathway between the somite and
overlying ectoderm to the dermis, eventually reaching their final
destinations in the epidermis and hair follicles. Cutaneous melanocytes
can also arise from Schwann cell precursors located along nerves in the
skin, which originate from the neural crest via the ventral pathway.5In
addition, melanoblasts migrate to the uveal tract of the eye (choroid,
ciliary body and iris), the inner ear (stria vascularis of the cochlea) and
the leptomeninges (pia mater). This distribution of melanocytes accounts
for the melanocytosis and risk of developing melanoma in the eye (e.g.
choroid) and leptomeninges that is seen in patients with nevus of Ota and
the occurrence of neurocutaneous melanocytosis in patients with large
and/or multiple congenital melanocytic nevi.

(Q.169) In operation of pancreaticoduodenectomy for caPancrease; one of the anastomons is


known to leak more frequently

(a) Choledochojejunostomy

(b) Gastrojejunostomy

(c) Pancreaticojejunostomy

(d) None of the above

Your Response :

Correct Answer : c

Exp: Pancreatic anastomotic leakage (Leak) is the most common major


complication after pancreaticoduodenectomy (PD).Some surgeons
use stents, glue to seal the anastomosis, or octreotide to decrease
pancreatic secretions. No matter what combination of these techniques is
used, the pancreatic leakage rate is always about 10%. Therefore, the
choice of techniques depends more on the surgeon's personal experience.
Traditionally, most surgeons place drains around the pancreatic and
biliary anastomoses because disruption of the
pancreaticojejunostomy cannot be avoided in one out of 10 patients.
This complication can lead to the development of an upper abdominal
abscess or can present as an external pancreatic fistula. Usually, a pure
pancreatic leak is controlled by the drains and will eventually seal
spontaneously. Combined pancreatic and biliary leaks are cause for
concern because bile will activate the pancreatic enzymes. In its most
virulent form, disruption leads to necrotizing retroperitoneal infection,
which can erode major arteries and veins of the upper abdomen,
including the exposed portal vein and its branches or the stump of the
gastroduodenal artery. Impending catastrophe is often preceded by a
small herald bleed from the drain site. Depending on the clinical
situation, such an event is an indication to perform an angiogram or
return the patient to the operating room to widely drain the
pancreaticojejunostomy and to repair the involved blood vessel. Open
packing may be necessary to control diffuse necrosis and infection. Some
studies have questioned the practice of routine drain placement after
pancreaticoduodenectomy. These studies indicated that most pancreatic
leaks can be managed with percutaneous drainage

(Q.170) The initial treatment of choice for bleeding oesophageal varices is

(a) TIPSS

(b) Surgical shunt

(c) Surgicaldevascularization

(d) Endoscopic Banding/Sclerotherapy

Your Response :

Correct Answer : d

Exp: Management of Acute Variceal Bleeding

Patients with acute variceal hemorrhage should be admitted to an ICU


for resuscitation and management. Blood resuscitation should be
performed carefully to a hemoglobin level of approximately 8 g/dL.
Overreplacment of packed red blood cells and the overzealous
administration of saline can lead to both rebleeding and increased
mortality. Administration of fresh-frozen plasma and platelets can be
considered in patients with severe coagulopathy. Use of recombinant
factor VIIa has not been shown to be more beneficial than standard
therapy and therefore is not recommended at this time. Cirrhotic patients
with variceal bleeding have a high risk of developing bacterial infections,
which are associated with rebleeding and a higher mortality rate. The use
of short-term prophylactic antibiotics has been shown both to decrease
the rate of bacterial infections and to increase survival. Therefore, their
use is recommended, and ceftriaxone 1 g/day IV is often given.
Pharmacologic therapy for the variceal hemorrhage can be initiated as
soon as the diagnosis of variceal bleeding is made. Vasopressin,
administered IV at a dose of 0.2 to 0.8 units/min, is the most potent
vasoconstrictor. However, its use is limited by its large number of side
effects, and it should be administered for only a short period of time at
high doses to prevent ischemic complications. Somatostatin and its
analogue octreotide (initial bolus of 50 microg IV followed by
continuous infusion of 50 microg/h) also cause splanchnic
vasoconstriction. Octreotide has the advantage that it can be administered
for 5 days or longer, and it is currently the preferred pharmacologic agent
for initial management of acute variceal bleeding. In addition to
pharmacologic therapy EGD should be carried out as soon as
possible and EVL should be performed. This combination of
pharmacologic and EVL therapy has been shown both to improve
the initial control of bleeding and to increase the 5-day hemostasis
rate.

Even when aggressive pharmacologic and endoscopic therapies are


initiated and these treatment options are maximized, 10 to 20% of
patients with variceal bleeding will continue to bleed. Shunt therapy,
with either surgical shunts or TIPS, has been shown to control
refractory variceal bleeding in >90% of treated individuals. Shunt
surgery usually is considered only in patients with preserved hepatic
function (i.e., CTP class A); TIPS is used in patients with
decompensated liver disease (i.e., CTP class B or C). However, the
use of these treatment options is dependent on local expertise.

Balloon tamponade using a Sengstaken-Blakemore tube will control


refractory variceal bleeding in >80% of patients. However, its
application is limited due to the potential for complications, which
include aspiration and esophageal perforation. Therefore, use of a
Sengstaken-Blakemore tube should be limited to short-term therapy
(<24 hours) in those patients awaiting definitive care.

(Q.171) Commonest malignancy in thyroid gland seen is

(a) Anaplastic carcinoma

(b) Follicular carcinoma

(c) Papillary carcinoma

(d) Medullary carcinoma

Your Response :

Correct Answer : c

Exp: Papillary Carcinoma

Papillary carcinoma accounts for 80% of all thyroid malignancies in


iodine-sufficient areas and is the predominant thyroid cancer in
children and individuals exposed to external radiation. Papillary
carcinoma occurs more often in women, with a 2:1 female-to-male ratio,
and the mean age at presentation is 30 to 40 years. Most patients are
euthyroid and present with a slow-growing painless mass in the neck.
Dysphagia, dyspnea, and dysphonia usually are associated with locally
advanced invasive disease. Lymph node metastases are common,
especially in children and young adults, and may be the presenting
complaint. "Lateral aberrant thyroid" almost always denotes a cervical
lymph node that has been invaded by metastatic cancer. Suspicion of
thyroid cancer often originates through physical examination of patients
and a review of their history. Diagnosis is established by FNAB of the
thyroid mass or lymph node. Once thyroid cancer is diagnosed on
FNAB, a complete neck ultrasound is strongly recommended to evaluate
the contralateral lobe and for lymph node metastases in the central and
lateral neck compartments. Distant metastases are uncommon at initial
presentation, but may ultimately develop in up to 20% of patients. The
most common sites are lungs, followed by bone, liver, and brain.

(Q.172) The part of colon worst affected by ischaemic colitis is

(a) Right colon

(b) Left colon

(c) Hepatic flexure

(d) Splenic flexure

Your Response :

Correct Answer : d

Exp: Ischemic Colitis

Intestinal ischemia occurs most commonly in the colon. Unlike small


bowel ischemia, colonic ischemia is rarely associated with major arterial
or venous occlusion. Instead, most colonic ischemia appears to result
from low flow and/or small vessel occlusion. Risk factors include
vascular disease, diabetes mellitus, vasculitis, and hypotension. In
addition, ligation of the inferior mesenteric artery during aortic surgery
predisposes to colonic ischemia. Occasionally, thrombosis or embolism
may cause ischemia. Although the splenic flexure is the most common
site of ischemic colitis, any segment of the colon may be affected. The
rectum is relatively spared because of its rich collateral circulation.

Signs and symptoms of ischemic colitis reflect the extent of bowel


ischemia. In mild cases, patients may have diarrhea (usually bloody)
without abdominal pain. With more severe ischemia, intense abdominal
pain (often out of proportion to the clinical examination), tenderness,
fever, and leukocytosis are present. Peritonitis and/or systemic toxicity
are signs of full-thickness necrosis and perforation.

The diagnosis of ischemic colitis is often based upon the clinical history
and physical examination. Plain films may reveal thumb printing, which
results from mucosal edema and submucosal hemorrhage. CT often
shows nonspecific colonic wall thickening and pericolic fat stranding.
Angiography is usually not helpful because major arterial occlusion is
rare. Although sigmoidoscopy may reveal characteristic dark,
hemorrhagic mucosa, the risk of precipitating perforation is high. For
this reason, sigmoidoscopy is relatively contraindicated in any patient
with significant abdominal tenderness. Contrast studies (Gastrografin or
barium enema) are similarly contraindicated during the acute phase of
ischemic colitis.

Treatment of ischemic colitis depends upon clinical severity. Unlike


ischemia of the small bowel, the majority of patients with ischemic
colitis can be treated medically. Bowel rest and broad-spectrum
antibiotics are the mainstay of therapy, and 80% of patients will recover
with this regimen. Hemodynamic parameters should be optimized,
especially if hypotension and low flow appear to be the inciting cause.
Long-term sequelae include stricture (10 to 15%) and chronic segmental
ischemia (15 to 20%). Colonoscopy should be performed after recovery
to evaluate strictures and to rule out other diagnoses such as
inflammatory bowel disease or malignancy. Failure to improve after 2 to
3 days of medical management, progression of symptoms, or
deterioration in clinical condition are indications for surgical exploration.
In this setting, all necrotic bowel should be resected. Primary
anastomosis should be avoided. Occasionally, repeated exploration (a
second-look operation) may be necessary.

(Q.173) Youngs Prostatectomy is a

(a) Retropubic Prostatectomy

(b) Perineal Prostatectomy

(c) Transvesical Prostatectomy

(d) Radical Prostatectomy

Your Response :

Correct Answer : b

Exp: Radical perineal prostatectomy (RPP) was described in 1905 by


Young.It was the first method used to remove the prostate as part of
cancer therapy.

In 1947, Millin first describedradical retropubic prostatectomy(RRP).He


suggested the procedure as an alternative to RPP, on the grounds that
patients often had pelvic lymph node metastases at diagnosis. As
expertise in performing RRP improved, the importance ofpelvic lymph
node dissection(PLND) for staging became evident. Over time, RRP
became the most common method of radical prostatectomy.

Most surgeons favor either a retropubic or a perineal approach to radical


prostatectomy, depending on the type of training they have received.
(Keller et al have experimented with a perineal approach for PLND and
have published encouraging results.)

Advantages of RPP over RRP include the following:

A small, hidden incision for better cosmesis

Avoidance of major muscle groups

Less pain and patient convalescence

Faster return to work and strenuous activities

Fewer adverse cardiovascular effects because fluid shifts are reduced

Less blood loss

Shorter operating time and duration of hospitalization

Excellent posterior exposure to limit positive margins posteriorly,


laterally, and apically

Precise watertight anastomosis performed under direct vision

Easier for patients who are obese

Avoidance of scar tissue from previous abdominal surgery

Better visualization of the prostatic apex The improved visualization of


the apex facilitates avoidance of positive apical margins, sparing of
neurovascular bundles, and visualization of the membranous urethra

Although most urologists believe that radical prostatectomy is the most


effective means of curing clinically localized prostate cancer, the
associated surgical morbidity has compromised patients overall quality
of life and hindered their acceptance of the procedure. Accordingly,
efforts have been made to reduce surgical morbidity and improve
postoperative quality of life by developing different surgical options for
prostatectomy

(Q.174) Couinaud Segment IX of liver belongs to:

(a) Left anatomical lobe (

(b) Left surgical lobe

(c) Right surgical lobe

(d) Right paracaval portion of posterior liver.


Your Response :

Correct Answer : d

Exp: D. Right paracaval portion of posterior liver REF : Hepatic


surgical anatomy Surg Clin N Am 84 (2004) 413435

Couinauds liver segmentation

The Couinard segmentation system is based on the distribution in the


liver of both the portal vein and the hepatic veins and shows a specic
consideration for the caudate lobe. Fissures of the three hepatic veins
(portal scissurae) divide the liver into four sectors (segments), lateral and
paramedian, on the right and left sides, respectively. The planes
containing portal pedicles are called hepatic scissurae. Eight segments
are described, one for the caudate lobe (segment I), three on the right
(segments II, III, and IV), and four on the left (segments V, VI, VII, and
VIII). In general, the segments of this classication correspond to
subsegments of Healey and Schroy. Couinauds system of liver
segmentation diers from Healey and Schroys system in several ways,
however. According to Couinaud, a subdivision of segment IV and the
caudate lobe into two parts is not justied. Furthermore, Couinaud
asserted that a study of organogenesis and comparative anatomy suggests
that the umbilical ssure is the hepatic scissura between segments III and
IV. For Healy and Schroy, however, the umbilical ssure is the plane of
separation between territories of biliary (and consequently portal vein)
branches between the medial and lateral segment of the left lobe. At the
close of the last century, several investigators, including Couinaud and
coworkers, used the term segment IX for an area of the dorsal sector of
the liver close to the IVC. In 2002, however, Abdalla, Vauthey and
Couinaud wrote, Because no separate veins, arteries, or ducts can be
dened for the right paracaval portion of the posterior liver and because
pedicles cross the proposed division between the right and left caudate,
the concept of segment IX is abandoned. The genesis and death of
segment IX is found in articles by Couinaud and other investigators

(Q.175) The treatment of cecal volvulus is:

(a) Observation alone

(b) Colonoscopic detorsion, Bowel prep. And elective resection

(c) Operative detorsion & cecopexy

(d) Operative detorsion, right hemicolectomy with ileocolostomy

Your Response :
Correct Answer : d

Exp: Cecal volvulus results from non fixation of the right colon. Rotation
occurs around the ileocolic blood vessels and vascular impairment occurs
early. Plain x-ray of the abdomen shows characteristic kidney-shaped, air
filled structure in the left upper quadrant (opposite the site of
obstruction), and a Gastrografin enema confirms obstruction at the level
of the volvulus.

Unlike sigmoid volvulus, cecal volvulus can almost never be detorsed


endoscopically. Moreover because vascular compromise occurs early in
the course of cecal volvulus, surgical exploration is necessary when the
diagnosis is made.

Right hemicolectomy with primary ilecolic anastomosis can be


performed safely and prevents recurrence. Simple detorsion or detorsion
and cecopexy are associated with a high rate of recurrence.

(Q.176) What is your diagnosis?

(a) RBBB

(b) WPW syndrome

(c) LBBB

(d) Hyperkalemia

Your Response :

Correct Answer : c

Exp: In this ECG there is wide QRS complex (>0.11 sec) and there is W
pattern in V1 and M pattern in V6. LVD is seen in MI, VSD.

(Q.177) The operation where in the stump of the stomach is directly anastomosed to the
stump of the duodenum is called

(a) Poly a gastrectomy

(b) Hofmeister gastrectomy


(c) Billroth I gastrectomy

(d) Billroth II gastrectomy

Your Response :

Correct Answer : c

Exp: In Bilroth type 1 gastrectomy the lower half of stomach is removed and
the cut stomach is anastomosed to first part of duodenum.

In type II two third of stomach is removed, duodenal stump is closed and


stomach is anastomosed to the jejunum.

(Q.178) Most common parotid tumor:

(a) Pleomorphic adenoma.

(b) Adenolymphoma.

(c) Acinic cell tumor.

(d) Adenoidcystic carcinoma.

Your Response :

Correct Answer : a

Exp: PLEOMORPHIC ADENOMA

n Initially described as a mixed tumor

n Accounts for 75% of parotid and 50% submandibular tumors

n Believed to have both epithelial and mesothelial elements

n Now appears to arise from ductal myoepithelial cells

n Male : female ratio approximately equal

n May undergo malignant change but risk is small

n Requires excision with 5-10 mm margin as local implantation of


cells can lead to recurrence

n Treatment of choice for pleomorphic adenoma is superficial


parotidectomy but, if the deep lobe of parotid is involved Total
parotidectomy is done.

(Q.179) Troisiers is:


(a) Left supraclavicular lymphadenopathy

(b) Right supraclavicular lymphadenopathy

(c) Left deep neck lymphadenopathy

(d) Right deep neck lymphadenopathy

Your Response :

Correct Answer : a

Exp: A (Left supraclavicular lymphadenopathy)

Troisiers sign Metastasis to left supraclavicular group of node (e.g. as


in Ca stomach)

Trousseaus In advanced cancers (e.g. gastric cancer) non-metastatic


sign effects of malignancy are seen, particularly
thrombophlebitis (migratory).

Trousseaus In hypoparathyroidism (with hypocalcemia)


sign Carpopedal spasm can be induced by occlusion of the
arm with a pressure cuff for 3 minutes.

180. B (Pectoralis major) (Ref. Bailey and Love, 24th ed., pg


340)

Mastectomy is now only strictly indicated for large tumors, central tumor
beneath or involving nipple, multifocal disease, and local recurrence or
for patient preference.

SIMPLE RADICAL MODIFIED RADICAL


MASTECTOMY MASTECTOMY (PATEYS)
(HALSTED)

Only breast and Breast Whole breast

Axillary tail of Axillary Lymph Large portion of Skin


breast nodes including nipple

No Dissection of Pectoralis major All fat, fascia and lymph


axilla nodes of axilla

Pectoralis minor Pectoral is minor either


divided or retracted intercostal
brachial nerves divided.

Structures Spared in MRM:-

- Pectoral is major - Axillary vein - Nerve to serratus anterior


and latissimus dorsi

(Q.180) In Pateys modified mastectomy which of the following is preserved?

(a) Intercostobrachial nerve

(b) Pectoralis major

(c) Pectoralis minor

(d) Axillary fascia

Your Response :

Correct Answer : b

Exp: A (Cloquets hernia) (Ref. Bailey and Love surgery 24th ed., 1282)

FEMORAL HERNIA

Types of femoral hernia

Laugiers Hernia through a gap in the lacunar (Gimbernats) ligament

Femoral
Hernia

Naraths Occurs only in patients with CDH (DDH) and is due


to lateral displacement of the psoas muscle. It lies hidden
femoral behind the femoral vessels.
hernia

Cloquets Sac lies under the fascia covering the Pectineus muscle.

femoral
hernia

(Q.181) Hernia that lies under the fascia of Pectines muscle is:

(a) Cloquets hernia.

(b) Laugiers hernia.

(c) Naraths hernia.

(d) Obturator hernia.

Your Response :

Correct Answer : a
Exp: (Ref. Bailey and Love surgery 24th ed., 1282)

FEMORAL HERNIA

Types of femoral hernia

Laugiers Hernia through a gap in the lacunar (Gimbernats) ligament

Femoral
Hernia

Naraths Occurs only in patients with CDH (DDH) and is due


to lateral displacement of the psoas muscle. It lies hidden
femoral behind the femoral vessels.
hernia

Cloquets Sac lies under the fascia covering the Pectineus muscle.

femoral
hernia

(Q.182) Most common cause of supracellar enlargement with calcification in brain in


children on C.T. scan is

(a) Craniopharyngioma

(b) Astrocytoma

(c) Meningioma

(d) Supracellar tuberculoma

Your Response :

Correct Answer : a

Exp: Craniophyaryngioma

Craniopharyngiomasare relatively benign (WHO grade I) neoplasms


which typically arise in the sellar/suprasellar region. They account for
~1-5% of primary brain tumours, and can occur anywhere along the
infundibulum (from floor of thethird ventricle, to thepituitary gland).

There are two pathological types, which are said to differ not only in
appearances, but also in prognosis and epidemiology. Whether or not
they represent distinct entities or a spectrum of morphology remains a
little controversial They are:

Adamantinomatous(paediatric)
papillary(adult)

mixed: ~15%, but share imaging and prognosis similar to


adamantinomatous

In the vast majority of cases, craniopharyngiomas have a significant


suprasellar component (95%), with most involving both the suprasellar
and intrasellar spaces (75%). A minority are purely suprasellar (20%),
whereas purely intrasellar location is quite uncommon (<5%), and may
be associated with expansion of the pituitary fossa3,7. Larger tumours can
extend in all directions, frequently distorting the optic chiasm, or
compressing the midbrain with resultingobstructive hydrocephalus.

Occasionally, craniopharyngiomas appear as intraventricular,


homogeneous, soft-tissue masses without calcification (papillary
subtype). The third ventricle is a particularly common location.

Rare or ectopic locations reported include: nasopharynx, posterior fossa,


extension down the cervical spine.

Adamantinomatous

Typically adamantinomatous craniopharyngiomas have a lobulated


contour as a result of usually multiple cystic lesions. Solid components
are present, but usually form a relatively minor component of the mass,
and enhance vividly on both CT and MRI. Overall, calcification is very
common, but this is only true of the adamantinomatous subtype (90% are
calcified)

These tumour have a predilection to be large, extending superiorly into


the third ventricle, and encasing vessels, and even being adherent to
adjacent structures

CT

Cysts

Typically large and a dominant feature

Near CSF density

Solid component

Soft tissue density

Vivid enhancement

Calcification

Seen in 90%
Typically stippled and often peripheral in location

MRI

Cysts: variable but ~80% are mostly or partly T2 hyperintense

T1:iso- to hyperintense to brain (due to high protein content machinery


oil cysts)

Solid component

T1 C+ (Gd):vivid enhancement

T2:variable or mixed

calcification

difficult to appreciate on conventional imaging

susceptible sequences may better demonstrate calcification

MR angiography:may demonstrate displacement of the A1 segment of


the anterior cerebral artery

MR spectroscopy:cyst contents may show a broad lipid spectrum, with


an otherwise flat baseline

(Q.183) Hyperdense basal ganglia and hypodensity of the white matter on CT scan is
diagnostic of

(a) Alexanders Disease

(b) Krabbes Disease

(c) Canavans Disease

(d) Metachromatic leukodystrophy

Your Response :

Correct Answer : b

Exp: Craniopharyngiomasare relatively benign (WHO grade I) neoplasms


which typically arise in the sellar/suprasellar region. They account for
~1-5% of primary brain tumours, and can occur anywhere along the
infundibulum (from floor of thethird ventricle, to thepituitary gland).

There are two pathological types, which are said to differ not only in
appearances, but also in prognosis and epidemiology. Whether or not
they represent distinct entities or a spectrum of morphology remains a
little controversial They are:
Adamantinomatous(paediatric)

papillary(adult)

mixed: ~15%, but share imaging and prognosis similar to


adamantinomatous

In the vast majority of cases, craniopharyngiomas have a significant


suprasellar component (95%), with most involving both the suprasellar
and intrasellar spaces (75%). A minority are purely suprasellar (20%),
whereas purely intrasellar location is quite uncommon (<5%), and may
be associated with expansion of the pituitary fossa3,7. Larger tumours can
extend in all directions, frequently distorting the optic chiasm, or
compressing the midbrain with resultingobstructive hydrocephalus.

Occasionally, craniopharyngiomas appear as intraventricular,


homogeneous, soft-tissue masses without calcification (papillary
subtype). The third ventricle is a particularly common location.

Rare or ectopic locations reported include: nasopharynx, posterior fossa,


extension down the cervical spine.

Adamantinomatous

Typically adamantinomatous craniopharyngiomas have a lobulated


contour as a result of usually multiple cystic lesions. Solid components
are present, but usually form a relatively minor component of the mass,
and enhance vividly on both CT and MRI. Overall, calcification is very
common, but this is only true of the adamantinomatous subtype (90% are
calcified)

These tumour have a predilection to be large, extending superiorly into


the third ventricle, and encasing vessels, and even being adherent to
adjacent structures

CT

Cysts

Typically large and a dominant feature

Near CSF density

Solid component

Soft tissue density

Vivid enhancement

Calcification
Seen in 90%

Typically stippled and often peripheral in location

MRI

Cysts: variable but ~80% are mostly or partly T2 hyperintense

T1:iso- to hyperintense to brain (due to high protein content machinery


oil cysts)

Solid component

T1 C+ (Gd):vivid enhancement

T2:variable or mixed

calcification

difficult to appreciate on conventional imaging

susceptible sequences may better demonstrate calcification

MR angiography:may demonstrate displacement of the A1 segment of


the anterior cerebral artery

MR spectroscopy:cyst contents may show a broad lipid spectrum, with


an otherwise flat baseline

(Q.184) Which of the following is a feature of malignant GIST of gastrointestinaltract ?

(a) Lobulated contour

(b) Striking absence of regional lymphadenopathy

(c) Endophytic growth

(d) Tumorhomogenicity

Your Response :

Correct Answer : a

Exp: Lobulated contour

Twenty-four patients with gastric mesenchymal tumors who underwent


endoscopic ultrasonography (EUS) and surgical treatment were enrolled.
GISTs were defined as c-kit (CD117)-positive tumors, leiomyomas as
desmin-positive and c-kit-negative tumors, and schwannomas as S-100-
positive and c-kit-negative tumors. Invasion to adjacent organs or more
than 20 mitotic counts per 50 high power fields indicated malignancy.
RESULTS:

There were 19 GISTs, three leiomyomas, and two schwannomas. All five
malignant tumors were GISTs. A marginal halo was found in 12 of 19
GISTs and in both of the schwannomas, but not in any of the three
leiomyomas. The echogenicities of GISTs were low but higher than that
of the normal proper muscle layer, whereas those of leiomyomas and
schwannomas were usually low. Lobulation of the tumor surface was
documented only in GISTs, particularly in malignant ones. The tumor
doubling time of a malignant GIST was 9.3 months, and that of six
benign GISTs was 18.7 months (range = 10.7-28.0 months).

CONCLUSION:

Marginal halo and relatively higher echogenicity on EUS might suggest


GIST. Marginal lobulation and a short doubling time may be signs of a
malignant GIST.

(Q.185) Modality of choice for diagnosis of congenital hypertrophic pyloric stenosis is

(a) Scintigraphy

(b) MRI

(c) CT Scan

(d) Ultrasonography

Your Response :

Correct Answer : d

Exp: Ultrasonography

ltrasound is currently the imaging modality of choice that reliably


establishes the diagnosis of hypertrophic pyloric stenosis. There are
various sonographic parameters that can be used to arrive at the
diagnosis and include pyloric length, pyloric diameter, muscle thickness
and also pyloric volume. Pyloric muscle thickness of 3 mm or greater,
pyloric canal length of 17 mm or greater and the absence of the passage
of a peristaltic wave through the pylorus, during the period of scanning
are the diagnostic ultrasound criteria. Classically seen is:

1. A ring on transverse section, resembling a doughnut or a bulls-eye


or a target with the echogenic pyloric canal in the center and
surrounded by the hypertrophied pyloric muscle.

2. An elongated and narrowed pyloric channel.

3. Prolapse of the redundant mucosa into the antrum creates an antral


nipple sign.

4. In many cases, an elongated pylorus that lies adjacent to and just


below the gall bladder provides the initial clue to the diagnosis.

5. A pyloric cervix sign has also been demonstrated and refers to the
appearance of hypertrophied pyloric musculature in longitudinal section
impinging upon a fluid filled antrum.

6. The ultrasonic double track sign is also appreciated here; however it


is not specific for pyloric stenosis and can be seen in infantile
pylorospasm as well.

7. Other features that may be seen are retrograde or hyper-peristaltic


contractions, though none of the contractions pass through the pylorus.

The only other imaging study that may be useful for diagnosis of this
condition is upper GI studies with barium. This study demonstrates the
narrowed pyloric channel as a thin stream of barium passes through it
[the string sign]. CT and MRI are not performed to diagnosis this
condition.

Surgery [pyloromyotomy] is the treatment of choice in these patients.

(Q.186) All is true regarding Digital Radiographyexcept ?

(a) Built in image capture plates used (No cassette required)

May offer potential for better image quality with lower radiation dose
(b)
than CR.

(c) Shorter turnaround time for viewing image.

(d) Lower cost

Your Response :

Correct Answer : d

Exp: Lower cost.

COMPUTED DIGITAL RADIOGRAPHY (DR)


RADIOGRAPHY(CR)

Prepare room Prepare room

Load cassette Position patient

Position patient Position tube


Position tube Perform exposure

Perform exposure Image is sent to viewing station

Transport cassette Assess image quality

Process cassete Wrap up appointment with patient.

Assess image quality

Wrap up appointment with


patient.

A PSP plate within the cassette Built in image capture plates used (No
is exposed. cassette required)

Latent image is captured in the Large area, flat panel detectors with
plate as electrons in the integrated TFT readout mechanisms,
phosphor are excited when Integrated PSP plate scanning
exposed to radiation. mechanism, or optic lens used to
translate the analog image to digital
Cassette is placed in a reader to image.
capture and analyse image data.

Laser and analog to digital


converter translates signal to
digital binary code.

Opprtunities to improve image Opprtunities to improve image


interpretation and diagnostic interpretation and diagnostic strength.
strength.
Potential for lower image noise and
Potential for lower image noise lower radiation exposure with
and lower radiation exposure appropriate system adjustments.
with appropriate system
adjustments. May offer potential for better image
quality with lower radiation dose than
CR.

Unlimited manipulation and Relatively faster workflow due to


positioning of image receptor elimination of cassettes.
for cross table projections is
possible (useful in trauma) Shorter turnaround time for viewing
image.

Freeing of staff time.

Slower, more complex Higher costs.


workflow.

Possibility of repetitive motion


injuries due to longterm cassette
handling

(Q.187) Halo sign is characteristically seen in ?

(a) Tuberculosis

(b) Aspergillosis

(c) Sarcoidosis

(d) Silicosis

Your Response :

Correct Answer : b

Exp: Aspergillus

SIGN / SPECIFIC FEATURE SEEN IN

Meniscus / Moon/ Air crescent / Hydatid cyst of lung


Double arch sign

Cumbo sign / Water lilly /


Camalotte sign

Serpent sign / Rising sun sign /


Empty cyst sign

Popcorn calcification Hamartoma

Mediastinal nodes of histoplasmosis

Westermark sign, Haptons Pulmonary thrombo-embolism


hump, Palla sign

Fleishner lines, Felsons sign

Sail sign, Mulvay Wave sign, Thymic enlargement


Notch sign

Comet tail sign Rounded atelectasis

Golden S sign RUL collapse secondary to a central


mass

Luftsichel sign LUL collapse

Broncholobar sign LLL collapse


Ring around artery sign, Pneumo-mediastinum
Continuous diaphragm sign

Tubular artery sign, Double


bronchial wall sign

V sign of Naclerio, Spinnaker sail


sign

Deep sulcus sign, Visceral pleural Pneumothorax


line

Thumb sign Epiglottitis

Steeple sign Croup

Air crescent sign Aspergilloma

Monod sign

Bulging fissure sign Klebsiella pneumonia

Batwing sign Pulmonary edema on CXR

Collar sign Diaphragmatic rupture

Dependant viscera sign

Feeding vessel sign Pulmonary septic emboli

Finger in glove sign ABPA

Halo sign Aspergillosis

Head cheese sign Subacute hypersensitivity


pneumonitis

Juxtaphrenic peak sign RUL atelectasis

Reversed halo sign Cryptogenic organized pneumonia

Saber sheath trachea COPD

Sandstorm lungs Alveolar microlithiasis

Signet ring sign Bronchiectasis

Superior triangle sign RLL atelectasis

Split pleura sign Empyema


Tree in bud sign on HRCT Endobronchial spread in TB

(Q.188) Classification systems for Aortic dissection are ?

(a) Crawford

(b) DeBakey

(c) Stanford

(d) All of above

Your Response :

Correct Answer : d

Exp: All of above

CLASSIFICATION SYSTEMS FOR AORTIC DISSECTION

Classification system

Site of dissection Crawford DeBakey Stanford

Both ascending and Proximal Type I Type A


descending aorta dissections

Ascending aorta and arch Proximal Type II Type A


only dissections

Descending aorta only Distal Type III IIIa Type B


(distal to left subclavian dissections limited to thoracic
artery) aorta

IIIbextends to
abdominal aorta

In addition to the above, all dissections are classified as acute if the


duration of symptoms is shorter than 14d and chronic if longer than this.

(Q.189) Prodromal symptoms of eclampsia are all EXCEPT

(a) Severe Headache

(b) Vomiting

(c) Epigastric pain

(d) Convulsions
Your Response :

Correct Answer : d

Exp: Convulsions

Preeclampsia and eclampsia may also be accompanied by several other


clinical symptoms. These include an unremitting headache, visual
disturbances, right upper-quadrant pain, midepigastric pain, nausea and
vomiting, oliguria, and shortness of breath

(Q.190) Following is true about oxytocin

(a) It is decapeptide

(b) Synthesized in posterior lobe of pituitary gland

(c) Alcohol stimulates its release

(d) It has half life of 3-4 minutes

Your Response :

Correct Answer : d

Exp: It has half life of 3-4 minutes

Produced by the hypothalamus and stored and secreted by the posterior


pituitary gland, oxytocin acts primarily as a neuromodulator in the brain.

Oxytocin plays an important role in the neuroanatomy of intimacy,


specifically in sexual reproduction of both sexes, in particular during and
after childbirth. It is released in large amounts after distension of the
cervix and uterus during labor, facilitating birth, maternal bonding, and,
after stimulation of the nipples, lactation. Both childbirth and milk
ejection result from positive feedback mechanisms. Recent studies have
begun to investigate oxytocin's role in various behaviors, including
orgasm, social recognition, pair bonding, anxiety, and maternal
behaviors. For this reason, it is sometimes referred to as the "bonding
hormone".

It has a half-life of typically about three minutes in the blood when given
intravenously.

Oxytocin is a peptide of nine amino acids (a nonapeptide)

(Q.191) All are features of infant born to diabetic mothers EXCEPT

(a) Obesity
(b) Learning Disability

(c) Ketotic hypoglycaemia

(d) Future Diabetes Mellitus

Your Response :

Correct Answer : b

Exp: Learning Disability

Clinical manifestations

Characteristics of pregnancy

- Polyhydramnios - Preterm labor - Restricted growth

- Increased rate of congenital abnormalities (hyperglycemia during first


trimester can be teratogenic)

- Increased fetal mortality rate (mainly thought to be due to lactic


acidosis)

Physical characteristics

- Macrosomia - Enlarged viscera

- Plump plethoric facies (resembles facies from patients treated with


corticosteroids)

- Congenital anomalies. Cardiac malformations (ventricular or atrial


septal defect, transposition of the great vessels, truncus arteriosus,
double-outlet right ventricle, coarctation of the aorta) and lumbosacral
agenesis are most common.

Specific systems most often affected

a) Central Nervous system

The infants tend to have an increased startle reflex and are tremulous
during the 1st 3 days of life, although hypotonia, lethargy, and poor
sucking may also occur. Seizures, if they occur are due to hypoglycemia
or hypocalcemia.

b) Pulmonary system

Tachypnea develops in many infants of diabetic mothers during the 1st 2


days of life

c) Cardiovascular system

Cardiomegaly is common (30%), and heart failure occurs in 510% of


infants of diabetic mothers. Asymmetric septal hypertrophy may occur
and become manifested similar to idiopathic hypertrophic subaortic
stenosis.

d) Hematologic system

Polycythemia.

e) Gastrointestinal system

Small left colon syndrome and decreased intestinal motility may be


present. Prognosis

Physical development is normal, but oversized infants may be


predisposed to childhood obesity that may extend into adult life.
symptomatic hypoglycemia increases the risk, as does maternal
ketonuria. The incidence of diabetes mellitus in infants of diabetic
mothers is increased in comparison to that of the general population.

(Q.192) Commonest cause of First trimester abortion is

(a) Monosomy

(b) Trisomy

(c) Triploidy

(d) Aneuploidy

Your Response :

Correct Answer : b

Exp: Trisomy

Causes of First Trimester Abortion

1. Defective Germ Plasm

a. Chromosomal Abnormalities

i. Most common among this is Autosomal Trisomy

b. Congenital Malforamtion

c. Blighted Ovum

2. Chromosomal Abonormalities

3. Hormonal deficiency
4. Trauma

5. Acute Infection

Causes of Mid Trimester Abortion

1. Cervical Incompetence 2. Uterine Malformation - bicornuate or


Septate uterus 3. Uterine Fibroid

4. Low implantation of Placenta 5. Twins 6.


Hydramnios

(Q.193) Bishops score includes following EXCEPT

(a) Effacement of cervix

(b) Contractions of uterus

(c) Dilatation of cervix

(d) Station of head

Your Response :

Correct Answer : b

Exp: Contractions of uterus

The Bishop score

Cervical feature Bishop score

0 12 34 56

Dilation (cm) 0 1 2 3

Effacement(%) 030 4050 6070 80

Station (relative to ischial 3 2 1/0 +1/+2


spines)

Consistency Firm Medium Soft

Position Posterior Mid Anterior

The modified Bishop score

Cervical feature Modified Bishop score

0 1 2 3
Dilation (cm) <1 12 24 > 4

Length of cervix (cm) >4 24 12 < 1

Station (relative to ischial 3 2 1/0 +1/+2


spines)

Consistency Firm Average Soft

Position Posterior Mid/anterior

(Q.194) Which of the following is the most dangerous cardiac lesion in pregnancy ?

(a) Mitral stenosis

(b) Mitral regurgitation

(c) Ventricular septal defect

(d) Eisenmengers syndrome

Your Response :

Correct Answer : d

Exp: Eisenmengers syndrome

Specific Congenital or Acquired Cardiac Lesions

Specific congenital or acquired cardiac lesions are classified as low,


intermediate, or high risk during pregnancy

Box 1: Maternal Cardiac Lesions and Risk of Cardiac Complications


During Pregnancy

Low Risk

Atrial septal defect

Ventricular septal defect

Patent ductus arteriosus

Asymptomatic aortic stenosis with low mean gradient (<50 mm Hg) and
normal left ventricle function (EF >50%)

Aortic regurgitation with normal left ventricle function and NYHA


functional class I or II

Mitral valve prolapse (isolated or with mild-to-moderate mitral


regurgitation and normal left ventricle function)

Mitral regurgitation with normal left ventricle function and NYHA class
I or II

Mild-to-moderate mitral stenosis (mitral valve area >1.5 cm2, mean


gradient <5 mm Hg) without severe pulmonary hypertension

Mild/moderate pulmonary stenosis

Repaired acyanotic congenital heart disease without residual cardiac


dysfunction

Intermediate Risk

Large left-to-right shunt

Coarctation of the aorta

Marfan syndrome with a normal aortic root

Moderate-to-severe mitral stenosis

Mild-to-moderate aortic stenosis

Severe pulmonary stenosis

High Risk

Eisenmenger's syndrome

Severe pulmonary hypertension

Complex cyanotic heart disease (tetralogy of Fallot, Ebsteins anomaly,


truncus arteriosus, transposition of the great arteries, tricuspid atresia)

Marfan syndrome with aortic root or valve involvement

Uncorrected severe aortic stenosis with or without symptoms

Uncorrected severe mitral stenosis with NYHA functional class II-IV


symptoms

Aortic and/or mitral valve disease (stenosis or regurgitation) with


moderate-to-severe left ventricle dysfunction (EF <40%)

NYHA class III-IV symptoms associated with valvular disease or


cardiomyopathy of any etiology

History of prior peripartum cardiomyopathy


(Q.195) Chromosomal complement of primary oocyte is

(a) 23 Y

(b) 23 X

(c) 46 XX

(d) 46 XY

Your Response :

Correct Answer : c

Exp: 46 XX

Cell type Ploidy/chromosomes chromatids Process Time of


completion

Oogonium diploid/46(2N) 2C Oocytogenesis third


trimester

primary diploid/46(2N) 4C Ootidogenesis (meiosis1) Dictyate in


Oocyte (Folliculogenesis prophase I
46XX for up to 50
years

secondary haploid/23(1N) 2C Ootidogenesis(meiosis2) Halted in


Oocyte metaphase
II until
fertilization

Ootid haploid/23(1N) 1C Ootidogenesis (meiosis 2) Minutes


after
fertilization

Ovum haploid/23(1N) 1C

(Q.196) Asymmetrical fetal growth restriction is associated with

(a) Chromosomal aberration

(b) Viral infection

(c) Idiopathic

(d) Placental insufficiency

Your Response :
Correct Answer : d

Exp: Placental insufficiency

Specific Distinctions Between Symmetric and Asymmetric Intrauterine


Growth Restriction (IUGR).

Symmetric IUGR Asymmetric IUGR

Incidence 20%-30% 70%-80%

Period of growth Begins first or second Begins third trimester


restriction trimester

Physical Small head and Large head size relative to


characteristics abdominal size small abdomen

Pathophysiology Impaired cellular Impaired cellular


embryonic division hypertrophy

Impaired cellular Decreased cell size


hyperplasia
hypertrophy

Decreased cell number


size

Etiology Mostly intrinsic: Mostly extrinsic: placental


chromosomal and maternal vascular
abnormalities and factors (eg, placental
congenital malformations insufficiency)

Drugs

Infections

Early-onset severe
preeclampsia

Preeclampsia <30 wk
superimposed with
chronic hypertension

Outcome Greater morbidity and Lower morbidity and


mortality mortality

(Q.197) Shoulder dystocia is common with

(a) Preterm birth


(b) Placenta previa

(c) Postdated pregnancy

(d) Polyhydramnios

Your Response :

Correct Answer : c

Exp: Postdated pregnancy

Risk Factors for Shoulder Dystocia

Maternal

Abnormal pelvic anatomy Gestational diabetes Post-dates


pregnancy

Previous shoulder dystocia Short stature

Fetal

Suspected macrosomia

Labor related

Assisted vaginal delivery (forceps or vacuum) Protracted active


phase of first-stage labor

Protracted second-stage labor

(Q.198) Commonest cause of Maternal Mortality in India is

(a) Eclampsia

(b) Sepsis

(c) Obstructed labour

(d) Haemorrhage

Your Response :

Correct Answer : d

Exp: Haemorrhage

Causes of maternal deaths

1. Direct Causes: 81%


Severe Bleeding 25%

Sepsis 15%

Unsafe abortions 13%

Eclampsia 12%

Obstructed Labour 8%

Other direct causes * 8%

2. Indirect Causes ** 19%

* Other direct causes: Ectopic pregnancy, embolism,


anaesthesia related.** Indirect Causes: Malaria, Anaemia,
heart diseases.

(Q.199 A child is below the third percentile for height. His growth velocity is normal, but
) chronological age is more than skeletal age. The most likely diagnosis is:

(a) Constitutional delay in growth

(b) Genetic short stature

(c) Primordial dwarfism

(d) Hypopituitarism

Your Response :

Correct Answer : a

Exp Constitutional delay in growth. (Ref. OP Ghai 6th /pg. 50)


:
Features Constitutiona Genetic Primordia Hypopituitaris Nutritional
l delay short l dwarfism m
stature dwarfism

Birth weight N More - N Slow weight


gain

Height N Failure to < 3 SD Delay after one Less than


achieve than year of age expected
expected normal
height

Average N and ultimate < Normal Not altered Slow growth Weight and
growth growth is markedly height
concurrently
velocity normal affected

Bone age viz Bone age < Bone age Bone age Bone age < Bone age <
chronologica chronological normal for normal or chronological chronologica
l age age chronologica moderately age l age
l age delayed

(Q.200) Pseudo-Darier sign is seen in

(a) Urticaria pigmentosa

(b) Congenital smooth muscle hamartoma

(c) Xanthogranuloma

(d) Dermato fibroma

Your Response :

Correct Answer : b

Exp: Congenital smooth muscle hamartoma

Conditions Associated With Darier's Sign

1.Cutaneous mastocytosis: In urticaria pigmentosa, the most frequent


clinical form of cutaneous mastocytosis, Darier's sign was present in
94% of cases.

2. Leukemia cutis: Leukemia cutis occurs in 25-30% of infants with


congenital leukemia and is frequently associated with acute myeloid
leukemia than with acute lymphoblastic leukemia. Urticaria-pigmentosa-
like lesions have been reported in acute lymphoblastic leukaemia.

3.Juvenile xanthogranuloma: It is the most common form of non-


Langerhans cell histiocytosis. Nagayo et al. reported Darier's sign in this
disorder.

4.Histiocytosis X : Foucar et al. described positive Darier's sign in a


patient with 'mast cell rich variant' of histiocytosis X.

5.Lymphoma: On rare instancs, Darier's sign has been reported in


cutaneous large T-cell lymphoma and in non-Hodgkin's lymphoma.

Differential Diagnosis

Pseudo-Darier's sign : It is a transient piloerection and elevation or


increased induration of a lesion induced by rubbing and is observed in
congenital smooth muscle hamartomas. A positive pseudo-Darier's sign
can be helpful in clinically distinguishing congenital smooth muscle
hamartoma from congenital hairy nevus.

Dermatographism ("skin writing"): It is a form of physical urticaria


that consists of local erythema due to capillary vasodilatation, followed
by edema and a surrounding flare due to axon reflex induced dilation of
arterioles, which is observed after the firm stroking of skin. The cause of
this phenomenon is thought to be hypersensitivity of the mast cells rather
than an increase in the mast cells, as observed in mastocytosis.

Technology Partner:Yoctel Solutions (P) Ltd.

Vous aimerez peut-être aussi